Monday, April 17, 2017

SchweserNotes CFA Level 2 Book

http://www.freeaccountingbooks.com/schwesernotes-cfa-level-2-book/


Free Accounting books provides unlimited PDF books, notes & guides of #Financial, #cost and #management #accounting, #ACCA, #CFA, #CIMA, #CPA, #CMA, #FRM and many more .......
#freeaccountingbooks.com
Book 1 – Ethical and Professional
Standards, Quantitative Methods,
and Economics
Readings and Learning Outcome Statements .......................................................... 6
Study Session 1 – Ethical and Professional Standards ............................................ 14
Study Session 2 – Ethical and Professional Standards: Application....................... 110
Self-Test – Ethical and Professional Standards ..................................................... 128
Study Session 3 – Quantitative Methods for Valuation ........................................ 138
Self-Test – Quantitative Methods for Valuation ................................................... 254
Study Session 4 – Economics for Valuation ......................................................... 260
Self-Test – Economics for Valuation .................................................................... 354
Formulas ............................................................................................................ 357
Appendices ........................................................................................................ 361
Index ................................................................................................................. 370
SCHWESERNOTES™ 2011 CFA LEVEL 2 BOOK 1: ETHICAL AND PROFESSIONAL STANDARDS,
QUANTITATIVE METHODS, AND ECONOMICS
©2010 Kaplan, Inc. All rights reserved.
Published in 2010 by Kaplan Schweser.
Printed in the United States of America.
ISBN: 978-1-4277-2747-3 / 1-4277-2747-3
PPN: 3200-0068
If this book does not have the hologram with the Kaplan Schweser logo on the back cover, it was distributed without permission of Kaplan
Schweser, a Division of Kaplan, Inc., and is in direct violation of global copyright laws. Your assistance in pursuing potential violators of this
law is greatly appreciated.
Required CFA Institute® disclaimer: “CFA® and Chartered Financial Analyst® are trademarks owned by CFA Institute. CFA Institute
(formerly the Association for Investment Management and Research) does not endorse, promote, review, or warrant the accuracy of the
products or services offered by Kaplan Schweser.”
Certain materials contained within this text are the copyrighted property of CFA Institute. The following is the copyright disclosure for
these materials: “Copyright, 2011, CFA Institute. Reproduced and republished from 2011 Learning Outcome Statements, Level 1, 2,
and 3 questions from CFA® Program Materials, CFA Institute Standards of Professional Conduct, and CFA Institute’s Global Investment
Performance Standards with permission from CFA Institute. All Rights Reserved.”
These materials may not be copied without written permission from the author. The unauthorized duplication of these notes is a violation of
global copyright laws and the CFA Institute Code of Ethics. Your assistance in pursuing potential violators of this law is greatly appreciated.
Disclaimer: The SchweserNotes should be used in conjunction with the original readings as set forth by CFA Institute in their 2011 CFA Level
2 Study Guide. The information contained in these Notes covers topics contained in the readings referenced by CFA Institute and is believed
to be accurate. However, their accuracy cannot be guaranteed nor is any warranty conveyed as to your ultimate exam success. The authors of
the referenced readings have not endorsed or sponsored these Notes.
©2010 Kaplan, Inc. Page 3
Welcome to the 2011 Level 2
SchweserNotes™
Thank you for trusting Kaplan Schweser to help you reach your goals. We are all very
pleased to be able to help you prepare for the Level 2 CFA Exam. In this introduction,
I want to explain the resources included with the SchweserNotes, suggest how you
can best use Schweser materials to prepare for the exam, and direct you toward other
educational resources you will find helpful as you study for the exam.
Besides the SchweserNotes themselves, there are many educational resources available at
Schweser.com. Just log in using the individual username and password that you received
when you purchased the SchweserNotes.
SchweserNotesTM
These consist of five volumes that include complete coverage of all 18 Study Sessions
and all Learning Outcome Statements (LOS) with examples, Concept Checkers
(multiple-choice questions for every topic review), and Challenge Problems for many
topic reviews to help you master the material and check your progress. At the end of
each major topic area, we include a Self-test. Self-test questions are created to be examlike
in format and difficulty in order for you to evaluate how well your study of each
topic has prepared you for the actual exam. The Level 2 SchweserNotes Package also
includes a sixth volume, the Level 1 Refresher, a review of important Level 1 material.
Practice Questions
To retain what you learn, it is important that you quiz yourself often. We offer CD,
download, and online versions of the SchweserPro™ QBank, which contains thousands
of Level 2 practice questions, item sets, and explanations. Quizzes are available for each
LOS, topic, or Study Session. Build your own exams by specifying the topics and the
number of questions you choose.
Practice Exams
Schweser offers six full 6-hour practice exams. Practice Exams Volume 1 and Volume 2
each contain three full 120-question exams. These are important tools for gaining the
speed and skills you will need to pass the exam. Each book contains answers with full
explanations for self-grading and evaluation. By entering your answers at Schweser.com,
you can use our Performance Tracker to find out how you have performed compared to
other Schweser Level 2 candidates.
Schweser Library
We have created reference videos, some of which are available to all SchweserNotes
purchasers. Schweser Library volumes range from 20 to 60 minutes in length and cover
such topics as: “Introduction to Item Sets,” “Hypothesis Testing,” “Foreign Exchange
Basics,” “Ratio Analysis,” and “Forward Contracts.” The full Schweser Library is
included with our 16-week live or online classes and with our video instruction (online
or CDs).
Page 4 ©2010 Kaplan, Inc.
Welcome to the 2011 Level 2 SchweserNotes™
Online Schweser Study Planner
Use your Online Access to tell us when you will start and what days of the week you can
study. The online Schweser Study Planner will create a study plan just for you, breaking
each study session into daily and weekly tasks to keep you on track and help you
monitor your progress through the curriculum.
Additional Resources
Purchasers of the Essential Self-Study or Premium Instruction Packages also receive
access to our Instructor-led Office Hours. Office Hours allow you to get your questions
about the curriculum answered in real time and to see others’ questions (and instructor
answers) as well. Office Hours is a text-based live interactive online chat with our team
of Level 2 experts. Archives of previous Office Hours sessions can be sorted by topic or
date and are posted shortly after each session.
The Level 2 CFA exam is a formidable challenge (70 topic reviews and 475+ Learning
Outcome Statements), and you must devote considerable time and effort to be properly
prepared. There is no shortcut! You must learn the material, know the terminology and
techniques, understand the concepts, and be able to answer 120 questions quickly and
(at least 70%) correctly. Fifteen to 20 hours per week for 20 weeks is a good estimate
of the study time required on average, but some candidates will need more or less time,
depending on their individual backgrounds and experience.
To help you master this material and be well prepared for the CFA Exam, we offer
several other educational resources, including:
Live Weekly Classroom Programs
We offer weekly classroom programs around the world. Please check Schweser.com for
locations, dates, and availability.
16-Week Online Classes
Our 16-Week Online Classes are available at New York time (6:30–9:30 pm) or London
time (6:00–9:00 pm) beginning in January. The approximate schedule for the 16-Week
Online Classes (3-hour sessions) is as follows:
Class # Class #
1) Exam Intro/Ethical Standards SS 1, 2 9) Equity SS 11, 12
2) Quantitative Methods SS 3 10) Equity SS 12
3) Economics for Valuation SS 4 11) Alternative Asset Valuation SS 13
4) Financial Reporting & Analysis SS 5 12) Fixed Income SS 14
5) Financial Reporting & Analysis SS 6 13) Fixed Income SS 15
6) Financial Reporting & Analysis SS 7 14) Derivatives SS 16
7) Corporate Finance SS 8 15) Derivatives & Portfolio Management SS 17
8) Corporate Finance & Equity SS 9, 10 16) Portfolio Management SS 18
Archived classes are available for viewing at any time throughout the season. Candidates
enrolled in the 16-Week Online Classes also have full access to supplemental on-demand
video instruction in the Schweser Library and an e-mail address to use to send questions
to the instructor at any time.
©2010 Kaplan, Inc. Page 5
Welcome to the 2011 Level 2 SchweserNotes™
Late Season Review
Whether you use self-study or in-class, online, or video instruction to learn the CFA
curriculum, a late-season review and exam practice can make all the difference. Our
most complete late-season review courses are our residence programs in Windsor,
Ontario (WindsorWeek) and Dallas/Fort Worth, Texas (DFW 5-day program). Each
covers the entire curriculum at all three levels. We also offer 3-day Exam Workshops in
many cities (and online) that combine curriculum review with an equal component of
hands-on practice with hundreds of questions and problem-solving techniques. Please
visit us at Schweser.com for complete listings and course descriptions for all our lateseason
review offerings.
Mock Exam and Multimedia Tutorial
On May 21, 2011, the Schweser Mock Exam will be offered live in many cities around
the world and as an online exam as well. The optional Multimedia Tutorial provides
extended explanation and topic tutorials to get you exam-ready in areas where you
miss questions on the Mock Exam. Please visit Schweser.com for a listing of cities and
locations.
How to Succeed
There are no shortcuts; depend on the fact that CFA Institute will test you in a way
that will reveal how well you know the Level 2 curriculum. You should begin early and
stick to your study plan. You should first read the SchweserNotes and complete the
Concept Checkers and Challenge Problems for each topic review. You should prepare
for and attend a live class, an online class, or a study group each week. You should take
quizzes often using SchweserPro Qbank and go back to review previous topics and Study
Sessions as well. At the end of each topic area, you should take the Self-test to check
your progress. You should finish the overall curriculum at least four weeks (preferably
five weeks) before the Level 2 exam so that you have sufficient time for Practice Exams
and for further review of those topics that you have not yet mastered.
I would like to thank Kevin Rygg, CFA Content Specialist; Stephanie Downey, Director
of Print Production; and Jeff Faas, Lead Editor, for their contributions to the 2011
Level 2 SchweserNotes for the CFA Exam.
Best regards,
Bijesh Tolia
Dr. Bijesh Tolia, CFA
VP and CFA Level 2 Manager
Kaplan Schweser
Page 6 ©2010 Kaplan, Inc.
Readings and
Learning Outcome Statements
Readings
The following material is a review of the Ethical and Professional Standards, Quantitative
Methods, and Economics principles designed to address the learning outcome statements set
forth by CFA Institute.
STUDY SESSION 1
Reading Assignments
Ethical and Professional Standards, CFA Program Curriculum, Volume 1, Level 2
(CFA Institute, 2011)
1. Code of Ethics and Standards of Professional Conduct page 14
2. Guidance for Standards I–VII page 14
3. CFA Institute Soft Dollar Standards page 92
4. CFA Institute Research Objectivity Standards page 101
STUDY SESSION 2
Reading Assignments
Ethical and Professional Standards, CFA Program Curriculum, Volume 1, Level 2
(CFA Institute, 2011)
5. The Glenarm Company page 110
6. Preston Partners page 112
7. Super Selection page 115
8. Trade Allocation: Fair Dealing and Disclosure page 118
9. Changing Investment Objectives page 120
10. Prudence in Perspective page 121
STUDY SESSION 3
Reading Assignments
Quantitative Methods for Valuation, CFA Program Curriculum, Volume 1, Level 2
(CFA Institute, 2011)
11. Correlation and Regression page 138
12. Multiple Regression and Issues in Regression Analysis page 172
13. Time-Series Analysis page 218
©2010 Kaplan, Inc. Page 7
Book 1 – Ethical & Professional Standards, Quantitative Methods, and Economics
Readings and Learning Outcome Statements
STUDY SESSION 4
Reading Assignments
Economics for Valuation, CFA Program Curriculum, Volume 1, Level 2
(CFA Institute, 2011)
14. Economic Growth page 260
15. Regulation and Antitrust Policy in a Globalized Economy page 278
16. Trading with the World page 285
17. The Exchange Rate and the Balance of Payments page 296
18. Currency Exchange Rates page 307
19. Foreign Exchange Parity Relations page 327
20. Measuring Economic Activity page 348
Learning Outcome Statements (LOS)
The CFA Institute Learning Outcome Statements are listed below. These are repeated in each
topic review; however, the order may have been changed in order to get a better fit with the
flow of the review.
STUDY SESSION 1
The topical coverage corresponds with the following CFA Institute assigned reading:
1. Code of Ethics and Standards of Professional Conduct
The candidate should be able to:
a. state the six components of the Code of Ethics and the seven Standards of
Professional Conduct. (page 14)
b. explain the ethical responsibilities required by the Code and Standards.
(page 15)
The topical coverage corresponds with the following CFA Institute assigned reading:
2. Guidance for Standards I–VII
The candidate should be able to:
a. demonstrate a thorough knowledge of the Code of Ethics and Standards of
Professional Conduct by applying the Code and Standards to specific situations.
(page 18)
b. recommend practices and procedures designed to prevent violations of the Code
of Ethics and Standards of Professional Conduct. (page 18)
The topical coverage corresponds with the following CFA Institute assigned reading:
3. CFA Institute Soft Dollar Standards
The candidate should be able to:
a. define soft-dollar arrangements and state the general principles of the Soft
Dollar Standards. (page 92)
b. critique company soft-dollar practices and policies. (page 93)
c. determine whether a product or service qualifies as “permissible research” that
can be purchased with client brokerage. (page 96)
Page 8 ©2010 Kaplan, Inc.
Book 1 – Ethical & Professional Standards, Quantitative Methods, and Economics
Readings and Learning Outcome Statements
The topical coverage corresponds with the following CFA Institute assigned reading:
4. CFA Institute Research Objectivity Standards
The candidate should be able to:
a. explain the objectives of the Research Objectivity Standards. (page 101)
b. critique company policies and practices related to research objectivity, and
distinguish between changes required and changes recommended for compliance
with the Research Objectivity Standards. (page 101)
STUDY SESSION 2
The topical coverage corresponds with the following CFA Institute assigned reading:
5. The Glenarm Company
6. Preston Partners
7. Super Selection
For each of these cases, the candidate should be able to:
a. critique the practices and policies presented. (pages 110, 112, 115)
b. explain the appropriate action to take in response to conduct that violates the
CFA Institute Code of Ethics and Standards of Professional Conduct.
(pages 110, 112, 115)
The topical coverage corresponds with the following CFA Institute assigned reading:
8. Trade Allocation: Fair Dealing and Disclosure
The candidate should be able to:
a. critique trade allocation practices, and determine whether compliance exists with
the CFA Institute Standards of Professional Conduct addressing fair dealing and
client loyalty. (page 118)
b. discuss appropriate actions to take in response to trade allocation practices that
do not adequately respect client interests. (page 119)
The topical coverage corresponds with the following CFA Institute assigned reading:
9. Changing Investment Objectives
The candidate should be able to:
a. critique the disclosure of investment objectives and basic policies and determine
whether they comply with the CFA Institute Standards of Professional Conduct.
(page 120)
b. discuss appropriate actions needed to ensure adequate disclosure of the
investment process. (page 120)
The topical coverage corresponds with the following CFA Institute assigned reading:
10. Prudence in Perspective
The candidate should be able to:
a. explain the basic principles of the new Prudent Investor Rule. (page 121)
b. explain the general fiduciary standards to which a trustee must adhere.
(page 122)
c. differentiate between the old Prudent Man Rule and the new Prudent Investor
Rule. (page 123)
d. explain the key factors that a trustee should consider when investing and
managing trust assets. (page 123)
©2010 Kaplan, Inc. Page 9
Book 1 – Ethical & Professional Standards, Quantitative Methods, and Economics
Readings and Learning Outcome Statements
STUDY SESSION 3
The topical coverage corresponds with the following CFA Institute assigned reading:
11. Correlation and Regression
The candidate should be able to:
a. calculate and interpret a sample covariance and a sample correlation coefficient,
and interpret a scatter plot. (page 138)
b. explain the limitations to correlation analysis, including outliers and spurious
correlation. (page 142)
c. formulate a test of the hypothesis that the population correlation coefficient
equals zero, and determine whether the hypothesis is rejected at a given level of
significance. (page 143)
d. distinguish between the dependent and independent variables in a linear
regression. (page 144)
e. explain the assumptions underlying linear regression, and interpret the
regression coefficients. (page 146)
f. calculate and interpret the standard error of estimate, the coefficient of
determination, and a confidence interval for a regression coefficient. (page 150)
g. formulate a null and alternative hypothesis about a population value of a
regression coefficient, select the appropriate test statistic, and determine whether
the null hypothesis is rejected at a given level of significance. (page 152)
h. calculate a predicted value for the dependent variable, given an estimated
regression model and a value for the independent variable, and calculate and
interpret a confidence interval for the predicted value of a dependent variable.
(page 153)
i. describe the use of analysis of variance (ANOVA) in regression analysis, interpret
ANOVA results, and calculate and interpret an F-statistic. (page 154)
j. discuss the limitations of regression analysis. (page 159)
The topical coverage corresponds with the following CFA Institute assigned reading:
12. Multiple Regression and Issues in Regression Analysis
The candidate should be able to:
a. formulate a multiple regression equation to describe the relation between a
dependent variable and several independent variables, determine the statistical
significance of each independent variable, and interpret the estimated
coefficients and their p-values. (page 173)
b. formulate a null and an alternative hypothesis about the population value of a
regression coefficient, calculate the value of the test statistic, determine whether
to reject the null hypothesis at a given level of significance by using a one-tailed
or two-tailed test, and interpret the results of the test. (page 175)
c. calculate and interpret 1) a confidence interval for the population value of a
regression coefficient and 2) a predicted value for the dependent variable, given
an estimated regression model and assumed values for the independent variables.
(page 179)
d. explain the assumptions of a multiple regression model. (page 181)
e. calculate and interpret the F-statistic, and discuss how it is used in regression
analysis. (page 181)
f. distinguish between and interpret the R2 and adjusted R2 in multiple regression.
(page 183)
Page 10 ©2010 Kaplan, Inc.
Book 1 – Ethical & Professional Standards, Quantitative Methods, and Economics
Readings and Learning Outcome Statements
g. infer how well a regression model explains the dependent variable by analyzing
the output of the regression equation and an ANOVA table. (page 185)
h. formulate a multiple regression equation by using dummy variables to represent
qualitative factors, and interpret the coefficients and regression results.
(page 189)
i. discuss the types of heteroskedasticity and the effects of heteroskedasticity and
serial correlation on statistical inference. (page 193)
j. describe multicollinearity, and discuss its causes and effects in regression
analysis. (page 199)
k. discuss the effects of model misspecification on the results of a regression
analysis, and explain how to avoid the common forms of misspecification.
(page 202)
l. discuss models with qualitative dependent variables. (page 206)
m. interpret the economic meaning of the results of multiple regression analysis and
critique a regression model and its results. (page 206)
The topical coverage corresponds with the following CFA Institute assigned reading:
13. Time-Series Analysis
The candidate should be able to:
a. calculate and evaluate the predicted trend value for a time series, modeled as
either a linear trend or a log-linear trend, given the estimated trend coefficients.
(page 218)
b. discuss the factors that determine whether a linear or a log-linear trend should
be used with a particular time series, and evaluate the limitations of trend
models. (page 224)
c. explain the requirement for a time series to be covariance stationary, and discuss
the significance of a series that is not stationary. (page 225)
d. discuss the structure of an autoregressive (AR) model of order p, and calculate
one- and two-period-ahead forecasts given the estimated coefficients. (page 226)
e. explain how autocorrelations of the residuals can be used to test whether the
autoregressive model fits the time series. (page 227)
f. explain mean reversion, and calculate a mean-reverting level. (page 228)
g. contrast in-sample and out-of-sample forecasts, and compare the forecasting
accuracy of different time-series models based on the root mean squared error
criterion. (page 229)
h. discuss the instability of coefficients of time-series models. (page 230)
i. describe the characteristics of random walk processes, and contrast them to
covariance stationary processes. (page 231)
j. discuss the implications of unit roots for time-series analysis, explain when unit
roots are likely to occur and how to test for them, and demonstrate how a time
series with a unit root can be transformed so it can be analyzed with an AR
model. (page 232)
k. discuss the steps of the unit root test for nonstationarity, and explain the relation
of the test to autoregressive time-series models. (page 232)
l. discuss how to test and correct for seasonality in a time-series model, and
calculate and interpret a forecasted value using an AR model with a seasonal lag.
(page 235)
m. explain autoregressive conditional heteroskedasticity (ARCH), and discuss how
ARCH models can be applied to predict the variance of a time series. (page 239)
n. explain how time-series variables should be analyzed for nonstationarity and/or
cointegration before use in a linear regression. (page 240)
©2010 Kaplan, Inc. Page 11
Book 1 – Ethical & Professional Standards, Quantitative Methods, and Economics
Readings and Learning Outcome Statements
o. select and justify the choice of a particular time-series model from a group of
models. (page 242)
STUDY SESSION 4
The topical coverage corresponds with the following CFA Institute assigned reading:
14. Economic Growth
The candidate should be able to:
a. define the sources of economic growth, and discuss the preconditions for
economic growth. (page 261)
b. discuss how the one-third rule can be used to explain the contributions of labor
and technological change to growth in labor productivity. (page 262)
c. discuss how faster economic growth can be achieved by increasing the growth of
physical capital, technological advances, and investment in human capital.
(page 265)
d. compare and contrast classical growth theory, neoclassical growth theory, and
new growth theory. (page 265)
The topical coverage corresponds with the following CFA Institute assigned reading:
15. Regulation and Antitrust Policy in a Globalized Economy
The candidate should be able to:
a. explain the rationale for government regulation in the form of 1) economic
regulation of natural monopolies and 2) social regulation of nonmonopolistic
industries. (page 278)
b. discuss the potential benefits and possible negative side effects of social
regulation. (page 280)
c. differentiate between the capture hypothesis and the share-the-gains, share-thepains
theory of regulator behavior. (page 280)
The topical coverage corresponds with the following CFA Institute assigned reading:
16. Trading with the World
The candidate should be able to:
a. explain comparative advantage and how countries can gain from international
trade. (page 285)
b. compare and contrast tariffs, nontariff barriers, quotas, and voluntary export
restraints. (page 288)
c. critique the arguments for trade restrictions. (page 290)
The topical coverage corresponds with the following CFA Institute assigned reading:
17. The Exchange Rate and the Balance of Payments
The candidate should be able to:
a. define an exchange rate, and differentiate between the nominal exchange rate
and the real exchange rate. (page 296)
b. explain the factors that influence supply and demand in the foreign exchange
market. (page 297)
c. discuss how the supply and demand for a currency changes the exchange rate.
(page 299)
d. differentiate between interest rate parity and purchasing power parity.
(page 300)
e. describe the balance of payments accounts. (page 301)
Page 12 ©2010 Kaplan, Inc.
Book 1 – Ethical & Professional Standards, Quantitative Methods, and Economics
Readings and Learning Outcome Statements
f. describe the following exchange rate policies: flexible exchange rates, fixed
exchange rates, and crawling pegs. (page 302)
The topical coverage corresponds with the following CFA Institute assigned reading:
18. Currency Exchange Rates
The candidate should be able to:
a. define direct and indirect methods of foreign exchange quotations, and convert
direct (indirect) foreign exchange quotations into indirect (direct) foreign
exchange quotations. (page 307)
b. calculate and interpret the spread on a foreign currency quotation, and explain
how spreads on foreign currency quotations can differ as a result of market
conditions, bank/dealer positions, and trading volume. (page 309)
c. calculate and interpret currency cross rates, given two spot exchange quotations
involving three currencies. (page 310)
d. calculate the profit on a triangular arbitrage opportunity, given the bid-ask
quotations for the currencies of three countries involved in the arbitrage.
(page 312)
e. distinguish between the spot and forward markets for foreign exchange.
(page 313)
f. calculate and interpret the spread on a forward foreign currency quotation, and
explain how spreads on forward foreign currency quotations can differ as a result
of market conditions, bank/dealer positions, trading volume, and maturity/
length of contract. (page 314)
g. calculate and interpret a forward discount or premium and express it as an
annualized rate. (page 315)
h. explain interest rate parity, and illustrate covered interest arbitrage. (page 316)
i. distinguish between spot and forward transactions, calculate the annualized
forward premium/discount for a given currency, and infer whether the currency
is “strong” or “weak.” (page 318)
The topical coverage corresponds with the following CFA Institute assigned reading:
19. Foreign Exchange Parity Relations
The candidate should be able to:
a. explain how exchange rates are determined in a flexible (or floating) exchange
rate system. (page 327)
b. explain the role of each component of the balance of payments accounts.
(page 327)
c. explain how current account deficits or surpluses and financial account deficits
or surpluses affect an economy. (page 328)
d. describe the factors that cause a nation’s currency to appreciate or depreciate.
(page 328)
e. explain how monetary and fiscal policies affect the exchange rate and balance of
payments components. (page 329)
f. describe a fixed exchange rate and a pegged exchange rate system. (page 330)
g. discuss absolute purchasing power parity and relative purchasing power parity.
(page 331)
h. calculate the end-of-period exchange rate implied by purchasing power parity,
given the beginning-of-period exchange rate and the inflation rates. (page 331)
i. discuss the international Fisher relation. (page 333)
©2010 Kaplan, Inc. Page 13
Book 1 – Ethical & Professional Standards, Quantitative Methods, and Economics
Readings and Learning Outcome Statements
j. calculate the real interest rate, given nominal interest rates and expected inflation
rates, using the international Fisher relation and its linear approximation.
(page 333)
k. discuss the theory of uncovered interest rate parity, and explain the theory’s
relation to other exchange rate parity theories. (page 335)
l. calculate the expected change in the exchange rate, given interest rates and the
assumption that uncovered interest rate parity holds. (page 335)
m. discuss the foreign exchange expectation relation between the forward exchange
rate and the expected exchange rate. (page 337)
The topical coverage corresponds with the following CFA Institute assigned reading:
20. Measuring Economic Activity
The candidate should be able to:
a. distinguish between the measures of economic activity (i.e., gross domestic
product, gross national income, and net national income), including their
components. (page 348)
b. differentiate between GDP at market prices and GDP at factor cost. (page 349)
c. differentiate between current and constant prices, and describe the GDP
deflator. (page 350)
Page 14 ©2010 Kaplan, Inc.
The following is a review of the Ethical and Professional Standards principles designed to address the learning
outcome statements set forth by CFA Institute®. This topic is also covered in:
CFA Institute Code of Ethics and
Standards of Professional Conduct
Guidance for Standards I–VII
Study Session 1
Exam Focus
In addition to reading this review of the ethics material, we strongly recommend that
all candidates for the CFA® examination read the Standards of Practice Handbook 10th
Edition (2010) multiple times. As a Level 2 CFA candidate, it is your responsibility to
comply with the Code and Standards. The complete Code and Standards are reprinted in
Volume 1 of the CFA Program Curriculum.
LOS 1.a: State the six components of the Code of Ethics and the seven
Standards of Professional Conduct.
Code of Ethics
Members of CFA Institute [including Chartered Financial Analyst® (CFA®)
charterholders] and candidates for the CFA designation (“Members and Candidates”)
must:1
Act with integrity, competence, diligence, respect, and in an 􀁳 ethical manner with
the public, clients, prospective clients, employers, employees, colleagues in the
investment profession, and other participants in the global capital markets.
􀁳 Place the integrity of the investment profession and the interests of clients above
their own personal interests.
􀁳 Use reasonable care and exercise independent professional judgment when
conducting investment analysis, making investment recommendations, taking
investment actions, and engaging in other professional activities.
􀁳 Practice and encourage others to practice in a professional and ethical manner that
will reflect credit on themselves and the profession.
􀁳 Promote the integrity of, and uphold the rules governing, capital markets.
􀁳 Maintain and improve their professional competence and strive to maintain and
improve the competence of other investment professionals.
The Standards of Professional Conduct
I: Professionalism
II: Integrity of Capital Markets
III: Duties to Clients
1. Copyright 2010, CFA Institute. Reproduced and republished from “The Code of Ethics,”
from Standards of Practice Handbook, 10th Ed., 2010, with permission from CFA Institute.
All rights reserved.
©2010 Kaplan, Inc. Page 15
Study Session 1
Cross-Reference to CFA Institute Assigned Readings #1 & 2 – Standards of Practice Handbook
IV: Duties to Employers
V: Investment Analysis, Recommendations, and Actions
VI: Conflicts of Interest
VII: Responsibilities as a CFA Institute Member or CFA Candidate
LOS 1.b: Explain the ethical responsibilities required by the Code and
Standards.
Standards of Professional Conduct2
I. PROFESSIO NALISM
A. Knowledge of the Law. Members and Candidates must understand and
comply with all applicable laws, rules, and regulations (including the CFA
Institute Code of Ethics and Standards of Professional Conduct) of any
government, regulatory organization, licensing agency, or professional
association governing their professional activities. In the event of conflict,
Members and Candidates must comply with the more strict law, rule, or
regulation. Members and Candidates must not knowingly participate or assist
in any violation of laws, rules, or regulations and must disassociate themselves
from any such violation.
B. Independence and Objectivity. Members and Candidates must use reasonable
care and judgment to achieve and maintain independence and objectivity in
their professional activities. Members and Candidates must not offer, solicit, or
accept any gift, benefit, compensation, or consideration that reasonably could
be expected to compromise their own or another’s independence and
objectivity.
C. Misrepresentation. Members and Candidates must not knowingly make any
misrepresentations relating to investment analysis, recommendations, actions,
or other professional activities.
D. Misconduct. Members and Candidates must not engage in any professional
conduct involving dishonesty, fraud, or deceit or commit any act that reflects
adversely on their professional reputation, integrity, or competence.
II. IN TEGRI TY OF CAPI TAL MARKETS
A. Material Nonpublic Information. Members and Candidates who possess
material nonpublic information that could affect the value of an investment
must not act or cause others to act on the information.
B. Market Manipulation. Members and Candidates must not engage in practices
that distort prices or artificially inflate trading volume with the intent to
mislead market participants.
2. Ibid.
Page 16 ©2010 Kaplan, Inc.
Study Session 1
Cross-Reference to CFA Institute Assigned Readings #1 & 2 – Standards of Practice Handbook
Study Session 1
III. DUT IES TO CLIEN TS
A. Loyalty, Prudence, and Care. Members and Candidates have a duty of loyalty
to their clients and must act with reasonable care and exercise prudent
judgment. Members and Candidates must act for the benefit of their clients
and place their clients’ interests before their employer’s or their own interests.
B. Fair Dealing. Members and Candidates must deal fairly and objectively with
all clients when providing investment analysis, making investment
recommendations, taking investment action, or engaging in other professional
activities.
C. Suitability.
1. When Members and Candidates are in an advisory relationship with a
client, they must:
a. Make a reasonable inquiry into a client’s or prospective clients’
investment experience, risk and return objectives, and financial
constraints prior to making any investment recommendation or taking
investment action and must reassess and update this information
regularly.
b. Determine that an investment is suitable to the client’s financial
situation and consistent with the client’s written objectives, mandates,
and constraints before making an investment recommendation or
taking investment action.
c. Judge the suitability of investments in the context of the client’s total
portfolio.
2. When Members and Candidates are responsible for managing a portfolio to
a specific mandate, strategy, or style, they must make only investment
recommendations or take investment actions that are consistent with the
stated objectives and constraints of the portfolio.
D. Performance Presentation. When communicating investment performance
information, Members or Candidates must make reasonable efforts to ensure
that it is fair, accurate, and complete.
E. Preservation of Confidentiality. Members and Candidates must keep
information about current, former, and prospective clients confidential unless:
1. The information concerns illegal activities on the part of the client or
prospective client,
2. Disclosure is required by law, or
3. The client or prospective client permits disclosure of the information.
©2010 Kaplan, Inc. Page 17
Study Session 1
Cross-Reference to CFA Institute Assigned Readings #1 & 2 – Standards of Practice Handbook
IV. DUT IES TO EMPLOYERS
A. Loyalty. In matters related to their employment, Members and Candidates
must act for the benefit of their employer and not deprive their employer of the
advantage of their skills and abilities, divulge confidential information, or
otherwise cause harm to their employer.
B. Additional Compensation Arrangements. Members and Candidates must not
accept gifts, benefits, compensation, or consideration that competes with, or
might reasonably be expected to create a conflict of interest with, their
employer’s interest unless they obtain written consent from all parties involved.
C. Responsibilities of Supervisors. Members and Candidates must make
reasonable efforts to detect and prevent violations of applicable laws, rules,
regulations, and the Code and Standards by anyone subject to their supervision
or authority.
V. INVESTMEN T ANALYSIS, RECOMMEN DAT IO NS, AN D AC T IO NS
A. Diligence and Reasonable Basis. Members and Candidates must:
1. Exercise diligence, independence, and thoroughness in analyzing
investments, making investment recommendations, and taking investment
actions.
2. Have a reasonable and adequate basis, supported by appropriate research
and investigation, for any investment analysis, recommendation, or action.
B. Communication with Clients and Prospective Clients. Members and
Candidates must:
1. Disclose to clients and prospective clients the basic format and general
principles of the investment processes used to analyze investments, select
securities, and construct portfolios and must promptly disclose any changes
that might materially affect those processes.
2. Use reasonable judgment in identifying which factors are important to their
investment analyses, recommendations, or actions and include those factors
in communications with clients and prospective clients.
3. Distinguish between fact and opinion in the presentation of investment
analysis and recommendations.
C. Record Retention. Members and Candidates must develop and maintain
appropriate records to support their investment analysis, recommendations,
actions, and other investment-related communications with clients and
prospective clients.
Page 18 ©2010 Kaplan, Inc.
Study Session 1
Cross-Reference to CFA Institute Assigned Readings #1 & 2 – Standards of Practice Handbook
Study Session 1
VI. CO NFLIC TS OF IN TEREST
A. Disclosure of Conflicts. Members and Candidates must make full and fair
disclosure of all matters that could reasonably be expected to impair their
independence and objectivity or interfere with respective duties to their clients,
prospective clients, and employer. Members and Candidates must ensure that
such disclosures are prominent, are delivered in plain language, and
communicate the relevant information effectively.
B. Priority of Transactions. Investment transactions for clients and employers
must have priority over investment transactions in which a Member or
Candidate is the beneficial owner.
C. Referral Fees. Members and Candidates must disclose to their employer,
clients, and prospective clients, as appropriate, any compensation,
consideration, or benefit received by, or paid to, others for the recommendation
of products or services.
VII. RESPO NSIBILI T IES AS A CFA INST I TUTE MEMBER OR CFA
CAN DIDATE
A. Conduct as Members and Candidates in the CFA Program. Members and
Candidates must not engage in any conduct that compromises the reputation
or integrity of CFA Institute or the CFA designation or the integrity, validity,
or security of the CFA examinations.
B. Reference to CFA Institute, the CFA Designation, and the CFA Program.
When referring to CFA Institute, CFA Institute membership, the CFA
designation, or candidacy in the CFA Program, Members and Candidates must
not misrepresent or exaggerate the meaning or implications of membership in
CFA Institute, holding the CFA designation, or candidacy in the CFA
Program.
LOS 2.a: Demonstrate a thorough knowledge of the Code of Ethics and
Standards of Professional Conduct by applying the Code and Standards to
specific situations.
LOS 2.b: Recommend practices and procedures designed to prevent
violations of the Code of Ethics and Standards of Professional Conduct.
I Professionalism
I(A) Knowledge of the Law. Members and Candidates must understand and
comply with all applicable laws, rules, and regulations (including the CFA Institute
Code of Ethics and Standards of Professional Conduct) of any government, regulatory
organization, licensing agency, or professional association governing their professional
activities. In the event of conflict, Members and Candidates must comply with the
more strict law, rule, or regulation. Members and Candidates must not knowingly
participate or assist in and must dissociate from any violation of such laws, rules, or
regulations.
©2010 Kaplan, Inc. Page 19
Study Session 1
Cross-Reference to CFA Institute Assigned Readings #1 & 2 – Standards of Practice Handbook
Professor’s Note: While we use the term “members” in the following, note that all
of the Standards apply to candidates as well.
Guidance—Code and Standards vs. Local Law
Members must know the laws and regulations relating to their professional activities in
all countries in which they conduct business. Members must comply with applicable
laws and regulations relating to their professional activity. Do not violate Code or
Standards even if the activity is otherwise legal. Always adhere to the most strict rules
and requirements (law or CFA Institute Standards) that apply.
Guidance—Participation or Association with Violations by Others
Members should dissociate, or separate themselves, from any ongoing client or employee
activity that is illegal or unethical, even if it involves leaving an employer (an extreme
case). While a member may confront the involved individual first, he must approach
his supervisor or compliance department. Inaction with continued association may be
construed as knowing participation.
Recommended Procedures for Compliance—Members
Members should have procedures to keep up with changes in applicable 􀁳 laws, rules,
and regulations.
􀁳 Compliance procedures should be reviewed on an ongoing basis to assure that they
address current law, CFAI Standards, and regulations.
􀁳 Members should maintain current reference materials for employees to access in
order to keep up to date on laws, rules, and regulations.
􀁳 Members should seek advice of counsel or their compliance department when in
doubt.
􀁳 Members should document any violations when they disassociate themselves from
prohibited activity and encourage their employers to bring an end to such activity.
􀁳 There is no requirement under the Standards to report violations to governmental
authorities, but this may be advisable in some circumstances and required by law in
others.
􀁳 Members are strongly encouraged to report other members’ violations of the Code
and Standards.
Recommended Procedures for Compliance—Firms
Members should encourage their firms to:
􀁳 Develop and/or adopt a code of ethics.
􀁳 Make available to employees information that highlights applicable laws and
regulations.
􀁳 Establish written procedures for reporting suspected violation of laws, regulations, or
company policies.
Members who supervise the creation and maintenance of investment services and
products should be aware of and comply with the regulations and laws regarding such
services and products both in their country of origin and the countries where they will
be sold.
Page 20 ©2010 Kaplan, Inc.
Study Session 1
Cross-Reference to CFA Institute Assigned Readings #1 & 2 – Standards of Practice Handbook
Study Session 1
Application of Standard I(A) Knowledge of the Law3
Example 1:
Michael Allen works for a brokerage firm and is responsible for an underwriting of
securities. A company official gives Allen information indicating that the financial
statements Allen filed with the regulator overstate the issuer’s earnings. Allen seeks the
advice of the brokerage firm’s general counsel, who states that it would be difficult for
the regulator to prove that Allen has been involved in any wrongdoing.
Comment:
Although it is recommended that members and candidates seek the advice of legal
counsel, the reliance on such advice does not absolve a member or candidate from the
requirement to comply with the law or regulation. Allen should report this situation to
his supervisor, seek an independent legal opinion, and determine whether the regulator
should be notified of the error.
Example 2:
Kamisha Washington’s firm advertises its past performance record by showing the 10-
year return of a composite of its client accounts. However, Washington discovers that the
composite omits the performance of accounts that have left the firm during the 10-year
period and that this omission has led to an inflated performance figure. Washington
is asked to use promotional material that includes the erroneous performance number
when soliciting business for the firm.
Comment:
Misrepresenting performance is a violation of the Code and Standards. Although she did
not calculate the performance herself, Washington would be assisting in violating this
standard if she were to use the inflated performance number when soliciting clients. She
must dissociate herself from the activity. She can bring the misleading number to the
attention of the person responsible for calculating performance, her supervisor, or the
compliance department at her firm. If her firm is unwilling to recalculate performance,
she must refrain from using the misleading promotional material and should notify
the firm of her reasons. If the firm insists that she use the material, she should consider
whether her obligation to dissociate from the activity would require her to seek other
employment.
Example 3:
An employee of an investment bank is working on an underwriting and finds out the
issuer has altered their financial statements to hide operating losses in one division.
These misstated data are included in a preliminary prospectus that has already been
released.
Comment:
The employee should report the problem to his supervisors. If the firm doesn’t get the
misstatement fixed, the employee should dissociate from the underwriting and, further,
seek legal advice about whether he should undertake additional reporting or other
actions.
3. Ibid.
©2010 Kaplan, Inc. Page 21
Study Session 1
Cross-Reference to CFA Institute Assigned Readings #1 & 2 – Standards of Practice Handbook
Example 4:
Laura Jameson, a U.S. citizen, works for an investment advisor based in the U.S. and
works in a country where investment managers are prohibited from participating in
IPOs for their own accounts.
Comment:
Jameson must comply with the strictest requirements among U.S. law (where her firm
is based), the CFA Institute Code and Standards, and the laws of the country where she
is doing business. In this case, that means she must not participate in any IPOs for her
personal account.
Example 5:
A junior portfolio manager suspects that a broker responsible for new business from
a foreign country is being allocated a portion of the firm’s payments for third-party
research and suspects that no research is being provided. He believes that the research
payments may be inappropriate and unethical.
Comment:
He should follow his firm’s procedures for reporting possible unethical behavior and try
to get better disclosure of the nature of these payments and any research that is being
provided.
I(B) Independence and Objectivity. Members and Candidates must use reasonable
care and judgment to achieve and maintain independence and objectivity in their
professional activities. Members and Candidates must not offer, solicit, or accept any
gift, benefit, compensation, or consideration that reasonably could be expected to
compromise their own or another’s independence and objectivity.
Guidance
Do not let the investment process be influenced by any external sources. Modest gifts
are permitted. Allocation of shares in oversubscribed IPOs to personal accounts is
NOT permitted. Distinguish between gifts from clients and gifts from entities seeking
influence to the detriment of the client. Gifts must be disclosed to the member’s
employer in any case, either prior to acceptance if possible, or subsequently.
Guidance—Investment Banking Relationships
Do not be pressured by sell-side firms to issue favorable research on current or
prospective investment-banking clients. It is appropriate to have analysts work with
investment bankers in “road shows” only when the conflicts are adequately and
effectively managed and disclosed. Be sure there are effective “firewalls” between
research/investment management and investment banking activities.
Page 22 ©2010 Kaplan, Inc.
Study Session 1
Cross-Reference to CFA Institute Assigned Readings #1 & 2 – Standards of Practice Handbook
Study Session 1
Guidance—Public Companies
Analysts should not be pressured to issue favorable research by the companies they
follow. Do not confine research to discussions with company management, but rather
use a variety of sources, including suppliers, customers, and competitors.
Guidance—Buy-Side Clients
Buy-side clients may try to pressure sell-side analysts. Portfolio managers may have large
positions in a particular security, and a rating downgrade may have an effect on the
portfolio performance. As a portfolio manager, there is a responsibility to respect and
foster intellectual honesty of sell-side research.
Guidance—Fund Manager Relationships
Members responsible for selecting outside managers should not accept gifts,
entertainment, or travel that might be perceived as impairing their objectivity.
Guidance—Credit Rating Agencies
Members employed by credit rating firms should make sure that procedures prevent
undue influence by the firm issuing the securities. Members who use credit ratings
should be aware of this potential conflict of interest and consider whether independent
analysis is warranted.
Guidance—Issuer-Paid Research
Remember that this type of research is fraught with potential conflicts. Analysts’
compensation for preparing such research should be limited, and the preference is for a
flat fee, without regard to conclusions or the report’s recommendations.
Guidance—Travel
Best practice is for analysts to pay for their own commercial travel when attending
information events or tours sponsored by the firm being analyzed.
Recommended Procedures for Compliance
Protect the integrity of opinions—make sure 􀁳 they are unbiased.
􀁳 Create a restricted list and distribute only factual information about companies on
the list.
􀁳 Restrict special cost arrangements—pay for one’s own commercial transportation
and hotel; limit use of corporate aircraft to cases in which commercial transportation
is not available.
􀁳 Limit gifts—token items only. Customary, business-related entertainment is okay
as long as its purpose is not to influence a member’s professional independence or
objectivity. Firms should impose clear value limits on gifts.
􀁳 Restrict employee investments in equity IPOs and private placements. Require preapproval
of IPO purchases.
􀁳 Review procedures—have effective supervisory and review procedures.
©2010 Kaplan, Inc. Page 23
Study Session 1
Cross-Reference to CFA Institute Assigned Readings #1 & 2 – Standards of Practice Handbook
Firms should have formal written policies on independence 􀁳 and objectivity of
research.
􀁳 Firms should appoint a compliance officer and provide clear procedures for
employee reporting of unethical behavior and violations of applicable regulations.
Application of Standard I(B) Independence and Objectivity
Example 1:
Steven Taylor, a mining analyst with Bronson Brokers, is invited by Precision Metals to
join a group of his peers in a tour of mining facilities in several western U.S. states. The
company arranges for chartered group flights from site to site and for accommodations
in Spartan Motels, the only chain with accommodations near the mines, for three nights.
Taylor allows Precision Metals to pick up his tab, as do the other analysts, with one
exception—John Adams, an employee of a large trust company who insists on following
his company’s policy and paying for his hotel room himself.
Comment:
The policy of the company where Adams works complies closely with Standard I(B) by
avoiding even the appearance of a conflict of interest, but Taylor and the other analysts
were not necessarily violating Standard I(B). In general, when allowing companies to pay
for travel and/or accommodations under these circumstances, members and candidates
must use their judgment, keeping in mind that such arrangements must not impinge
on a member or candidate’s independence and objectivity. In this example, the trip was
strictly for business and Taylor was not accepting irrelevant or lavish hospitality. The
itinerary required chartered flights, for which analysts were not expected to pay. The
accommodations were modest. These arrangements are not unusual and did not violate
Standard I(B) so long as Taylor’s independence and objectivity were not compromised.
In the final analysis, members and candidates should consider both whether they can
remain objective and whether their integrity might be perceived by their clients to have
been compromised.
Example 2:
Walter Fritz is an equity analyst with Hilton Brokerage who covers the mining industry.
He has concluded that the stock of Metals & Mining is overpriced at its current level,
but he is concerned that a negative research report will hurt the good relationship
between Metals & Mining and the investment-banking division of his firm. In fact, a
senior manager of Hilton Brokerage has just sent him a copy of a proposal his firm has
made to Metals & Mining to underwrite a debt offering. Fritz needs to produce a report
right away and is concerned about issuing a less-than-favorable rating.
Comment:
Fritz’s analysis of Metals & Mining must be objective and based solely on consideration
of company fundamentals. Any pressure from other divisions of his firm is inappropriate.
This conflict could have been eliminated if, in anticipation of the offering, Hilton
Brokerage had placed Metals & Mining on a restricted list for its sales force.
Example 3:
Tom Wayne is the investment manager of the Franklin City Employees Pension Plan.
He recently completed a successful search for firms to manage the foreign equity
Page 24 ©2010 Kaplan, Inc.
Study Session 1
Cross-Reference to CFA Institute Assigned Readings #1 & 2 – Standards of Practice Handbook
Study Session 1
allocation of the plan’s diversified portfolio. He followed the plan’s standard procedure
of seeking presentations from a number of qualified firms and recommended that his
board select Penguin Advisors because of its experience, well-defined investment strategy,
and performance record, which was compiled and verified in accordance with the
CFA Institute Global Investment Performance Standards. Following the plan selection
of Penguin, a reporter from the Franklin City Record called to ask if there was any
connection between the action and the fact that Penguin was one of the sponsors of an
“investment fact-finding trip to Asia” that Wayne made earlier in the year. The trip was
one of several conducted by the Pension Investment Academy, which had arranged the
itinerary of meetings with economic, government, and corporate officials in major cities
in several Asian countries. The Pension Investment Academy obtains support for the cost
of these trips from a number of investment managers, including Penguin Advisors; the
Academy then pays the travel expenses of the various pension plan managers on the trip
and provides all meals and accommodations. The president of Penguin Advisors was one
of the travelers on the trip.
Comment:
Although Wayne can probably put to good use the knowledge he gained from the trip
in selecting portfolio managers and in other areas of managing the pension plan, his
recommendation of Penguin Advisors may be tainted by the possible conflict incurred
when he participated in a trip paid for partly by Penguin Advisors and when he was in
the daily company of the president of Penguin Advisors. To avoid violating Standard
I(B), Wayne’s basic expenses for travel and accommodations should have been paid
by his employer or the pension plan; contact with the president of Penguin Advisors
should have been limited to informational or educational events only; and the trip, the
organizer, and the sponsor should have been made a matter of public record. Even if his
actions were not in violation of Standard I(B), Wayne should have been sensitive to the
public perception of the trip when reported in the newspaper and the extent to which
the subjective elements of his decision might have been affected by the familiarity that
the daily contact of such a trip would encourage. This advantage would probably not be
shared by competing firms.
Example 4:
An analyst in the corporate finance department promises a client that her firm will
provide full research coverage of the issuing company after the offering.
Comment:
This is not a violation, but she cannot promise favorable research coverage. Research
must be objective and independent.
Example 5:
An employee’s boss tells him to assume coverage of a stock and maintain a buy rating.
Comment:
Research opinions and recommendations must be objective and independently arrived
at. Following the boss’s instructions would be a violation if the analyst determined a buy
rating is inappropriate.
©2010 Kaplan, Inc. Page 25
Study Session 1
Cross-Reference to CFA Institute Assigned Readings #1 & 2 – Standards of Practice Handbook
Example 6:
A money manager receives a gift of significant value from a client as a reward for good
performance over the prior period and informs her employer of the gift.
Comment:
No violation here since the gift is from a client and is not based on performance going
forward, but the gift must be disclosed to her employer. If the gift were contingent on
future performance, the money manager would have to obtain permission from her
employer. The reason for both the disclosure and permission requirements is that the
employer must ensure that the money manager does not give advantage to the client
giving or offering additional compensation, to the detriment of other clients.
Example 7:
An analyst enters into a contract to write a research report on a company, paid for
by that company, for a flat fee plus a bonus based on attracting new investors to the
security.
Comment:
This is a violation because the compensation structure makes total compensation depend
on the conclusions of the report (a favorable report will attract investors and increase
compensation). Accepting the job for a flat fee that does not depend on the report’s
conclusions or its impact on share price is permitted, with proper disclosure of the fact
that the report is funded by the subject company.
Example 8:
A trust manager at a bank selects mutual funds for client accounts based on the profits
from “service fees” paid to the bank by the mutual fund sponsor.
Comment:
This is a violation because the trust manager has allowed the fees to affect his objectivity.
Example 9:
An analyst performing sensitivity analysis for a security does not use only scenarios
consistent with recent trends and historical norms.
Comment:
This is a good thing and is not a violation.
Page 26 ©2010 Kaplan, Inc.
Study Session 1
Cross-Reference to CFA Institute Assigned Readings #1 & 2 – Standards of Practice Handbook
Study Session 1
I(C) Misrepresentation. Members and Candidates must not knowingly make any
misrepresentations relating to investment analysis, recommendations, actions, or
other professional activities.
Guidance
Trust is a foundation in the investment profession. Do not make any misrepresentations
or give false impressions. This includes oral and electronic communications.
Misrepresentations include guaranteeing investment performance and plagiarism.
Plagiarism encompasses using someone else’s work (e.g., reports, forecasts, models, ideas,
charts, graphs, and spreadsheet models) without giving them credit. Knowingly omitting
information that could affect an investment decision is considered misrepresentation.
Models and analysis developed by others at a member’s firm are the property of the firm
and can be used without attribution. A report written by another analyst employed by
the firm cannot be released as another analyst’s work.
Recommended Procedures for Compliance
A good way to avoid misrepresentation is for firms to provide employees who deal with
clients or prospects a written list of the firm’s available services and a description of the
firm’s qualifications. Employee qualifications should be accurately presented as well.
To avoid plagiarism, maintain records of all materials used to generate reports or other
firm products and properly cite sources (quotes and summaries) in work products.
Information from recognized financial and statistical reporting services need not be
cited.
Members should encourage their firms to establish procedures for verifying marketing
claims of third parties whose information the firm provides to clients.
Application of Standard I(C) Misrepresentation
Example 1:
Allison Rogers is a partner in the firm of Rogers and Black, a small firm offering
investment advisory services. She assures a prospective client who has just inherited
$1 million that “we can perform all the financial and investment services you need.”
Rogers and Black is well equipped to provide investment advice but, in fact, cannot
provide asset allocation assistance or a full array of financial and investment services.
Comment:
Rogers has violated Standard I(C) by orally misrepresenting the services her firm can
perform for the prospective client. She must limit herself to describing the range of
investment advisory services Rogers and Black can provide and offer to help the client
obtain elsewhere the financial and investment services that her firm cannot provide.
Example 2:
Anthony McGuire is an issuer-paid analyst hired by publicly traded companies to
electronically promote their stocks. McGuire creates a Web site that promotes his
research efforts as a seemingly independent analyst. McGuire posts a profile and a strong
buy recommendation for each company on the Web site, indicating that the stock is
©2010 Kaplan, Inc. Page 27
Study Session 1
Cross-Reference to CFA Institute Assigned Readings #1 & 2 – Standards of Practice Handbook
expected to increase in value. He does not disclose the contractual relationships with
the companies he covers on his Web site, in the research reports he issues, or in the
statements he makes about the companies on Internet chat rooms.
Comment:
McGuire has violated Standard I(C) because the Internet site and e-mails are misleading
to potential investors. Even if the recommendations are valid and supported with
thorough research, his omissions regarding the true relationship between himself and the
companies he covers constitute a misrepresentation. McGuire has also violated Standard
VI(C) by not disclosing the existence of an arrangement with the companies through
which he receives compensation in exchange for his services.
Example 3:
Claude Browning, a quantitative analyst for Double Alpha, Inc., returns in great
excitement from a seminar. In that seminar, Jack Jorrely, a well-publicized quantitative
analyst at a national brokerage firm, discussed one of his new models in great detail,
and Browning is intrigued by the new concepts. He proceeds to test this model, making
some minor mechanical changes but retaining the concept, until he produces some
very positive results. Browning quickly announces to his supervisors at Double Alpha
that he has discovered a new model and that clients and prospective clients alike should
be informed of this positive finding as ongoing proof of Double Alpha’s continuing
innovation and ability to add value.
Comment:
Although Browning tested Jorrely’s model on his own and even slightly modified it, he
must still acknowledge the original source of the idea. Browning can certainly take credit
for the final, practical results; he can also support his conclusions with his own test. The
credit for the innovative thinking, however, must be awarded to Jorrely.
Example 4:
Paul Ostrowski runs a 2-person investment management firm. Ostrowski’s firm
subscribes to a service from a large investment research firm that provides research
reports that can be repackaged by smaller firms for those firms’ clients. Ostrowski’s firm
distributes these reports to clients as its own work.
Comment:
Ostrowski can rely on third-party research that has a reasonable and adequate basis,
but he cannot imply that he is the author of the report. Otherwise, Ostrowski would
misrepresent the extent of his work in a way that would mislead the firm’s clients or
prospective clients.
Example 5:
A member makes an error in preparing marketing materials and misstates the amount of
assets his firm has under management.
Comment:
The member must attempt to stop distribution of the erroneous material as soon as
the error is known. Simply making the error unintentionally is not a violation, but
Page 28 ©2010 Kaplan, Inc.

CMA USA Part 2 MCQs 2015 Gleim

http://www.freeaccountingbooks.com/cma-usa-part-2-mcqs-2015-gleim/


Free Accounting books provides unlimited PDF books, notes & guides of #Financial, #cost and #management #accounting, #ACCA, #CFA, #CIMA, #CPA, #CMA, #FRM and many more .......
#freeaccountingbooks.com
Study Unit 1: Ethics for Management Accountants | Subunit 1: Ethics for Management Accountants
Question: 1
At Key Enterprises, the controller is responsible for directing the budgeting process. In this role, the controller has significant influence with executive management as individual department budgets are modified and approved. For the current year, the controller was instrumental in the approval of a particular line manager’s budget without modification, even though significant reductions were made to the budgets submitted by other line managers. As a token of appreciation, the line manager in question has given the controller a gift certificate for a popular local restaurant. In considering whether or not to accept the certificate, the controller should refer to which section of IMA’s Statement of Ethical Professional Practice?
A.
Competence.
B.
Confidentiality.
C.
Integrity.
Answer (C) is correct. The integrity standard requires an IMA member to “refrain from engaging in any conduct that would prejudice carrying out duties ethically.”
D.
Credibility.
Study Unit 1: Ethics for Management Accountants | Subunit 1: Ethics for Management Accountants
Question: 2
In accordance with IMA’s Statement of Ethical Professional Practice, a member who fails to perform professional duties in accordance with relevant standards is acting contrary to which one of the following standards?
A.
Competence.
Answer (A) is correct. One of the responsibilities of an IMA member under the competence standard is to “maintain an appropriate level of professional expertise by continually developing knowledge and skills.” (S)he must also “perform professional duties in accordance with relevant laws, regulations, and technical standards.” The third requirement under this standard is to “provide decision support information and recommendations that are accurate, clear, concise, and timely.”
B.
Confidentiality.
C.
Integrity.
D.
Credibility.
Study Unit 1: Ethics for Management Accountants | Subunit 1: Ethics for Management Accountants
Question: 3
According to IMA’s Statement of Ethical Professional Practice, a member has a responsibility to recognize professional limitations. Under which standard of ethical conduct would this responsibility be included?
A.
Competence.
Answer (A) is correct. The competence standard pertains to an IMA member’s responsibility to “recognize and communicate professional limitations or other constraints that would preclude responsible judgment or successful performance of an activity.”
B.
Confidentiality.
C.
Integrity.
D.
Credibility.
Study Unit 1: Ethics for Management Accountants | Subunit 1: Ethics for Management Accountants
Question: 4
If an IMA member has a problem in identifying unethical behavior or resolving an ethical conflict, the first action (s)he should normally take is to
A.
Consult the board of directors.
B.
Discuss the problem with his or her immediate superior.
Answer (B) is correct. IMA’s Statement of Ethical Professional Practice states that the member should first discuss an ethical problem with his or her immediate superior. If the superior is involved, the problem should be taken initially to the next higher managerial level.
C.
Notify the appropriate law enforcement agency.
D.
Resign from the company.
Study Unit 1: Ethics for Management Accountants | Subunit 1: Ethics for Management Accountants
Question: 5
If an IMA member discovers unethical conduct in his or her organization and fails to act, (s)he will be in violation of which of IMA’s ethical standard(s)?
A.
“Refrain from engaging in any conduct that would prejudice carrying out duties correctly.”
B.
“Communicate information fairly and objectively.”
C.
“Disclose all relevant information that could reasonably be expected to influence an intended user’s understanding of reporting analyses or recommendations.”
D.
All of the answers are correct.
Answer (D) is correct. An IMA member displays his or her competence and credibility and maintains integrity by taking the appropriate action within the organization to resolve an ethical problem. All of these activities should be a part of an IMA member’s normal job processes.
Study Unit 1: Ethics for Management Accountants | Subunit 1: Ethics for Management Accountants
Question: 6
IMA’s Statement of Ethical Professional Practice requires an IMA member to follow the established policies of the organization when faced with an ethical conflict. If these policies do not resolve the conflict, the member should
A.
Consult the board of directors immediately.
B.
Discuss the problem with the immediate superior if (s)he is involved in the conflict.
C.
Communicate the problem to authorities outside the organization.
D.
Contact the next higher managerial level if initial presentation to the immediate superior does not resolve the conflict.
Answer (D) is correct. In these circumstances, the problem should be discussed with the immediate superior unless (s)he is involved. In that case initial presentation should be to the next higher managerial level. If the problem is not satisfactorily resolved after initial presentation, the question should be submitted to the next higher level.
Study Unit 1: Ethics for Management Accountants | Subunit 1: Ethics for Management Accountants
Question: 7
In which situation is an IMA member permitted to communicate confidential information to individuals or authorities outside the firm?
A.
There is an ethical conflict and the board has refused to take action.
B.
Such communication is legally prescribed.
Answer (B) is correct. According to IMA’s Statement of Ethical Professional Practice, members are responsible for observing the standard of confidentiality. Thus, the IMA member should “refrain from disclosing confidential information acquired in the course of his or her work except when authorized, unless legally obligated to do so.”
C.
The IMA member knowingly communicates the information indirectly through a subordinate.
D.
An officer at the IMA member’s bank has requested information on a transaction that could influence the firm’s stock price.
Study Unit 1: Ethics for Management Accountants | Subunit 1: Ethics for Management Accountants
Question: 8
Which ethical standard is most clearly violated if an IMA member knows of a problem that could mislead users but does nothing about it?
A.
Competence.
B.
Legality.
C.
Credibility.
Answer (C) is correct. Credibility is the fourth part of IMA’s Statement of Ethical Professional Practice. It requires that information be communicated “fairly and objectively,” and that all information that could reasonably influence users be disclosed.
D.
Confidentiality.
Study Unit 1: Ethics for Management Accountants | Subunit 1: Ethics for Management Accountants
Question: 9
IMA’s Statement of Ethical Professional Practice includes an integrity standard, which requires an IMA member to
A.
Decline to become a minority partner in a partnership that is a supplier of the member’s employer.
Answer (A) is correct. One of the responsibilities of an IMA member under the integrity standard is to “refrain from engaging in any conduct that would prejudice carrying out duties ethically.”
B.
Report any relevant information that could influence users of financial statements.
C.
Disclose confidential information when authorized by his or her firm or required under the law.
D.
Refuse gifts from anyone.
Study Unit 1: Ethics for Management Accountants | Subunit 1: Ethics for Management Accountants
Question: 10
IMA’s Statement of Ethical Professional Practice includes a competence standard, which requires an IMA member to
A.
Report information, whether favorable or unfavorable.
B.
Develop his or her professional proficiency on a continual basis.
Answer (B) is correct. One of the responsibilities of an IMA member under the competence standard is to “maintain an appropriate level of professional expertise by continually developing knowledge and skills.”
C.
Discuss ethical conflicts and possible courses of action with an unbiased counselor.
D.
Discuss, with subordinates, their responsibilities regarding the disclosure or information about the firm.
Study Unit 1: Ethics for Management Accountants | Subunit 1: Ethics for Management Accountants
Question: 11
Sheila is a financial manager who has discovered that her company is violating environmental regulations. If her immediate superior is involved, her appropriate action is to
A.
Do nothing since she has a duty of loyalty to the organization.
B.
Consult the audit committee.
C.
Present the matter to the next higher managerial level.
Answer (C) is correct. To resolve an ethical problem, the financial manager/management accountant’s first step is usually to consult his or her immediate superior. If that individual is involved, the matter should be taken to the next higher level of management.
D.
Confront her immediate superior.
Study Unit 1: Ethics for Management Accountants | Subunit 1: Ethics for Management Accountants
Question: 12
IMA members are obligated to maintain the highest standards of ethical conduct. Accordingly, IMA’s Statement of Ethical Professional Practice explicitly requires that IMA members
A.
Obtain sufficient competent evidence when expressing an opinion.
B.
Not condone violations by others.
Answer (B) is correct. The principles section of IMA’s Statement of Ethical Professional Practice, “IMA’s overarching ethical principles include: Honesty, Fairness, Objectivity, and Responsibility. Members shall act in accordance with these principles and shall encourage others within their organizations to adhere to them.”
C.
Comply with generally accepted auditing standards.
D.
Adhere to generally accepted accounting principles.
Study Unit 1: Ethics for Management Accountants | Subunit 1: Ethics for Management Accountants
Question: 13
Integrity is an ethical requirement for all IMA members. One aspect of integrity requires
A.
Performance of professional duties in accordance with relevant laws.
B.
Avoidance of conflict of interest.
Answer (B) is correct. According to IMA’s Statement of Ethical Professional Practice, IMA members must “mitigate actual conflicts of interest. Regularly communicate with business associates to avoid apparent conflicts of interest. Advise all parties of any potential conflicts.”
C.
Refraining from using confidential information for unethical or illegal advantage.
D.
Maintenance of an appropriate level of professional expertise.
Study Unit 1: Ethics for Management Accountants | Subunit 1: Ethics for Management Accountants
Question: 14
Under the express terms of IMA’s Statement of Ethical Professional Practice, an IMA member may not
A.
Advertise.
B.
Encroach on the practice of another IMA member.
C.
Disclose confidential information unless authorized or legally required.
Answer (C) is correct. IMA members may not disclose confidential information acquired in the course of their work unless authorized or legally required to do so. They must also “inform all relevant parties regarding appropriate use of confidential information. Monitor subordinates’ activities to ensure compliance.”
D.
Accept other employment while serving as a financial manager or management accountant.
Study Unit 1: Ethics for Management Accountants | Subunit 1: Ethics for Management Accountants
Question: 15
An IMA member discovers a problem that could mislead users of the firm’s financial data and has informed his or her immediate superior. (S)he should report the circumstances to the audit committee and/or the board of directors only if
A.
The immediate superior, who reports to the chief executive officer, knows about the situation but refuses to correct it.
B.
The immediate superior assures the member that the problem will be resolved.
C.
The immediate superior reports the situation to his or her superior.
D.
The immediate superior, the firm’s chief executive officer, knows about the situation but refuses to correct it.
Answer (D) is correct. According to IMA’s Statement of Ethical Professional Practice, an IMA member should “discuss the issue with your immediate supervisor except when it appears that the supervisor is involved. In that case, present the issue to the next level. If you cannot achieve a satisfactory resolution, submit the issue to the next management level. If your immediate supervisor is the chief executive office or equivalent, the acceptable reviewing authority may be a group such as the audit committee, executive committee, board of directors, board of trustees, or owners.”
Study Unit 1: Ethics for Management Accountants | Subunit 1: Ethics for Management Accountants
Question: 16
Recently, Fan Club, Inc., submitted to management a budget for the coming year. Included in the budget were the plans for a new product, a rechargeable fan. The new fan will not only last longer than the competitor’s product but is also more quiet. While not yet approved, the budget called for aggressive advertising to support its sales targets, as the business community was not yet aware that Fan Club was close to production of a new fan. A member of the management
accounting staff “shared” the budget with a distributor. In accordance with IMA’s Statement of Ethical Professional Practice, which one of the following would best represent an ethical conflict in this situation?
A.
The budget has not been approved and therefore is not for publication.
B.
The price has not been established, so expectations must be managed.
C.
The staff member exposed the company to a potential lawsuit.
D.
The employee should refrain from disclosing confidential information.
Answer (D) is correct. IMA’s Statement of Ethical Professional Practice states that every member has a responsibility to keep information confidential except when disclosure is authorized or legally required.
Study Unit 1: Ethics for Management Accountants | Subunit 1: Ethics for Management Accountants
Question: 17
A new management accountant is concerned about complying with the ethical standard of competence in the IMA’s Statement of Ethical Professional Practice. Which one of the following is not required under the standard of competence?
A.
Maintain expertise in all areas of accounting.
Answer (A) is correct. Maintaining expertise in all areas of accounting would be a difficult task. According to the ethical standard of competence in the IMA’s Statement of Ethical Professional Practice, a CMA only needs to recognize and communicate professional limitations or other constraints that would preclude responsible judgment or successful performance of an activity.
B.
Continually develop knowledge and skills.
C.
Perform duties in accordance with relevant regulations and standards.
D.
Provide recommendations that are accurate and timely.
Study Unit 1: Ethics for Management Accountants | Subunit 1: Ethics for Management Accountants
Question: 18
Scott Jon, a new accounting clerk at a firm that had recently terminated several employees due to budgetary cutbacks, accidentally viewed his supervisor’s biweekly paycheck. Not realizing that the paycheck included an annual bonus, Jon erroneously multiplied the gross pay by 26 to find annual earnings. Jon was amazed that his supervisor appeared to earn more than twice the local average for employees in an accounting supervisory position. Jon discussed this situation with a friend, a recently terminated employee of the company who now worked for a local newspaper. As a result of this discussion, the supervisor’s “outrageous” salary was made public. Which one of the standards of the IMA’s Statement of Ethical Professional Practice did Jon’s actions violate?
A.
Competence.
B.
Confidentiality.
Answer (B) is correct. The standard of confidentiality states each member has a responsibility to (1) keep information confidential except when disclosure is authorized or legally required, (2) inform all relevant parties regarding appropriate use of confidential information and monitor subordinates’ activities to ensure compliance, and (3) refrain from using confidential information for unethical or illegal advantage.
C.
Integrity.
D.
Credibility.
Study Unit 1: Ethics for Management Accountants | Subunit 1: Ethics for Management Accountants
Question: 19
Chris Benedict was recently tried and convicted in court for producing and selling illegal narcotic drugs. None of the activity occurred during work hours, and Benedict performed duties as a management accountant without incident during the period of illegal activity. Benedict has argued that because the illegal activity was unrelated to Benedict’s service as a management accountant, no ethical violation had been committed. Which provision of the IMA’s Statement of Ethical Professional Practice is most likely to apply to Benedict’s illegal actions?
A.
Competence.
B.
Confidentiality.
C.
Integrity.
Answer (C) is correct. The integrity standard states that each member has a responsibility to abstain from engaging in or supporting any activity that might discredit the profession. Being convicted of producing and selling illegal narcotic drugs is an activity that would discredit the profession.
D.
Credibility.
Study Unit 1: Ethics for Management Accountants | Subunit 1: Ethics for Management Accountants
Question: 20
At NC Corporation, year-end bonuses at each branch office are based on branch profitability. Due to a slow economy, profitability through the third quarter at the Northeast branch is under budget. To address this issue, the accounting staff at the Northeast branch develops a list of end-of-year actions designed to boost earnings for the year. Which one of the following is most likely to violate IMA’s Statement of Ethical Professional Practice?
A.
Requesting the branch’s advertising agency to delay billing third-quarter advertisements until January.
Answer (A) is correct. IMA’s Statement of Ethical Professional Practice states that its members have responsibilities in the areas of competence, confidentiality, integrity, and credibility. Requesting the branch’s advertising agency to delay billing third quarter advertisements until January will most likely violate IMA’s Statement of Ethical
Professional Practice in the areas of integrity and credibility. Integrity is impaired because an IMA member should abstain from engaging in or supporting any activity that might discredit the profession, and credibility is impaired because an IMA member should disclose all relevant information that could reasonably be expected to influence an intended user’s understanding of the reports, analyses, or recommendations.
B.
Deferring advertising expense by reducing the number of newspaper advertisements run in the third quarter.
C.
Deferring planned painting and refurbishment of the warehouse until the following year.
D.
Offering additional discounts to customers to entice them to increase purchases in the third quarter.
Study Unit 1: Ethics for Management Accountants | Subunit 1: Ethics for Management Accountants
Question: 21
Mark Tian, a staff accountant, becomes aware of an off-balance-sheet bank account where funds have been diverted with offsetting credits approved by his immediate supervisor. His immediate supervisor refuses to discuss it and suggests Tian forget about it. Which one of the following should be Tian’s next course of action in this circumstance?
A.
Put concerns in writing to the immediate supervisor and copy the company’s independent auditor.
B.
Discuss concerns with the level of management above the immediate supervisor.
Answer (B) is correct. In applying the Standards of Ethical Professional Practice, it may be necessary to identify unethical behavior or resolve an ethical conflict. When faced with ethical issues, it is important to follow an organization’s established policies on the resolution of such conflict. If these policies do not resolve the ethical conflict, the first option involves discussing the issue with the immediate superior. However, when it appears that the supervisor is involved, the issue should be presented to the next level.
C.
Communicate concerns confidentially to the company’s independent auditor.
D.
Communicate concerns confidentially to the company’s external legal counsel.
Study Unit 1: Ethics for Management Accountants | Subunit 1: Ethics for Management Accountants
Question: 22
A company has a December 31 year end. Which one of the following options to increase net income during the last month of the company’s fiscal year end would least likely result in a violation of the IMA’s Statement of Ethical Professional Practice?
A.
Persuade suppliers to postpone billing until January .
B.
Delay the year-end closing until January 4 to capture sales over the New Year’s holiday in the current year.
C.
Reduce the calculated allowance for bad debts and bad debt expenses.
D.
Postpone planned marketing expenditures until January.
Answer (D) is correct. Postponing planned marketing expenditures until a future date is least likely a violation of the IMA’s Statement of Ethical Professional Practice because as long as the marketing funds have not been expended, the expense need not be recorded until the following period.
Study Unit 1: Ethics for Management Accountants | Subunit 1: Ethics for Management Accountants
Question: 23
You have examined your organization’s financial statements and determined that they present a number of significant items in a fraudulent manner. You know that you should report this situation to management but are concerned that reporting it might result in your employment being terminated. Failure to report this situation is a violation of which of the ethical standard(s) outlined in IMA’s Statement of Ethical Professional Practice?
A.
Credibility.
Answer (A) is correct. The IMA’s Statement of Ethical Professional Practice’s ethical standard of credibility involves communicating information fairly and objectively; disclosing all relevant information that could reasonably be expected to influence an intended user’s understanding of the reports, analyses, or recommendations; and disclosing delays or deficiencies in information, timeliness, processing, or internal controls in conformance with organization policy and/or applicable law. Credibility has been violated because presenting a number of significant items in a fraudulent manner does not communicate information fairly. The relevant information that could reasonably be expected to influence an intended user’s understanding is also not completely disclosed.
B.
Confidentiality.
C.
Competence.
D.
Confidentiality and Integrity.
Study Unit 1: Ethics for Management Accountants | Subunit 1: Ethics for Management Accountants
Question: 24
An accountant has concerns that a particular transaction is being recorded in a manner that does not reflect the nature of the transaction and believes that alternative accounting is being used to avoid recording the appropriate expense for a period. The accountant attempted to speak to the accounting manager, but the manager rebuffed the accountant and said there was no time to discuss the issue further. According to IMA’s Statement of Ethical Professional Practice, what is the next appropriate step the accountant should take regarding this issue?
A.
Discuss the issue with the next level of management over the accounting manager.
Answer (A) is correct.
When faced with ethical issues, the accountant should follow the organization’s established policies on the resolution of such conflict. If these policies do not resolve the ethical conflict, (s)he should first discuss the issue with his or her immediate superior except when it appears that the supervisor is involved. In that case, (s)he should present the issue to the next level.
B.
Discuss the issue with the company’s independent accounting firm.
C.
Discuss the issue with the firm’s legal counsel.
D.
Resign from the position with the company.
Study Unit 1: Ethics for Management Accountants | Subunit 1: Ethics for Management Accountants
Question: 25
An accountant has frequent business contact with customers, suppliers, and creditors in the course of performing professional duties. Which of the following circumstances would most likely threaten the accountant’s adherence to the ethical principles and standards of IMA’s Statement of Ethical Professional Practice?
A.
The accountant accepted two World Cup tickets from a supplier and flew to the stadium in the company’s jet along with a number of the supplier’s executives.
Answer (A) is correct. The accountant’s acceptance of two World Cup tickets from a supplier and flying to the stadium in the company’s jet along with a number of the supplier’s executives would threaten the accountant’s adherence to the integrity standard. The integrity standard states that each member has a responsibility to mitigate actual conflicts of interests, regularly communicate with business associates to avoid apparent conflicts of interest, and advise all parties of any potential conflicts. In order to avoid conflicts of interest, employees should refuse any gift, favor, or hospitality that would influence or appear to influence their actions.
B.
The accountant speaks quarterly with analysts regarding the company’s past performance and future prospects.
C.
The accountant attends a professional conference where the accountant goes out to dinner and socializes with accountants from other companies in the industry.
D.
The accountant attends a charity event at the invitation of the company’s audit firm, which bought two tables of tickets to support the event.
Study Unit 1: Ethics for Management Accountants | Subunit 1: Ethics for Management Accountants
Question: 26
You have examined your organization’s financial statements and determined that they present a number of significant items in a fraudulent manner. You know that you should report this situation to management but are concerned that reporting it might result in your employment being terminated. Failure to report this situation is a violation of which of the ethical standard(s) outlined in IMA’s Statement of Ethical Professional Practice?
A.
Credibility.
Answer (A) is correct. The IMA’s Statement of Ethical Professional Practice’s ethical standard of credibility involves communicating information fairly and objectively; disclosing all relevant information that could reasonably be expected to influence an intended user’s understanding of the reports, analyses, or recommendations; and disclosing delays or deficiencies in information, timeliness, processing, or internal controls in conformance with organization policy and/or applicable law. Credibility has been violated because presenting a number of significant items in a fraudulent manner does not communicate information fairly. The relevant information that could reasonably be expected to influence an intended user’s understanding is also not completely disclosed.
B.
Confidentiality.
C.
Competence.
D.
Confidentiality and Integrity.
Study Unit 1: Ethics for Management Accountants | Subunit 2: Corporate Ethics and Legislation
Question: 27
The Foreign Corrupt Practices Act prohibits
A.
Bribes to all foreigners.
B.
Small bribes to foreign officials that serve as facilitating or grease payments.
C.
Bribery only by corporations and their representatives.
D.
Bribes to foreign officials to influence official acts.
Answer (D) is correct. The Foreign Corrupt Practices Act (FCPA) prohibits any U.S. firm from making bribes to foreign officials to influence official acts. The businesses subject to the FCPA include corporations, partnerships, limited partnerships, business trusts, and unincorporated organizations. Violations of the FCPA are federal felonies. The penalties are up to 5 years in prison or up to a $100,000 fine or both for an officer, director, or shareholder who helps make the bribe.
Study Unit 1: Ethics for Management Accountants | Subunit 2: Corporate Ethics and Legislation
Question: 28
A major impact of the Foreign Corrupt Practices Act of 1977 is that registrants subject to the Securities Exchange Act of 1934 are now required to
A.
Keep records that reflect the transactions and dispositions of assets and to maintain a system of internal accounting controls.
Answer (A) is correct. The main purpose of the Foreign Corrupt Practices Act of 1977 is to prevent bribery by firms that do business in foreign countries. A major ramification is that it requires all companies that must register with the SEC under the Securities Exchange Act of 1934 to maintain adequate accounting records and a system of
internal accounting control.
B.
Provide access to records by authorized agencies of the federal government.
C.
Prepare financial statements in accord with international accounting standards.
D.
Produce full, fair, and accurate periodic reports on foreign commerce and/or foreign political party affiliations.
Study Unit 1: Ethics for Management Accountants | Subunit 2: Corporate Ethics and Legislation
Question: 29
The reporting of accounting information plays a central role in the regulation of business operations. The importance of sound internal control practices is underscored by the Foreign Corrupt Practices Act of 1977, which requires publicly owned U.S. corporations to maintain systems of internal control that meet certain minimum standards. Preventive controls are an integral part of virtually all accounting processing systems, and much of the information generated by the accounting system is used for preventive control purposes. Which one of the following is not an essential element of a sound preventive control system?
A.
Separation of responsibilities for the recording, custodial, and authorization functions.
B.
Sound personnel practices.
C.
Documentation of policies and procedures.
D.
Implementation of state-of-the-art software and hardware.
Answer (D) is correct. Preventive controls are designed to prevent an error or irregularity from occurring. State-of-the-art hardware and software would presumably incorporate the latest control features, but a less advanced system could very well contain a sound preventive control structure. Hence, state-of-the-art components are not essential for effective control.
Study Unit 1: Ethics for Management Accountants | Subunit 2: Corporate Ethics and Legislation
Question: 30
What law prohibits U.S. companies from paying bribes to foreign officials for the purpose of obtaining or retaining business?
A.
Federal Ethical Standards Act.
B.
Robinson-Patman Act.
C.
Foreign Corrupt Practices Act.
Answer (C) is correct. The Foreign Corrupt Practices Act of 1977 prohibits bribes to foreign officials for purposes of obtaining or retaining business. The Act also requires companies to maintain effective systems of internal control.
D.
North American Free Trade Agreement.
Study Unit 1: Ethics for Management Accountants | Subunit 2: Corporate Ethics and Legislation
Question: 31
Which of the following is not an aspect of the Foreign Corrupt Practices Act of 1977?
A.
It subjects management to fines and imprisonment.
B.
It prohibits bribes to foreign officials.
C.
It requires the establishment of independent audit committees.
Answer (C) is correct. The Foreign Corrupt Practices Act of 1977 prohibits bribes to foreign officials and requires firms to have adequate systems of internal control. Violation of the Act subjects individual managers to fines and/or imprisonment. The Act does not specifically require the establishment of audit committees, but many firms have established audit committees as one means of dealing with the internal control provisions of the Act.
D.
It requires an internal control system to be developed and maintained.
Study Unit 1: Ethics for Management Accountants | Subunit 2: Corporate Ethics and Legislation
Question: 32
Firms subject to the reporting requirements of the Securities Exchange Act of 1934 are required by the Foreign Corrupt Practices Act of 1977 to maintain satisfactory internal control. The role of the independent auditor relative to this act is to
A.
Report clients with unsatisfactory internal control to the SEC.
B.
Provide assurances to users as part of the traditional audit attest function that the client is in compliance with the present legislation.
C.
Express an opinion on the sufficiency of the client’s internal control to meet the requirements of the Act.
D.
Attest to the financial statements.
Answer (D) is correct. Whether a client is in conformity with the Foreign Corrupt Practices Act is a legal question. Auditors cannot be expected to provide clients or users of the financial statements with legal advice. The role of the auditor is to assess control risk in the course of an engagement to attest to the fair presentation of the financial statements.
Study Unit 1: Ethics for Management Accountants | Subunit 2: Corporate Ethics and Legislation
Question: 33
The requirement of the Foreign Corrupt Practices Act of 1977 to devise and maintain adequate internal control is assigned in the act to the
A.
Chief financial officer.
B.
Board of directors.
C.
Director of internal auditing.
D.
Company as a whole with no designation of specific persons or positions.
Answer (D) is correct. The accounting requirements apply to all public companies that must register under the Securities Exchange Act of 1934. The responsibility is thus placed on companies, not individuals.
Study Unit 1: Ethics for Management Accountants | Subunit 2: Corporate Ethics and Legislation
Question: 34
Which of the following corporations are subject to the accounting requirements of the Foreign Corrupt Practices Act (FCPA)?
A.
All corporations engaged in interstate commerce.
B.
All domestic corporations engaged in international trade.
C.
All corporations that have made a public offering under the Securities Act of 1933.
D.
All corporations whose securities are registered pursuant to the Securities Exchange Act of 34.
Answer (D) is correct. The accounting requirements of the FCPA apply to all companies required to register and report under the Securities Exchange Act of 1934. These companies must maintain books, records, and accounts in reasonable detail that accurately and fairly reflect transactions. The FCPA also requires these companies to maintain a system of internal accounting control that provides certain reasonable assurances, including that corporate assets are not used for bribes.
Study Unit 1: Ethics for Management Accountants | Subunit 2: Corporate Ethics and Legislation
Question: 35
The Foreign Corrupt Practices Act of 1977 prohibits bribery of foreign officials. Which of the following statements correctly describes the act’s application to corporations engaging in such practices?
A.
It applies only to multinational corporations.
B.
It applies to all domestic corporations engaged in interstate commerce.
Answer (B) is correct. Although the requirements of the FCPA relating to the maintenance of accounting records and systems of internal accounting control apply only to companies required to register under the Securities Exchange Act of 1934, the antibribery provisions apply to all domestic business concerns engaged in interstate commerce.
C.
It applies only to corporations whose securities are registered under the Securities Exchange Act of 1934.
D.
It applies only to corporations engaged in foreign commerce.
Study Unit 1: Ethics for Management Accountants | Subunit 2: Corporate Ethics and Legislation
Question: 36
Under the Foreign Corrupt Practices Act (FCPA), an action may be brought that seeks
A.
Treble damages by a private party.
B.
Injunctive relief by a private party.
C.
Criminal sanctions against both the corporation and its officers by the Department of Justice.
Answer (C) is correct. The SEC may investigate violations of the FCPA, bring civil actions for its enforcement, and recommend that the Justice Department prosecute criminal violations.
D.
Damages and injunctive relief by the Securities and Exchange Commission.
Study Unit 1: Ethics for Management Accountants | Subunit 2: Corporate Ethics and Legislation
Question: 37
The U.S. Foreign Corrupt Practices Act is particularly focused on the dealings of financial institutions and the safeguarding of the global financial system. Financial institutions must implement robust controls to ensure knowledge of their customers and the nature of their business transactions and be in a position to prove to regulators a high level of due diligence. These safeguards are required to minimize all of the following except
A.
Money laundering.
B.
Insider trading.
Answer (B) is correct. The safeguards of the global financial system relating to the U.S. Foreign Corrupt Practices Act deal with minimizing money laundering, terrorist financing, and extortion and bribery. Insider trading is not a focus of the safeguards.
C.
Terrorist financing.
D.
Extortion and bribery.
Study Unit 1: Ethics for Management Accountants | Subunit 2: Corporate Ethics and Legislation
Question: 38
Corporations have the responsibility to issue financial statements that are timely, accurate, and transparent, reflecting all the transactions of the company. Which of the following documents refer to this responsibility?
I. IMA’s Statement of Ethical Professional Practice
II. SOX Section 406: Code of Ethics for Senior Financial Officers
III. IMA’s Statement on Management Accounting “Values and Ethics: From Inception to Practice”
IV. U.S. Foreign Corrupt Practices Act
A.
I and II only.
B.
I and III only.
C.
II and III only.
D.
II and IV only.
Answer (D) is correct. SOX Section 406: Code of Ethics for Senior Financial Officers and the U.S. Foreign Corrupt Practices Act both refer to the corporate responsibility to issue financial statements that are timely, accurate, and transparent, reflecting all the transactions of the company.
Study Unit 1: Ethics for Management Accountants | Subunit 2: Corporate Ethics and Legislation
Question: 39
Which of the following best describes an important provision of the U.S. Foreign Corrupt Practices Act?
A.
Auditors cannot provide bookkeeping or other services related to the accounting records or financial statements of the audit client.
B.
Companies must follow the laws of the their home country as well as the laws of the countries where any foreign subsidiaries are located.
C.
The CEO and CFO must certify that they have no knowledge of any corrupt practices occurring in any overseas subsidiaries of U.S. companies.
D.
The internal accounting controls should be examined, and if material weaknesses are found, controls must be strengthened.
Answer (D) is correct. The FCPA states that all public companies registered under the 1934 Act must devise and maintain a system of internal accounting control sufficient to provide reasonable assurance against material misstatements.
Study Unit 1: Ethics for Management Accountants | Subunit 2: Corporate Ethics and Legislation
Question: 40
Which one of the following statements best characterizes the provisions of the Foreign Corrupt Practices Act (FCPA)?
A.
The FCPA requires corporations to keep records and accounts in sufficient detail to reflect transactions.
Answer (A) is correct. The FCPA was enacted for the purpose of making it unlawful to make payments to foreign government officials to assist in obtaining or retaining business. Corporations are thus required to keep records to reflect all transactions so that they will not be covering up this transaction.
B.
The FCPA requires compliance with corporate codes of conduct to be reviewed and
reported on by external auditors.
C.
The FCPA provides for treble damages in civil cases brought under the law.
D.
The FCPA provides for criminal penalties for auditors who fail to report a corporation’s participation in bribery.
Study Unit 1: Ethics for Management Accountants | Subunit 2: Corporate Ethics and Legislation
Question: 41
Which of the following issues is addressed by Section 406 of the Sarbanes-Oxley Act?
I. Full, fair, timely, and accurate financial statement disclosure.
II. Whistleblower protection.
III. Form 8-K disclosure of changes to the Ethics Code for Senior Financial Officers.
IV. Compliance with the U.S. Foreign Corrupt Practices Act.
V. Reporting the existence of an Ethics Code for Senior Financial Officers.
A.
I, II, III, and IV only.
B.
II, IV, and V only.
C.
I, III, and V only.
Answer (C) is correct. Section 406 of the Sarbanes-Oxley Act addresses full, fair, timely, and accurate financial statement disclosure; Form 8-K disclosure of changes to the Ethics Code for Senior Financial Officers; and reporting the existence of an Ethics Code for Senior Financial Officers.
D.
III and IV only.
Study Unit 1: Ethics for Management Accountants | Subunit 2: Corporate Ethics and Legislation
Question: 42
Which of the following provisions are covered in the U.S Foreign Corrupt Practices Act?
I. Illegal payments to foreign officials to assist in obtaining business.
II. Transparency of accounting records reflecting all transactions.
III. Payments to agents for the purpose of influencing foreign officials.
IV. Maintenance of an adequate system of internal controls.
A.
I only.
B.
I and III only.
C.
II, III, and IV only.
D.
I, II, III, and IV.
Answer (D) is correct. The U.S. Foreign Corrupt Practices Act covers illegal payments to foreign officials
to assist in obtaining business, transparency of accounting records reflecting all transactions, payments to agents for the purpose of influencing foreign officials, and maintenance of an adequate system of internal controls.
Study Unit 1: Ethics for Management Accountants | Subunit 3: Corporate Responsibility for Ethical Behavior
Question: 43
Which one of the following is a true statement regarding organizational ethics?
A.
As long as officer and employee behavior meet the requirements of the law, the organization can be considered to have a functioning system of ethical behavior.
B.
A strong sense of ethics on the part of employees who are in the best position to appropriate cash and other assets is the most vital part of a functioning system of ethical behavior.
C.
If an organization has a strong code of ethical conduct in place, the role of employee training can be downplayed.
D.
Paying attention to “whistleblowers” plays a significant role in maintaining an effective ethical atmosphere.
Answer (D) is correct. “Values and Ethics: From Inception to Practice” states, in part, “A whistleblowing framework (e.g., an ethics helpline) is an important component in maintaining an ethical organizational culture. An effective feedback system includes having a confidential framework for employees to report possible violations of the organization’s code of ethics and to receive advice on the ethical aspects of challenging decisions. Statistics show that a large number of occupational fraud cases are detected through an employee “hotline” or other reporting method ... ” (IX. Measuring and Improving Ethical Compliance.)
Study Unit 1: Ethics for Management Accountants | Subunit 3: Corporate Responsibility for Ethical Behavior
Question: 44
IMA’s Statement on Management Accounting, “Values and Ethics: From Inception to Practice,” recommends a defined code of conduct and ethical behavior for all organizations. One advantage of having such a code is that it
A.
Provides employees with guidance for handling unfamiliar situations.
Answer (A) is correct. “Values and Ethics: From Inception to Practice” states, in part, “... what does an employee do when unplanned events occur? What reference does an individual look to for help in making decisions? ... This is why it is important to have a defined set of organizational values and code of ethics – they create the “touchstone” against which every unanticipated decision must be judged. Failure to have every individual in the organization know and understand these values and ethical code leads to inconsistency and, in the worst cases, unethical or fraudulent
behavior.” (IV. Values, Ethics, and Accounting.)
B.
Ensures ethical behavior by all employees.
C.
Shields the organization from liability in cases of loss of stockholder value due to fraud.
D.
Eases the investigative process performed by police and prosecutors in cases of suspected fraud.
Study Unit 1: Ethics for Management Accountants | Subunit 3: Corporate Responsibility for Ethical Behavior
Question: 45
Which one of the following is a true statement regarding organizational ethics?
A.
A comprehensive framework of corporate ethical behavior is a prerequisite for an effective system of internal control.
Answer (A) is correct. A comprehensive framework of corporate ethical behavior is a prerequisite for an effective system of internal control. “Values and Ethics: From Inception to Practice” states, in part, “CEOs and CFOs have to place their own integrity on the line by attesting to compliance with an adequate level of internal controls (as well as all other certifications). Creating a thorough, integrated system for developing, implementing, sustaining, and monitoring ethical performance within the organization will allow executives to make such declarations with confidence that a code of ethics is the foundation of the organization’s culture and is fully integrated into the thinking process of every employee and business partner.” (IX. Measuring and Improving Ethical Compliance.)
B.
An effective system of internal control is a prerequisite for corporate ethical behavior.
C.
If a functioning system of ethical behavior is in place, an organization is able to devote fewer resources to developing human capital.
D.
“Organizational culture” is determined mostly by the industry(ies) in which the firm operates.
Study Unit 1: Ethics for Management Accountants | Subunit 3: Corporate Responsibility for Ethical Behavior
Question: 46
A company’s code of conduct states, “Our employees are our most valuable asset.” Which one of the following policies best illustrates that management strives to provide leadership by example in ethical matters concerning employees?
A.
The company relies on supervisors rather than manuals to train employees in their responsibilities.
B.
Final terms on all major purchase and sales contracts are negotiated only by management.
C.
Management and the board of directors meet annually at a luxury resort for a strategic planning conference.
D.
Management declines to accept bonuses earned in any year in which no raises are given to employees.
Answer (D) is correct. This illustrates that management does not hold themselves to a different standard than employees. This shows that the company values its employees.
Study Unit 1: Ethics for Management Accountants | Subunit 3: Corporate Responsibility for Ethical Behavior
Question: 47
Which of the following statements describe the importance of a whistleblowing framework in maintaining an ethical organizational culture?
I. It provides measurable feedback for determining whether employees are following a code of ethics.
II. It creates opportunities to enhance and improve internal controls.
III. It empowers management to become better role models for employees.
IV. It helps to identify potential errors or risks at each task level within the organization.
A.
I and II only.
Answer (A) is correct. The collection, analysis, and summarization of ethics issues can provide insight into the operation of its code of ethics and the degree to which employees are following it. In addition, tracking and monitoring issues raised through a whistleblowing framework creates opportunities to enhance and improve internal control. For instance, a large number of fraud cases are reported through a whistleblowing framework, which can highlight areas where internal controls were lacking.
B.
II and IV only.
C.
III and IV only.
D.
I, III, and IV only.
Study Unit 1: Ethics for Management Accountants | Subunit 3: Corporate Responsibility for Ethical Behavior
Question: 48
Quentin Jam is a new accountant assisting in the month-end close of the books for Sheldrake, Ltd. His supervisor told him to accrue a large receivable and said that he would provide the supporting documentation later. Jam made the accrual, and the books were closed. Subsequently, Jam found out that the company would have missed the earnings estimate without the receivable. Jam requested the documentation, but the supervisor could not provide it. Other associates told Jam that this supervisor had directed that undocumented entries be recorded in the books in the past and that the former accountant had left Sheldrake because he was uncomfortable making the entries. Recommend the best course of action for Jam.
A.
Jam should evaluate the materiality of the receivable to determine if it is worthy of follow-up.
B.
Jam should confront his supervisor about the undocumented receivable that appeared to be inaccurate.
C.
Jam should present this issue to his supervisor’s manager to resolve it.
Answer (C) is correct. When faced with ethical issues, the accountant should follow the organization’s established policies on the resolution of such conflict. If these policies do not resolve the ethical conflict, (s)he should first discuss the issue with his or her immediate superior except when it appears that the supervisor is involved. In that case, (s)he should present the issue to the next level. Since Jam’s supervisor is involved in the conflict, Jam should present the issue to his supervisor’s manager.
D.
Jam should report to the board of directors that Sheldrake did not truly meet the earnings estimate.
Study Unit 1: Ethics for Management Accountants | Subunit 3: Corporate Responsibility for Ethical Behavior
Question: 49
Games Unlimited has convened a group of employees to review the company’s code of ethics and propose revisions and improvements. One of the suggested improvements is the development of a whistleblowing framework as recommended by IMA’s Statement on Management Accounting, “Values and Ethics: From Inception to Practice.” This framework will provide all of the following benefits except
A.
Opportunities to enhance and improve internal controls.
B.
A confidential means for employees to report possible violations.
C.
A method for defining the organization’s behavioral values.
Answer (C) is correct. An effective feedback system includes having a confidential framework for employees to report possible violations of the organization’s code of ethics. Whichever approach an organization chooses, the collection, analysis, and summarization of ethics issues can provide insight into the operation of its code of ethics and the degree to which employees are following it. In addition, tracking and monitoring issues raised through a whistleblowing framework creates opportunities to enhance and improve internal controls. However, a whistleblowing framework does not define the organization’s behavioral values.
D.
A means for the collection, analysis, and summarization of ethical issues.
Study Unit 1: Ethics for Management Accountants | Subunit 3: Corporate Responsibility for Ethical Behavior
Question: 50
The management team of GranMark, Inc., attended an ethics training session at the IMA Annual Conference and subsequently made plans to enhance GranMark’s ethics program. The president plans to chair a committee of employees to review the company’s behavioral values, while the CFO intends to review the ethical standards applicable to the Finance Department. The manager of the Human Resources Department will investigate the feasibility of establishing a whistleblowing framework that includes a “hotline” for reporting ethics violations. These activities at GranMark exemplify
A.
Organizational transparency.
B.
Leadership by example.
Answer (B) is correct. “Tone at the top” plays an important role in determining an organization’s ethical environment. For a code of ethics to be effective, its application must be demonstrated by those in positions of power and leadership. Leaders must be seen living and managing by the code of ethics.
C.
Measurement of ethical compliance.
D.
Alignment of internal controls with ethical standards.
Study Unit 1: Ethics for Management Accountants | Subunit 3: Corporate Responsibility for Ethical Behavior
Question: 51
In order for an ethics code to become a reality in practice, every aspect of a company’s activity should be affected by the code. Ethical behaviors should focus not only on clients and customers but also on employees, society at large, shareholders, and suppliers. All activities, from design and development through after-sales support and services, should also be considered when applying a company’s ethical principles. When focusing on society at large, ethical considerations would most likely include
A.
Fair value, cycle time, quality, and service warranty.
B.
Reputation, risk, cost/benefit, and value-stream return.
C.
Negotiation, problem resolution, delivery, and inventory support.
D.
Environment, resource usage, outage impact, and waste/disposal.
Answer (D) is correct. The ethical considerations of environment, resource usage, outage impact, and waste/disposal focus on social responsibility to benefit the society at large.
Study Unit 1: Ethics for Management Accountants | Subunit 3: Corporate Responsibility for Ethical Behavior
Question: 52
Which one of the following ethics-related actions by management is least effective in encouraging acceptance by employees of an organization’s code of ethics?
A.
Management appoints an ethics officer to monitor and report to management on employee compliance.
Answer (A) is correct. This approach does not involve employees at all and projects the image that employees need to be supervised because they are likely to be unethical. This can discourage some employees.
B.
Management follows ethical principles in decisions made on behalf of the organization.
C.
Management keeps promises and commitments made to employees, customers, and vendors.
D.
Management supports employees in adhering to ethics standards.
Study Unit 1: Ethics for Management Accountants | Subunit 4: Fraud and the Fraud Risk Model (Fraud Triangle)
Question: 53
Misappropriation of assets is most often perpetrated by
A.
Employees.
Answer (A) is correct. Employees who have fiduciary responsibilities for assets are most likely to steal them.
B.
Customers.
C.
Management.
D.
Auditors.
Study Unit 1: Ethics for Management Accountants | Subunit 4: Fraud and the Fraud Risk Model (Fraud Triangle)
Question: 54
Inappropriate earnings management is typically considered one form of
A.
Embezzlement.
B.
Fraudulent financial reporting.
Answer (B) is correct. Inappropriate earnings management has been defined as the purposeful intercession in the financial reporting process.
C.
Theft of assets.
D.
Misappropriation of assets.
Study Unit 1: Ethics for Management Accountants | Subunit 4: Fraud and the Fraud Risk Model (Fraud Triangle)
Question: 55
When none of the three fraud risk factors are present, an accountant
A.
Can rule out the presence of fraud.
B.
Should consider the likelihood of fraud to be high.
C.
Should not rule out the presence of fraud completely.
Answer (C) is correct. Even when no factors are observed, an accountant cannot completely exclude the risk of fraud. Factors may be present but hidden from the accountant.
D.
Will likely search more diligently for fraud.
Study Unit 1: Ethics for Management Accountants | Subunit 4: Fraud and the Fraud Risk Model (Fraud Triangle)
Question: 56
Management is often able to rationalize the commission of fraud by
A.
Blaming it on others.
B.
Forcing staff to perpetrate it.
C.
Hiding it.
D.
Reasoning that it is in the best interests of the company.
Answer (D) is correct. Rationalization is a person’s ability to justify actions as consistent with his or her personal code of ethics. A person may rationalize a fraud by believing that the misdeed is to help the company to prosper or survive. Thus, a person may believe that the ends justify the means.
Study Unit 1: Ethics for Management Accountants | Subunit 4: Fraud and the Fraud Risk Model (Fraud Triangle)
Question: 57
High risk of employee fraud is most likely when there is pressure, rationalization, and
A.
Opportunity.
Answer (A) is correct. Opportunity creates risk of employee fraud.
B.
Internal control.
C.
Personal integrity.
D.
Limited responsibility.
Study Unit 1: Ethics for Management Accountants | Subunit 4: Fraud and the Fraud Risk Model (Fraud Triangle)
Question: 58
The fraud risk factor that may be mitigated by internal controls is
A.
Rationalization.
B.
Motive.
C.
Pressure.
D.
Opportunity.
Answer (D) is correct. The opportunity for individuals to perpetrate a fraud can be mitigated by proper controls. Examples are appropriate oversight, segregation of duties, and the audit process itself.
Study Unit 1: Ethics for Management Accountants | Subunit 4: Fraud and the Fraud Risk Model (Fraud Triangle)
Question: 59
In the fraud risk model, rationalization relates to
A.
The ability of a person not only to perpetrate but also to conceal fraud.
B.
The need for cash or other resources.
C.
A person’s ability to justify actions as consistent with his or her personal code of ethics.
Answer (C) is correct. Rationalization relates to the person’s ability to justify the commission of a fraud. Auditors should assume that anyone has this ability.
D.
The belief that the ends justify the means.
Study Unit 1: Ethics for Management Accountants | Subunit 4: Fraud and the Fraud Risk Model (Fraud Triangle)
Question: 60
The term or expression most associated with misappropriation of assets is
A.
Fraudulent financial statements.
B.
Stealing.
Answer (B) is correct. Misappropriation of assets results from theft, embezzlement, or an action that causes payment for items not received. It is an act of stealing by employees.
C.
Earnings management.
D.
Failure to file a tax return.
Study Unit 1: Ethics for Management Accountants | Subunit 4: Fraud and the Fraud Risk Model (Fraud Triangle)
Question: 61
Most employee fraud cases involve
A.
Misstatement of financial statements.
B.
Overstatement of revenues.
C.
Need for social acceptance.
D.
Financial or vice-related pressures.
Answer (D) is correct. Most employee fraud cases involve financial or vice-related pressures. Employees often have an actual or a perceived need for money.
Study Unit 1: Ethics for Management Accountants | Subunit 4: Fraud and the Fraud Risk Model (Fraud Triangle)
Question: 62
Who is responsible for minimizing the opportunity to misappropriate assets in an organization?
A.
The external auditor.
B.
Employees.
C.
Law enforcement.
D.
Management.
Answer (D) is correct. Management is expected to create systems and procedures to mitigate exposure to fraud and to cope with it effectively when discovered.
Study Unit 1: Ethics for Management Accountants | Subunit 4: Fraud and the Fraud Risk Model (Fraud Triangle)
Question: 3
Rationalization of a fraud by an employee may be in the form of all the following except
A.
Pressure from one’s spouse.
Answer (A) is correct. Pressure from one’s spouse is a motive for the misappropriation of assets.
B.
Feelings of being underpaid.
C.
Belief in being overworked.
D.
Belief that rank has its privileges.
Study Unit 1: Ethics for Management Accountants | Subunit 4: Fraud and the Fraud Risk Model (Fraud Triangle)
Question: 64
Which of the fraud risk factors related to employee fraud can be effectively controlled by the organization?
A.
Pressure.
B.
Motive.
C.
Rationalization.
D.
Opportunity.
Answer (D) is correct. Opportunity is mitigated by internal controls, which can be effectively designed and implemented by the organization.
Study Unit 1: Ethics for Management Accountants | Subunit 4: Fraud and the Fraud Risk Model (Fraud Triangle)
Question: 65
Fraudulent financial reporting is most often committed by
A.
Management to deceive financial statement users.
Answer (A) is correct. Fraudulent financial reporting is committed by management in an attempt to deceive financial statement users.
B.
An auditor while performing an audit.
C.
Employees stealing assets.
D.
Customers.
Study Unit 1: Ethics for Management Accountants | Subunit 4: Fraud and the Fraud Risk Model (Fraud Triangle)
Question: 66
Public record searches may be effective in certain instances. Which of the following is a limitation on public record searches?
A.
It is often very costly to search public records.
B.
Very few types of information are available.
C.
The information from public sources is most often incorrect.
D.
Availability of records may be limited.
Answer (D) is correct. Records for early periods may be limited or nonexistent.
Study Unit 1: Ethics for Management Accountants | Subunit 4: Fraud and the Fraud Risk Model (Fraud Triangle)
Question: 67
Based on the fraud risk model, which of the following is the most likely motive for employee fraud?
A.
Gambling losses.
Answer (A) is correct. An employee’s actual or perceived need for assets, for example, from gambling losses, can create the motivation to commit fraud.
B.
Ineffective internal controls.
C.
Belief that rank has its privileges.
D.
Ineffective supervision.
Study Unit 1: Ethics for Management Accountants | Subunit 4: Fraud and the Fraud Risk Model (Fraud Triangle)
Question: 68
The Public Company Accounting Oversight Board and the SEC would be most interested in an organization’s activities related to
A.
The misappropriation of assets.
B.
Failure to adequately compensate employees.
C.
Effectiveness of internal controls.
D.
Fraudulent financial reporting.
Answer (D) is correct. Fraudulent financial reporting is most often committed by management to deceive financial statement users. The PCAOB and the SEC would be most interested in these types of activities.
Study Unit 1: Ethics for Management Accountants | Subunit 4: Fraud and the Fraud Risk Model (Fraud Triangle)
Question: 69
Based on the fraud risk model, which of the following most likely is not an opportunity to commit employee fraud?
A.
Lack of transaction authorizations.
B.
Living beyond one’s means.
Answer (B) is correct. Living beyond one’s means is pressure or motivation to commit fraud.
C.
Poor accounting records.
D.
Lack of physical controls.
Study Unit 1: Ethics for Management Accountants | Subunit 4: Fraud and the Fraud Risk Model (Fraud Triangle)
Question: 70
Which of the following are red flags indicating misappropriation of assets?
I. Unexplained budget variances
II. Poor supervision
III. Failure of certain employees to take vacations
A.
I and II only.
B.
II and III only.
C.
I and III only.
D.
I, II, and III.
Answer (D) is correct. Among other things, unexplained budget variances, poor supervision, and failure of certain employees to take vacations are red flags indicating misappropriation of assets.
Study Unit 1: Ethics for Management Accountants | Subunit 4: Fraud and the Fraud Risk Model (Fraud Triangle)
Question: 71
An employee is stealing office supplies and believes that everybody else is doing it. The fraud risk factor represented by the employee is
A.
Opportunity.
B.
Ability.
C.
Rationalization.
Answer (C) is correct. Certain employee attitudes or rationalizations are often associated with acts of fraud, including believing everybody else is doing it.
D.
Motive.
Study Unit 1: Ethics for Management Accountants | Subunit 4: Fraud and the Fraud Risk Model (Fraud Triangle)
Question: 72
Ghosting employees is a form of fraud that
A.
Expects employees to work for nothing.
B.
Understates labor expense by failing to record payroll.
C.
Records payroll to another account in an attempt to understate payroll expense.
D.
Maintains employees on the payroll who no longer work for the organization.
Answer (D) is correct. Ghosting maintains employees on the payroll who no longer work for the organization.
Study Unit 1: Ethics for Management Accountants | Subunit 4: Fraud and the Fraud Risk Model (Fraud Triangle)
Question: 73
Which of the following is a motive for fraudulent financial reporting?
A.
Oversight of management is lacking.
B.
There is no internal auditing function.
C.
The board of directors includes a number of related parties.
D.
A manager's compensation is tied to reported financial results.
Answer (D) is correct. When a manager's compensation is tied to reported financial results, the motive for fraud is stronger.
Study Unit 1: Ethics for Management Accountants | Subunit 4: Fraud and the Fraud Risk Model (Fraud Triangle)
Question: 74
What is the most likely reason for management to overstate expenses?
A.
To minimize tax liability.
Answer (A) is correct. The most likely reason for management to understate income by overstating expenses is to minimize tax liability.
B.
To earn a bonus.
C.
To maximize net income.
D.
To maximize cash on hand.
Study Unit 1: Ethics for Management Accountants | Subunit 5: Addressing Fraud and Error
Question: 75
A proper segregation of duties requires that an individual
A.
Authorizing a transaction records it.
B.
Authorizing a transaction maintain custody of the asset that resulted from the transaction.
C.
Maintaining custody of an asset be entitled to access the accounting records for the asset.
D.
Recording a transaction not compare the accounting record of the asset with the asset itself.
Answer (D) is correct. One person should not be responsible for all phases of a transaction, i.e., for authorization, recording, and custodianship of the related assets. These duties should be performed by separate individuals to reduce the opportunities for any person to be in a position of both perpetrating and concealing errors or fraud in the normal course of his or her duties. For instance, an employee who receives and lists cash receipts should not be responsible for comparing the recorded accountability for cash with existing amounts.
Study Unit 1: Ethics for Management Accountants | Subunit 5: Addressing Fraud and Error
Question: 76
The frequency of the comparison of recorded accountability with assets (for the purpose of safeguarding assets) should be determined by
A.
The amount of assets independent of the cost of the comparison.
B.
The nature and amount of the asset and the cost of making the comparison.
Answer (B) is correct. Assets should be compared with the recorded accountability as frequently as the nature and amount of the assets require, within the limits of acceptable costs of comparison. The costs of safeguarding assets should not exceed the expected benefits.
C.
The cost of the comparison and whether the susceptibility to loss results from errors or fraud.
D.
The auditor in consultation with client management.
Study Unit 1: Ethics for Management Accountants | Subunit 5: Addressing Fraud and Error
Question: 77
An adequate system of internal controls is most likely to detect a fraud perpetrated by a
A.
Group of employees in collusion.
B.
Single employee.
Answer (B) is correct. Segregation of duties and other control procedures serve to prevent or detect a fraud committed by an employee acting alone. One employee may not have the ability to engage in wrongdoing or may be subject to detection by other employees in the course of performing their assigned duties. However, collusion may circumvent controls. For example, comparison of recorded accountability with assets may fail to detect fraud if persons having custody of assets collude with record keepers.
C.
Group of managers in collusion.
D.
Single manager.
Study Unit 1: Ethics for Management Accountants | Subunit 5: Addressing Fraud and Error
Question: 78
Internal control cannot be designed to provide reasonable assurance regarding the achievement of objectives concerning
A.
Reliability of financial reporting.
B.
Elimination of all fraud.
Answer (B) is correct. Internal control is a process designed to provide reasonable assurance regarding the achievement of organizational objectives. Because of inherent limitations, however, no system can be designed to eliminate all fraud.
C.
Compliance with applicable laws and regulations.
D.
Effectiveness and efficiency of operations.
Study Unit 1: Ethics for Management Accountants | Subunit 5: Addressing Fraud and Error
Question: 79
Internal controls may be preventive, detective, corrective, or directive. Which of the following is preventive?
A.
Requiring two persons to open mail.
Answer (A) is correct. Preventive controls are designed to prevent an error or an irregularity. Detective and corrective controls attempt to identify and correct errors or irregularities that have already occurred. Preventive controls are usually more cost beneficial than detective or corrective controls. Assigning two individuals to open mail is an attempt to prevent misstatement of cash receipts.
B.
Reconciling the accounts receivable subsidiary file with the control account.
C.
Using batch totals.
D.
Preparing bank reconciliations.
Study Unit 1: Ethics for Management Accountants | Subunit 5: Addressing Fraud and Error
Question: 80
Segregation of duties is a fundamental concept in an effective system of internal control. Nevertheless, the internal auditor must be aware that this safeguard can be compromised through
A.
Lack of training of employees.
B.
Collusion among employees.
Answer (B) is correct. By segregating duties, organizations make it more difficult for one person to perpetrate a fraud. When custody of the asset and recordkeeping for the asset are invested in different persons, a fraud generally cannot be executed by one of the two parties. However, if they collude, the internal control aspect of the segregation is nullified.
C.
Irregular employee reviews.
D.
Absence of internal auditing.
Study Unit 2: Ratio Analysis | Subunit 1: Liquidity Ratios -- Calculations
Fact Pattern:
Tosh Enterprises reported the following account information:
Accounts receivable
Accounts payable
Bonds payable, due in 10 years
Cash
Interest payable, due in 3 months
Inventory
Land
Short-term prepaid expense
Question: 1 The current ratio for Tosh Enterprises is
A.
B.
C.
D.
Answer (D) is correct.
The current ratio equals current assets divided by current liabilities. Current assets
consist of cash, accounts receivable, inventory, and prepaid expenses, a total of
$1,480,000 ($400,000 + $200,000 + $800,000 + $80,000). Current liabilities
consist of accounts payable and interest payable, a total of $280,000 ($260,000 +
$20,000). Hence, the current ratio is 5.29 ($1,480,000 ÷ $280,000).
Study Unit 2: Ratio Analysis | Subunit 1: Liquidity Ratios -- Calculations
Fact Pattern:
Tosh Enterprises reported the following account information:
Accounts receivable
Accounts payable
Bonds payable, due in 10 years
Cash
Interest payable, due in 3 months
Inventory
Land
Short-term prepaid expense
Question: 2 What is Tosh Enterprises’ quick (acid test) ratio?
A.
B.
C.
Answer (C) is correct.
The quick (acid test) ratio equals the quick assets divided by current liabilities. For
Tosh, quick assets consist of cash ($200,000) and accounts receivable ($400,000), a
total of $600,000. Current liabilities consist of accounts payable ($260,000) and
interest payable ($20,000) for a total of $280,000. Hence, the quick ratio is 2.14
($600,000 ÷ $280,000).
D.
Study Unit 2: Ratio Analysis | Subunit 1: Liquidity Ratios -- Calculations
Fact Pattern:
Tosh Enterprises reported the following account information:
Accounts receivable
Accounts payable
Bonds payable, due in 10 years
Cash
Interest payable, due in 3 months
Inventory
Land
Short-term prepaid expense
Question: 3 Tosh Enterprises’ amount of working capital is
A.
B.
C.
Answer (C) is correct.
Working capital equals current assets minus current liabilities. For Tosh
Enterprises, current assets consist of cash, accounts receivable, inventory, and
prepaid expenses, a total of $1,480,000 ($400,000 + $200,000 + $800,000 +
$80,000). Current liabilities consist of accounts payable and interest payable for a
total of $280,000 ($260,000 + $20,000). Accordingly, working capital is
$1,200,000 ($1,480,000 – )
D.
Study Unit 2: Ratio Analysis | Subunit 1: Liquidity Ratios -- Calculations
Question: 4 A financial analyst has obtained the following data from Kryton Industries’ financial
statements:
Cash
Marketable securities
Accounts receivable, net
Inventories, net
Prepaid expenses
Total current assets
Accounts payable
Income taxes
Accrued liabilities
Current portion of
long-term debt
Total current liabilities
In order to determine Kryton’s ability to pay current obligations the financial analyst would
calculate Kryton’s cash ratio as
A.
Answer (A) is correct.
The cash ratio, a more conservative measure of liquidity than the quick ratio, is
calculated as follows:
Cash ratio = (Cash + Marketable securities) ÷ Current liabilities
= ($200,000 + $100,000) ÷ $600,000
=
B.
C.
D.
Study Unit 2: Ratio Analysis | Subunit 1: Liquidity Ratios -- Calculations
Question: 5 Given an acid test ratio of 2.0, current assets of $5,000, and inventory of $2,000, the value of
current liabilities is
A.
Answer (A) is correct.
The acid test, or quick, ratio equals the quick assets (cash, marketable securities,
and accounts receivable) divided by current liabilities. Current assets equal the
quick assets plus inventory and prepaid expenses. (This question assumes that the
entity has no prepaid expenses.) Given current assets of $5,000, inventory of
$2,000, and no prepaid expenses, the quick assets must be $3,000. Because the acid
test ratio is 2.0, the quick assets are double the current liabilities. Current liabilities
therefore are equal to $1,500 ($3,000 quick assets ÷ 2.0).
B.
C.
D.
Study Unit 2: Ratio Analysis | Subunit 1: Liquidity Ratios -- Calculations
Fact Pattern: The selected data pertain to Tilghman Company at December 31:
Quick assets
Acid test ratio 2.6 to 1
Current ratio 3.5 to 1
Net sales for the year
Cost of sales for the year
Average total assets for the year
Question: 6 Tilghman Company’s current liabilities at December equal
A.
B.
Answer (B) is correct.
Current liabilities can be calculated using the following relationship:
Acid test ratio = Quick assets ÷ Current liabilities
= $208,000 ÷ Current liabilities
Current liabilities × 2.6 =
Current liabilities = $208,000 ÷ 2.6
=
C.
D.
Study Unit 2: Ratio Analysis | Subunit 1: Liquidity Ratios -- Calculations
Fact Pattern: The selected data pertain to Tilghman Company at December 31:
Quick assets
Acid test ratio 2.6 to 1
Current ratio 3.5 to 1
Net sales for the year
Cost of sales for the year
Average total assets for the year
Question: 7 Tilghman Company’s inventory balance at December 31 is
A.
Answer (A) is correct.
Ending inventory can be calculated using the following relationships:
Acid test ratio =Q uick assets ÷ Current liabilities
=$ 208,000 ÷ Current liabilities
Current liabilities × 2.6 =
Current liabilities = $208,000 ÷ 2.6
=
Current assets ÷ Current liabilities = Current ratio
Current assets ÷ $80,000 =
Current assets =
Assuming that Tilghman has no prepaid expenses, inventory is the only difference
between current assets and quick assets. Thus, the ending balance of inventory must
be $72,000 ($280,000 – )
B.
C.
D.
Study Unit 2: Ratio Analysis | Subunit 1: Liquidity Ratios -- Calculations
Question: 8 The following financial information applies to Sycamore Company:
Cash
Marketable securities
Accounts receivable
Inventories
Prepaid expenses
Accounts payable
Long-term debt -- current portion
Long-term debt
Sales
What is the acid test (or quick) ratio for Sycamore?
A.
Answer (A) is correct.
The acid test (quick) ratio equals the quick assets (cash, marketable securities, and
accounts receivable) divided by current liabilities Sycamore’s acid test ratio is thus
1.558 [($10,000 + $18,000 + $120,000) ÷ ($75,000 + $20,000)].
B.
C.
D.
Study Unit 2: Ratio Analysis | Subunit 1: Liquidity Ratios -- Calculations
Fact Pattern: The information below pertains to Devlin Company.
Statement of Financial Position as of May 31
(in thousands)
Year
Year
Assets
Current assets
Cash
Trading securities
Accounts receivable (net)
Inventory
Prepaid expenses
Total current assets
Investments, at equity
Property, plant, and equipment
(net)
Intangible assets (net)
Total assets
Liabilities
Current liabilities
Notes payable
Accounts payable
Accrued expenses
Income taxes payable
Total current liabilities
Long-term debt
Deferred taxes
Total liabilities
Equity
Preferred stock, 6%, $100 par
value,
cumulative
Common stock, $10 par value
Additional paid-in capital --
Income Statement for the year ended May
(in thousands)
Year
Year
Net sales
Costs and expenses
Costs of goods sold
Selling, general, and
administrative
Interest expense
Income before taxes
Income taxes
Net income
common stock
Retained earnings
Total equity
Total liabilities and equity
Question: 9 Devlin Company’s acid test ratio at May Year 2, was
A. 0.60 to 1.
B. 0.90 to 1.
C. 1.14 to 1.
Answer (C) is correct.
The acid test, or quick, ratio equals quick assets (cash, trading securities, and
accounts receivable) divided by current liabilities. Quick assets total $143 ($45 +
$30 + $68), so the acid test ratio is 1.14 ($143 ÷ $125 current liabilities).
D. 1.86 to 1.
Study Unit 2: Ratio Analysis | Subunit 1: Liquidity Ratios -- Calculations
Fact Pattern: The Statement of Financial Position for King Products Corporation for the
fiscal years ended June 30, Year 2, and June 30, Year 1, is presented below. Net sales and cost
of goods sold for the year ended June 30, Year 2, were $600,000 and $440,000, respectively.
King Products Corporation
Statement of Financial Position
(in thousands)
June 30
Year Year
Cash
Marketable securities (at market)
Accounts receivable (net)
Inventories (at lower of cost or market)
Prepaid items
Total current assets
Land (at cost)
Building (net)
Equipment (net)
Patents (net)
Goodwill (net)
Total long-term assets
Total assets
Notes payable
Accounts payable
Accrued interest
Total current liabilities
Notes payable, 10% due 12/31/Year 7
Bonds payable, 12% due 6/30/Year 10
Total long-term debt
Total liabilities
Preferred stock -- 5% cumulative, $100 par, nonparticipating,
authorized, issued and outstanding, 2,000 shares
Common stock -- $10 par, 40,000 shares authorized,
30,000 shares issued and outstanding
Additional paid-in capital -- common
Retained earnings
Total equity
Total liabilities & equity
Question: 10 King Products Corporation’s quick (acid test) ratio at June Year was
A.
B.
Answer (B) is correct.
The quick (acid test) ratio equals quick assets divided by current liabilities King’s
quick assets consist of cash, receivables, and marketable securities. Accordingly,
the quick ratio is 1.11 [($60 cash + $40 marketable securities + $90 accounts
receivable) ÷ $170 current liabilities].
C.
D.
Study Unit 2: Ratio Analysis | Subunit 1: Liquidity Ratios -- Calculations
Fact Pattern: The data presented below show actual figures for selected accounts of McKeon
Company for the fiscal year ended May 31, Year 1, and selected budget figures for the Year 2
fiscal year McKeon’s controller is in the process of reviewing the Year budget and
calculating some key ratios based on the budget. McKeon Company monitors yield or return
ratios using the average financial position of the company. (Round all calculations to three
decimal places if necessary.)
5/31/Year 2 5/31/Year 1
Current assets
Noncurrent assets
Current liabilities
Long-term debt
Common stock ($30 par value)
Retained earnings
Year 2
Operations
Sales*
Cost of goods sold
Interest expense
Income taxes (40% rate)
Dividends declared and paid in Year 2
Administrative expense
*All sales are credit sales.
Current Assets
5/31/Year 2 5/31/Year 1
Cash
Accounts receivable
Inventory
Prepaid expenses
Question: 11 McKeon Company’s current ratio for Year is
A.
B.
C.
D.
Answer (D) is correct.
The current ratio equals the ratio of current assets to current liabilities. Dividing
$210,000 of current assets at year end by $78,000 of current liabilities at year end
results in a current ratio of 2.692.
Study Unit 2: Ratio Analysis | Subunit 1: Liquidity Ratios -- Calculations
Question: 12 A service company’s working capital at the beginning of January of the current year was
$70,000. The following transactions occurred during January:
Performed services on account
Purchased supplies on account
Consumed supplies
Purchased office equipment for cash
Paid short-term bank loan
Paid salaries
Accrued salaries
What is the amount of working capital at the end of January?
A.
B.
Answer (B) is correct.
Working capital is the excess of total current assets (CA) over total current
liabilities (CL). Thus, working capital at the end of January equals $80,500
computed as follows:
Current Assets* Current Liabilities*
Beginning working capital
Performed services on account I N
Purchased supplies on account I I
Consumed supplies ( ) D N
Purchased office equipment ( ) D N
Paid short-term bank loan D D
Paid salaries ( ) D N
Accrued salaries ( ) N I
Working capital, end of
January
* N = no effect; I = increase; D =
decrease
C.
D.
Study Unit 2: Ratio Analysis | Subunit 1: Liquidity Ratios -- Calculations
Fact Pattern:
CPZ Enterprises had the following account information.
Accounts receivable
Accounts payable Bonds payable, due in 10 years Cash Interest payable, due in 3 months Inventory Land Notes payable, due in 6 months Prepaid expenses The company has an operating cycle of 5 months. Question: 13 The current ratio for CPZ Enterprises is A. B. C. D. Answer (D) is correct. The current ratio equals current assets divided by current liabilities. This company’s current assets consist of cash accounts receivable inventory and prepaid expenses, a total of $740,000 ($100,000 + $200,000 + $400,000 + $40,000). The current liabilities consist of accounts payable, interest payable, and notes payable, which total $140,000 ($80,000 + $10,000 + $50,000). Thus, the current ratio is 5.29 ($740,000 ÷ $140,000). Study Unit 2: Ratio Analysis | Subunit 1: Liquidity Ratios -- Calculations Fact Pattern: CPZ Enterprises had the following account information. Accounts receivable Accounts payable Bonds payable, due in 10 years Cash
Interest payable, due in 3 months Inventory Land Notes payable, due in 6 months Prepaid expenses The company has an operating cycle of 5 months. Question: 14 What is CPZ’s acid test (quick) ratio? A. B. C. Answer (C) is correct. The acid test, or quick, ratio equals quick assets divided by current liabilities. Quick assets consist of cash ($100,000) and accounts receivable ($200,000), for a total of $300,000. The current liabilities consist of accounts payable, interest payable, and notes payable, for a total of $140,000 ($80,000 + $10,000 + $50,000). Hence, the quick ratio is 2.14 ($300,000 ÷ $140,000). D. Study Unit 2: Ratio Analysis | Subunit 1: Liquidity Ratios -- Calculations Question: 15 Beatnik Company has a current ratio of 2.5 and a quick ratio of 2.0. If the firm experienced $2 million in cost of sales and sustains an inventory turnover of what are the firm’s current assets? A. B. C. D. Answer (D) is correct. The only major difference between the current ratio and the quick ratio is the inclusion of inventory in the numerator. If cost of sales is $2 million and inventory turns over 8 times per year, then average inventory is $250,000 ($2,000,000 ÷ 8). Since the only difference between the two ratios is inventory, then inventory must equal .5 (2.5 – 2.0) times current liabilities; therefore, current liabilities are $500,000. Thus, current assets divided by $500,000 equals 2.5. Therefore, current assets must equal $1,250,000 (2.5 × $500,00 )
Study Unit 2: Ratio Analysis | Subunit 1: Liquidity Ratios -- Calculations
Fact Pattern:
Lisa, Inc.
Statement of Financial Position
December 31, Year 2
(000s)
Assets Year 2 Year 1
Current assets:
Cash
Trading securities
Accounts receivable (net)
Inventories (at lower of cost or market)
Prepaid items
Total current assets
Long-term investments:
Securities (at cost)
Property, plant, & equipment:
Land (at cost)
Building (net)
Equipment (net)
Intangible assets
Patents (net)
Goodwill (net)
Total long-term assets
Total assets
Liabilities & Shareholders’ Equity
Current liabilities:
Notes payable
Accounts payable
Accrued interest
Total current liabilities
Long-term debt:
Notes payable 10% due 12/31/Year 9
Bonds payable 12% due 12/31/Year 8
Total long-term debt
Total liabilities
Shareholders’ equity:
Preferred -- 5% cumulative, $100 par,
non-participating, 1,000 shares
authorized, issued and outstanding
Common -- $10 par 20,000 shares
authorized, 15,000 issued and
outstanding shares
Additional paid-in capital -- common
Retained earnings
Total shareholders’ equity
Total liabilities & equity
Question: 16 Lisa Inc ’s acid test (quick) ratio at December Year was
A. :
Answer (A) is correct.
The acid test, or quick, ratio is calculated by dividing total quick assets by current
liabilities. Quick assets are those that can be quickly converted into cash. Besides
cash they include trading securities and accounts receivable Lisa’s quick assets
total $95,000 ($30,000 + $20,000 + $45,000). Dividing $95,000 by the $85,000 of
current liabilities results in a ratio of 1.1.
B. :
C. :
D. :
Study Unit 2: Ratio Analysis | Subunit 1: Liquidity Ratios -- Calculations
Fact Pattern: A company has a current ratio of 1.4, a quick, or acid test, ratio of 1.2, and the
following partial summary balance sheet:
Cash
Accounts receivable ___
Inventory ___
Fixed assets ___
Total assets
Current liabilities $___
Long-term liabilities
Stockholders’ equity
Total liabilities and equity $___
Question: 17 The company has an accounts receivable balance of
A.
Answer (A) is correct.
Total assets equal total liabilities and equity. Hence, if total assets equal $100, total
liabilities and equity must equal $100, and current liabilities must equal $30 ($100
– $40 – $30). Because the quick ratio equals the quick assets (cash + accounts
receivable) divided by current liabilities, the quick assets must equal $36 ($30 × 1.2
quick ratio), and the accounts receivable balance is $26 ($36 – $10 cash).
B.
C.
D.
Study Unit 2: Ratio Analysis | Subunit 1: Liquidity Ratios -- Calculations
Fact Pattern: A company has a current ratio of 1.4, a quick, or acid test, ratio of 1.2, and the
following partial summary balance sheet:
Cash
Accounts receivable ___
Inventory ___
Fixed assets ___
Total assets
Current liabilities $___
Long-term liabilities
Stockholders’ equity
Total liabilities and equity $___
Question: 18 The company has a fixed assets balance of
A.
B.
Answer (B) is correct.
Total assets (given as $100) equals the sum of cash (given as $10), accounts
receivable ($26 as calculated using the quick ratio), inventory, and fixed assets.
Inventory can be determined because it is included in current, but not quick, assets,
and the current and quick ratios are known. Current assets equal $42 (1.4 current
ratio × $30 current liabilities), and the quick assets equal $36 (1.2 quick ratio × $30
current liabilities). Thus, inventory, which is the only difference in this question
between current and quick assets, equals $6 ($42 – $36). Fixed assets must then
equal $58 ($100 total assets – $10 cash – $26 accounts receivable – $6 inventory).
C.
D.
Study Unit 2: Ratio Analysis | Subunit 1: Liquidity Ratios -- Calculations
Fact Pattern:
RST Corporation Comparative Income
Statements for the Years 5 and 6
Year 6 Year 5
Sales (all are credit)
Cost of goods sold
Gross profit
Selling and administrative expenses
Income before interest and income taxes
Interest expense
Income before income taxes
Income tax expense
Net income
RST Corporation
Comparative Balance Sheets
End of Years 5 and 6
Assets Year 6 Year 5
Current assets:
Cash
Short-term marketable investments
Accounts receivable (net)
Inventory
Total current assets
Noncurrent assets:
Long-term investments
Property, plant, and equipment
Intangibles
Total assets
Liabilities and Stockholders’ Equity
Current liabilities:
Accounts payable
Accrued payables
Total current liabilities
Long-term Liabilities:
0% Bonds payable, due in Year 12
Total liabilities
Stockholders’ equity:
Common stock, 2,400 shares, $10 par
Retained earnings Total stockholders’ equity Total liabilities and stockholders’ equity The market value of RST’s common stock at the end of Year Six was per share Question: 19 RST’s acid test (or quick) ratio at the end of Year is A. 2.40 to 1. B. 2.18 to 1. C. 2.00 to 1. Answer (C) is correct. Liquidity ratios measure the ability of a company to meet its short-term obligations. A commonly used liquidity ratio is the acid test or quick ratio, which equals the sum of the quick assets (net accounts receivable, short-term marketable securities, and cash) divided by current liabilities. This ratio at the end of Year Six is 2.0 [($5,000 + $3,000 + $16,000) ÷ $12,000]. D. 1.50 to 1. Study Unit 2: Ratio Analysis | Subunit 1: Liquidity Ratios -- Calculations Fact Pattern: Broomall Corporation has decided to include certain financial ratios in its year-end annual report to shareholders. Selected information relating to its most recent fiscal year is provided below. Cash Accounts receivable: – Beginning of year – End of year Prepaid expenses Inventory: – Beginning of year
– End of year
Available-for-sale securities:
– Historical cost
– Fair value at year end
Accounts payable
Notes payable (due in 90 days)
Bonds payable (due in 10 years)
Net credit sales for year
Cost of goods sold
Question: 20 Broomall’s working capital at year end is
A.
Answer (A) is correct.
Working capital consists of current assets net of current liabilities Broomall’s
working capital at year end can thus be calculated as follows:
Cash
Accounts receivable
Prepaid expenses
Inventory
Available-for-sale securities
Accounts payable ( )
Notes payable ( )
Working capital B. C. D. Study Unit 2: Ratio Analysis | Subunit 1: Liquidity Ratios -- Calculations Fact Pattern: Broomall Corporation has decided to include certain financial ratios in its year-end annual report to shareholders. Selected information relating to its most recent fiscal year is provided below. Cash Accounts receivable: – Beginning of year – End of year Prepaid expenses Inventory: – Beginning of year – End of year Available-for-sale securities: – Historical cost – Fair value at year end Accounts payable
Notes payable (due in 90 days)
Bonds payable (due in 10 years)
Net credit sales for year
Cost of goods sold
Question: 21 Broomall’s quick (acid test) ratio at year end is
A. 2.00 to 1.
B. 1.925 to 1.
C. 1.80 to 1.
D. 1.05 to 1.
Answer (D) is correct.
The quick (acid test) ratio consists of the quick assets (cash, marketable securities,
and accounts receivable) divided by current liabilities Broomall’s quick ratio at
year end is thus 1.05 [($10,000 + $12,000 + $20,000) ÷ ($15,000 + $25,000)].
Study Unit 2: Ratio Analysis | Subunit 1: Liquidity Ratios -- Calculations
Fact Pattern: Shown below are beginning and ending balances for certain of Grimaldi Inc ’s
accounts.
January 1 December 31
Cash
Marketable securities
Accounts receivable (net)
Inventory
Property, plant, and
equipment (net)
Accounts payable
Accrued liabilities
Deferred taxes (noncurrent)
7% long-term bonds payable
Grimaldi’s net income for the year was
Question: 22 Grimaldi’s acid test ratio or quick ratio at the end of the year is
A.
B.
C.
D.
Answer (D) is correct.
The acid test, or quick, ratio consists of the quick assets (cash, marketable
securities and accounts receivable) divided by current liabilities Grimaldi’s quick
ratio at year end is thus 1.516 [($62,000 + $35,000 + $47,000) ÷ ($84,000 +
)
Study Unit 2: Ratio Analysis | Subunit 1: Liquidity Ratios -- Calculations
Fact Pattern: Shown below are beginning and ending balances for certain of Grimaldi Inc ’s
accounts.
January 1 December 31
Cash
Marketable securities
Accounts receivable (net)
Inventory
Property, plant, and
equipment (net)
Accounts payable
Accrued liabilities
Deferred taxes (noncurrent)
7% long-term bonds payable
Grimaldi’s net income for the year was
Question: 23 Grimaldi’s current ratio at the end of the year is
A.
B.
C.
D.
Answer (D) is correct. The current ratio, also called the working capital ratio, is the ratio of current assets to current liabilities Grimaldi’s current assets consist of cash marketable securities, accounts receivable, and inventory ($62,000 + $35,000 + $47,000 + ) The company’s current liabilities consist of accounts payable and accrued liabilities ( + ) Grimaldi’s current ratio is thus 2.968 ($282,000 ÷ $95,000). Study Unit 2: Ratio Analysis | Subunit 1: Liquidity Ratios -- Calculations Question: 24 All of the following are included when calculating the acid test ratio except A. Six-month treasury bills. B. Prepaid insurance. Answer (B) is correct. The acid test (quick) ratio consists of the quick assets (cash, marketable securities, and net accounts receivable) divided by current liabilities. Prepaid insurance is an illiquid current asset and thus not appropriate to include in the numerator. C. Accounts receivable. D. 60-day certificates of deposit. Study Unit 2: Ratio Analysis | Subunit 1: Liquidity Ratios -- Calculations Question: 25 A company’s cash ratio will decrease if the company A. Purchases commercial paper. B. Purchases materials on account. Answer (B) is correct. The cash ratio can be expressed as cash and marketable securities divided by current liabilities. If the company purchases materials on account, it will cause current liabilities to increase (the denominator of the cash ratio) while having no effect on cash or marketable securities (the numerator of the cash ratio). Because the denominator is increasing while the numerator is staying constant, the ratio will decrease. C. Sells goods for cash at a selling price lower than cost. D. Receives cash by issuing a short-term note payable. Study Unit 2: Ratio Analysis | Subunit 1: Liquidity Ratios -- Calculations
Question: 26 In analyzing the short-term liquidity of a firm, many analysts prefer to use the quick (or acid
test) ratio rather than the current ratio. The primary reason for this preference is that the
A. Quick ratio excludes account receivables.
B. Current ratio includes marketable securities that may be mispriced.
C. Pro-forma cash flow statements focus on cash only.
D. Conversion of inventory into cash is less reliable.
Answer (D) is correct.
The quick (or acid test) ratio consists of the quick assets (cash, marketable
securities, and accounts receivable) divided by current liabilities. Inventory is not
included in the quick ratio because conversion of inventory into cash is less
reliable.
Study Unit 2: Ratio Analysis | Subunit 1: Liquidity Ratios -- Calculations
Question: 27 A firm has gathered financial statement data from three companies applying for credit as new
customers. The company extends credit to customers on the credit terms 2/10, net 30. Prior to
accepting the customers, a financial analyst with the firm performs a liquidity analysis.
Summary data is shown below:
Company F
Company G
Company H
Cash
Accounts receivable
Inventory
Current assets
Total assets
Current liabilities
Total liabilities
Total shareholders’ equity
When evaluating the liquidity of the three potential customers, which one of the following
conclusions is correct?
A. Company H has the highest current ratio but the lowest acid-test ratio, so the firm
should not accept Company H as a new customer.
B. Company G has a low debt to total assets ratio, so the firm should accept Company G
as a new customer.
C. Company F has a low current ratio, so the firm should not accept Company F as a
new customer.
Answer (C) is correct.
Low current and acid-test ratios indicate that a company may not be able to repay
its debts. Therefore, the firm would not want to accept it as a credit customer.
D. Company F has a high long-term debt to equity ratio, so the firm should accept
Company F as a new customer.
Study Unit 2: Ratio Analysis | Subunit 1: Liquidity Ratios -- Calculations
Question: 28 A corporation has $90 million in current assets. If the corporation has a current ratio of 1.2 and
a quick ratio of 0.9, what is net working capital?
A. $10 million.
B. $15 million.
Answer (B) is correct.
Net working capital is Current assets – Current liabilities. The current ratio is
Current assets ÷ Current liabilities. Current liabilities can be found as follows:
Current ratio = Current assets ÷ Current liabilities
$90 million ÷ Current liabilities
Current liabilities = $90 million ÷ 1.2
= $75 million
Therefore, net working capital is $15 million ($90 million – $75 million).
C. $81 million.
D. $108 million.
Study Unit 2: Ratio Analysis | Subunit 2: Liquidity Ratios -- Effects of Transactions
Question: 29 Bond Corporation has a current ratio of 2 to 1 and a quick ratio (acid test) of 1 to 1. A
transaction that would change Bond’s quick ratio but not its current ratio is the
A. Sale of inventory on account at cost.
Answer (A) is correct.
The quick (acid test) ratio equals the quick assets (cash, marketable securities, and
accounts receivable) divided by current liabilities. The current ratio is equal to
current assets divided by current liabilities. The sale of inventory (not a quick
current asset) on account increases accounts receivable (a quick asset), thereby
changing the quick ratio. The sale of inventory on account, however, replaces one
current asset with another, and the current ratio is unaffected.
B. Collection of accounts receivable.
C. Payment of accounts payable.
D. Purchase of a patent for cash.
Study Unit 2: Ratio Analysis | Subunit 2: Liquidity Ratios -- Effects of Transactions
Fact Pattern:
Tosh Enterprises reported the following account information:
Accounts receivable
Accounts payable
Bonds payable, due in 10 years
Cash
Interest payable, due in 3 months
Inventory
Land
Short-term prepaid expense
Question: 30 What will happen to the ratios below if Tosh Enterprises uses cash to pay 25% of the accounts
payable?
Current Ratio Quick Ratio
A. Increase Increase
Answer (A) is correct.
Tosh’s current ratio is ( current assets ÷ current
liabilities), and its quick ratio is 2.14 ($600,000 quick assets ÷ $280,000 current
liabilities). Using cash to pay 25% of the accounts payable decreases the numerator
and denominator by $65,000 ($260,000 accounts payable × 25%). The new current
ratio will be 6.58 ($1,415,000 ÷ $215,000), and the new quick ratio will be 2.49
($535,000 ÷ $215,000). If a ratio exceeds 1.0, equal decreases in the numerator and
denominator increase the ratio.
B. Decrease Decrease
C. Increase Decrease
D. Decrease Increase
Study Unit 2: Ratio Analysis | Subunit 2: Liquidity Ratios -- Effects of Transactions
Question: 31 Rice, Inc., uses the allowance method to account for uncollectible accounts. An account
receivable that was previously determined uncollectible and written off was collected during
May The effect of the collection on Rice’s current ratio and total working capital is
Current Ratio Working Capital
A. None None
Answer (A) is correct.
The entry to record this transaction is to debit receivables, credit the allowance,
debit cash, and credit receivables. The result is to increase both an asset (cash) and
a contra asset (allowance for bad debts). These appear in the current asset section of
the balance sheet. Thus, the collection changes neither the current ratio nor working
capital because the effects are offsetting. The credit for the journal entry is made to
the allowance account on the assumption that another account will become
uncollectible. The company had previously estimated its bad debts and established
an appropriate allowance. It then (presumably) wrote off the wrong account.
Accordingly, the journal entry reinstates a balance in the allowance account to
absorb future uncollectibles.
B. Increase Increase C. Decrease Decrease D. None Increase Study Unit 2: Ratio Analysis | Subunit 2: Liquidity Ratios -- Effects of Transactions Fact Pattern: Depoole Company is a manufacturer of industrial products that uses a calendar year for financial reporting purposes. Assume that total quick assets exceeded total current liabilities both before and after the transaction described. Further assume that Depoole has positive profits during the year and a credit balance throughout the year in its retained earnings account. Question: 32 Depoole’s payment of a trade account payable of will A. Increase the current ratio, but the quick ratio would not be affected. B. Increase the quick ratio, but the current ratio would not be affected. C. Increase both the current and quick ratios. Answer (C) is correct. Current assets consist of more assets than quick assets; thus, if quick assets exceed current liabilities, then current assets do also. It can also be concluded that both ratios are greater than 1. An equal reduction in the numerator and the denominator, such as a payment of a trade payable, will cause each ratio to increase. D. Decrease both the current and quick ratios. Study Unit 2: Ratio Analysis | Subunit 2: Liquidity Ratios -- Effects of Transactions Fact Pattern: Depoole Company is a manufacturer of industrial products that uses a calendar year for financial reporting purposes. Assume that total quick assets exceeded total current liabilities both before and after the transaction described. Further assume that Depoole has positive profits during the year and a credit balance throughout the year in its retained earnings account. Question: 33 Depoole’s purchase of raw materials for on open account will A. Increase the current ratio. B. Decrease the current ratio. Answer (B) is correct. The purchase increases both the numerator and denominator of the current ratio by adding inventory to the numerator and payables to the denominator. Because the ratio before the purchase was greater than 1, the ratio is decreased. C. Increase net working capital. D. Decrease net working capital. Study Unit 2: Ratio Analysis | Subunit 2: Liquidity Ratios -- Effects of Transactions Fact Pattern: Depoole Company is a manufacturer of industrial products that uses a calendar year for financial reporting purposes. Assume that total quick assets exceeded total current
liabilities both before and after the transaction described. Further assume that Depoole has positive profits during the year and a credit balance throughout the year in its retained earnings account. Question: 34 Depoole’s collection of a current accounts receivable of will A. Increase the current ratio. B. Decrease the current ratio and the quick ratio. C. Increase the quick ratio. D. Not affect the current or quick ratios. Answer (D) is correct. Collecting current accounts receivable has no effect on either the current ratio or the quick ratio because assets (both current and quick) are reduced for the collection of receivables and increased by the same amount for the receipt of cash. Current liabilities are unchanged by the transaction. Study Unit 2: Ratio Analysis | Subunit 2: Liquidity Ratios -- Effects of Transactions Fact Pattern: Depoole Company is a manufacturer of industrial products that uses a calendar year for financial reporting purposes. Assume that total quick assets exceeded total current liabilities both before and after the transaction described. Further assume that Depoole has positive profits during the year and a credit balance throughout the year in its retained earnings account. Question: 35 Obsolete inventory of $125,000 was written off by Depoole during the year. This transaction A. Decreased the quick ratio. B. Increased the quick ratio. C. Increased net working capital. D. Decreased the current ratio. Answer (D) is correct. Writing off obsolete inventory reduces current assets, but not quick assets (cash, marketable securities, and accounts receivable). Thus, the current ratio was reduced and the quick ratio was unaffected. Study Unit 2: Ratio Analysis | Subunit 2: Liquidity Ratios -- Effects of Transactions Fact Pattern: Depoole Company is a manufacturer of industrial products that uses a calendar year for financial reporting purposes. Assume that total quick assets exceeded total current liabilities both before and after the transaction described. Further assume that Depoole has positive profits during the year and a credit balance throughout the year in its retained earnings account. Question: 36 Depoole’s issuance of serial bonds in exchange for an office building with the first installment of the bonds due late this year, A. Decreases net working capital. B. Decreases the current ratio.
C. Decreases the quick ratio. D. Affects all of the answers as indicated. Answer (D) is correct. The first installment is a current liability; thus the amount of current liabilities increases with no corresponding increase in current assets. The effect is to decrease working capital, the current ratio, and the quick ratio. Study Unit 2: Ratio Analysis | Subunit 2: Liquidity Ratios -- Effects of Transactions Fact Pattern: Depoole Company is a manufacturer of industrial products that uses a calendar year for financial reporting purposes. Assume that total quick assets exceeded total current liabilities both before and after the transaction described. Further assume that Depoole has positive profits during the year and a credit balance throughout the year in its retained earnings account. Question: 37 Depoole’s early liquidation of a long-term note with cash affects the A. Current ratio to a greater degree than the quick ratio. B. Quick ratio to a greater degree than the current ratio. Answer (B) is correct. The numerators of the quick and current ratios are decreased when cash is expended. Early payment of a long-term liability has no effect on the denominator (current liabilities). Since the numerator of the quick ratio, which includes cash, net receivables, and marketable securities, is less than the numerator of the current ratio, which includes all current assets, the quick ratio is affected to a greater degree. C. Current and quick ratio to the same degree. D. Current ratio but not the quick ratio. Study Unit 2: Ratio Analysis | Subunit 2: Liquidity Ratios -- Effects of Transactions Question: 38 Windham Company has current assets of $400,000 and current liabilities of $500,000. Windham Company’s current ratio will be increased by A. The purchase of $100,000 of inventory on account. Answer (A) is correct. The current ratio equals current assets divided by current liabilities. An equal increase in both the numerator and denominator of a current ratio less than 1.0 causes the ratio to increase Windham Company’s current ratio is ( ÷ $500,000). The purchase of $100,000 of inventory on account would increase the current assets to $500,000 and the current liabilities to $600,000, resulting in a new current ratio of .833. B. The payment of $100,000 of accounts payable. C. The collection of $100,000 of accounts receivable.
D. Refinancing a $100,000 long-term loan with short-term debt.
Study Unit 2: Ratio Analysis | Subunit 2: Liquidity Ratios -- Effects of Transactions
Question: 39 Peters Company has a 2-to-1 current ratio. This ratio would increase to more than 2 to 1 if
A. A previously declared stock dividend were distributed.
B. The company wrote off an uncollectible receivable.
C. The company sold merchandise on open account that earned a normal gross margin.
Answer (C) is correct.
The current ratio equals current assets divided by current liabilities. Thus, an
increase in current assets or a decrease in current liabilities, by itself, increases the
current ratio. The sale of inventory at a profit increases current assets without
changing liabilities. Inventory decreases, and receivables increase by a greater
amount. Thus, total current assets and the current ratio increase.
D. The company purchased inventory on open account.
Study Unit 2: Ratio Analysis | Subunit 2: Liquidity Ratios -- Effects of Transactions
Question: 40 Merit, Inc., uses the direct write-off method to account for uncollectible accounts receivable. If
the company subsequently collects an account receivable that was written off in a prior
accounting period, the effect of the collection of the account receivable on Merit’s current ratio
and total working capital would be
Current Ratio Working Capital
A. None None
B. Increase Increase
Answer (B) is correct.
Because the company uses the direct write-off method, the original entry involved a
debit to a bad debt expense account (closed to retained earnings). The subsequent
collection required a debit to cash and a credit to bad debt expense or retained
earnings. Thus, only one current asset account was involved in the collection entry,
and current assets (cash) increased as a result. If current assets increase and no
change occurs in current liabilities, the current ratio and working capital both
increase.
C. Increase None
D. None Decrease
Study Unit 2: Ratio Analysis | Subunit 2: Liquidity Ratios -- Effects of Transactions
Fact Pattern: Excerpts from the statement of financial position for Landau Corporation as of
September 30 of the current year are presented as follows. Cash Accounts receivable (net) Inventories Total current assets Accounts payable Accrued liabilities Total current liabilities The board of directors of Landau Corporation met on October 4 of the current year and declared the regular quarterly cash dividend amounting to $750,000 ($.60 per share). The dividend is payable on October 25 of the current year to all shareholders of record as of October 12 of the current year. Assume that the only transactions to affect Landau Corporation during October of the current year are the dividend transactions and that the closing entries have been made. Question: 41 Landau Corporation’s working capital was A. Unchanged by the dividend declaration and decreased by the dividend payment. B. Decreased by the dividend declaration and increased by the dividend payment. C. Unchanged by either the dividend declaration or the dividend payment. D. Decreased by the dividend declaration and unchanged by the dividend payment. Answer (D) is correct. Working capital is the excess of current assets over current liabilities. The declaration of a dividend requires a debit to retained earnings and a credit to dividends payable (a current liability). Thus, working capital is decreased by the amount of the increased current liability. The subsequent payment of the dividend has no effect on working capital because current assets (cash) and current liabilities (dividends payable) are both decreased by the same amount. Study Unit 2: Ratio Analysis | Subunit 2: Liquidity Ratios -- Effects of Transactions Fact Pattern: Excerpts from the statement of financial position for Landau Corporation as of September 30 of the current year are presented as follows. Cash Accounts receivable (net) Inventories Total current assets Accounts payable
Accrued liabilities
Total current liabilities
The board of directors of Landau Corporation met on October 4 of the current year and
declared the regular quarterly cash dividend amounting to $750,000 ($.60 per share). The
dividend is payable on October 25 of the current year to all shareholders of record as of
October 12 of the current year. Assume that the only transactions to affect Landau Corporation
during October of the current year are the dividend transactions and that the closing entries
have been made.
Question: 42 Landau Corporation’s current ratio was
A. Decreased by the dividend declaration and increased by the dividend payment.
Answer (A) is correct.
The dividend declaration decreased retained earnings and increased current
liabilities by $750,000. The subsequent payment decreased both current assets and
current liabilities by $750,000. Before the dividend declaration, the current ratio
was 3.03 (5,431,000 ÷ $1,789,000). The declaration increased current liabilities to
$2,539,000, and the new current ratio was 2.14 ($5,431,000 ÷ $2,539,000). The
payment reduced current assets to $4,681,000 and current liabilities to $1,789,000.
Thus, after the payment, the current ratio was 2.61 ($4,681,000 ÷ $1,789,000).
B. Unchanged by either the dividend declaration or the dividend payment.
C. Decreased by the dividend declaration and unchanged by the dividend payment.
D. Increased by the dividend declaration and unchanged by the dividend payment.
Study Unit 2: Ratio Analysis | Subunit 2: Liquidity Ratios -- Effects of Transactions
Fact Pattern: Excerpts from the statement of financial position for Markham Corporation as
of April 30 of the current year are presented as follows:
Cash
Accounts receivable (net)
Inventories
Total current assets
Accounts payable
Accrued liabilities
Total current liabilities
The board of directors of Markham met on May 5 of the current year and declared a quarterly
cash dividend in the amount of $800,000 ($.50 per share). The dividend was paid on May 28
of the current year to shareholders of record as of May 15 of the current year. Assume that the
only transactions that affected Markham during May of the current year were the dividend
transactions and that the closing entries have been made.
Question: 43 Markham’s working capital would be
A. Decreased by the dividend declaration and increased by the dividend payment.
B. Unchanged by either the dividend declaration or the dividend payment.
C. Decreased by the dividend declaration and unchanged by the dividend payment.
Answer (C) is correct.
Working capital is the excess of current assets over current liabilities. The
declaration of a dividend reduces retained earnings and creates a new current
liability. Thus, the declaration of a dividend reduces working capital because
current liabilities are increased without a corresponding increase in current assets.
The subsequent payment of the dividend has no effect on working capital because
current assets (cash) will be reduced by the same amount that current liabilities
(dividends payable) are reduced.
D. Increased by the dividend declaration and unchanged by the dividend payment.
Study Unit 2: Ratio Analysis | Subunit 2: Liquidity Ratios -- Effects of Transactions
Fact Pattern: Excerpts from the statement of financial position for Markham Corporation as
of April 30 of the current year are presented as follows:
Cash
Accounts receivable (net)
Inventories
Total current assets
Accounts payable
Accrued liabilities
Total current liabilities
The board of directors of Markham met on May 5 of the current year and declared a quarterly
cash dividend in the amount of $800,000 ($.50 per share). The dividend was paid on May 28
of the current year to shareholders of record as of May 15 of the current year. Assume that the
only transactions that affected Markham during May of the current year were the dividend
transactions and that the closing entries have been made.
Question: 44 Markham’s current ratio would be
A. Decreased by the dividend declaration and increased by the dividend payment.
Answer (A) is correct.
The current ratio equals current assets divided by current liabilities. The declaration
of a dividend results in a decrease in retained earnings and an increase in current
liabilities. The effect is to decrease the current ratio because current liabilities are
increased at the time of the declaration without a change in current assets. The
April 30 current ratio is 2.57 ($5,310,000 ÷ $2,067,000). Following the declaration
of an $800,000 dividend, the current ratio is 1.85 ($5,310,000 ÷ $2,867,000). The
subsequent payment of the dividend will increase the ratio because the ratio is
greater than 1.0 and both current assets and current liabilities will decline by the
same amount. The current ratio after the payment is 2.18 ($4,510,000 ÷
)
B. Increased by the dividend declaration and unchanged by the dividend payment.
C. Unchanged by either the dividend declaration or the dividend payment.
D. Unchanged by the dividend declaration and decreased by the dividend payment.
Study Unit 2: Ratio Analysis | Subunit 2: Liquidity Ratios -- Effects of Transactions
Question: 45 The following transactions occurred during a company’s first year of operations:
I. Purchased a delivery van for cash
II. Borrowed money by issuance of short-term debt
III. Purchased treasury stock
Which of the items above caused a change in the amount of working capital?
A. I only.
B. I and II only.
C. II and III only.
D. I and III only.
Answer (D) is correct.
Working capital is computed by deducting total current liabilities from total current
assets. The purchase of a delivery van for cash reduces current assets and has no
effect on current liabilities. The borrowing of cash by incurring short-term debt
increases current assets by the same amount as it increases current liabilities; hence,
it will have no effect on working capital. The purchase of treasury stock decreases
current assets but has no effect on current liabilities. Thus, the purchases of the van
and treasury stock affect working capital.
Study Unit 2: Ratio Analysis | Subunit 2: Liquidity Ratios -- Effects of Transactions
Fact Pattern:
CPZ Enterprises had the following account information.
Accounts receivable
Accounts payable
Bonds payable, due in 10 years
Cash
Interest payable, due in 3 months
Inventory
Land
Notes payable, due in 6 months
Prepaid expenses
The company has an operating cycle of 5 months.
Question: 46 What will happen to the ratios below if CPZ Enterprises uses cash to pay 50% of the accounts
payable?
Current Ratio Quick Ratio
A. Increase Increase
Answer (A) is correct.
Using cash to pay accounts payable will affect both ratios in a positive way. For
instance, before the payment, current assets totaled $740,000 and current liabilities
were $140,000, yielding a current ratio of 5.29. Paying $40,000 of the accounts
payable ($80,000 × 50%) would reduce current assets to $700,000 and current
liabilities to $100,000, for a new current ratio of 7.00. The quick assets would
decline from $300,000 to $260,000, and the current liabilities from $140,000 to
$100,000, for a new quick ratio of 2.60, an increase over the old ratio of 2.14.
B. Decrease Decrease
C. Increase Decrease
D. Decrease Increase
Study Unit 2: Ratio Analysis | Subunit 2: Liquidity Ratios -- Effects of Transactions
Fact Pattern: Jensen Corporation’s board of directors met on June and declared a regular
quarterly cash dividend of $.40 per share for a total value of $200,000. The dividend is payable
on June 24 to all stockholders of record as of June 17. Excerpts from the statement of financial
position for Jensen Corporation as of May 31 are presented as follows.
Cash
Accounts receivable (net)
Inventories
Total current assets
Total current liabilities
Assume that the only transactions to affect Jensen Corporation during June are the dividend
transactions.
Question: 47 Jensen’s working capital would be
A. Unchanged by the dividend declaration and decreased by the dividend payment.
B. Decreased by the dividend declaration and increased by the dividend payment.
C. Unchanged by either the dividend declaration or the dividend payment.
D. Decreased by the dividend declaration and unchanged by the dividend payment.
Answer (D) is correct.
Working capital is defined as current assets minus current liabilities. The
declaration of a dividend increases current liabilities and thus reduces working
capital. The subsequent payment has no effect on working capital since current
assets and current liabilities decrease by the same amount.
Study Unit 2: Ratio Analysis | Subunit 2: Liquidity Ratios -- Effects of Transactions
Fact Pattern: Jensen Corporation’s board of directors met on June and declared a regular
quarterly cash dividend of $.40 per share for a total value of $200,000. The dividend is payable
on June 24 to all stockholders of record as of June 17. Excerpts from the statement of financial
position for Jensen Corporation as of May 31 are presented as follows.
Cash
Accounts receivable (net)
Inventories
Total current assets
Total current liabilities
Assume that the only transactions to affect Jensen Corporation during June are the dividend
transactions.
Question: 48 Jensen’s current ratio would be
A. Unchanged by the dividend declaration and decreased by the dividend payment.
B. Decreased by the dividend declaration and increased by the dividend payment.
Answer (B) is correct.
The current ratio is computed by dividing current assets by current liabilities. The
declaration of a dividend increases liabilities (to $1,200,000) and thus decreases the
ratio. After the declaration, the ratio is 2.0 ($2,400,000 ÷ $1,200,000). It was
originally 2.4 ($2,400,000 ÷ $1,000,000). The subsequent payment decreases both
current assets and current liabilities by $200,000. The ratio is then 2.2 ($2,200,000
÷ $1,000,000). Therefore, the payment of the current liability increases the current
ratio.
C. Unchanged by either the dividend declaration or the dividend payment.
D. Decreased by the dividend declaration and unchanged by the dividend payment.
Study Unit 2: Ratio Analysis | Subunit 2: Liquidity Ratios -- Effects of Transactions
Fact Pattern: Jensen Corporation’s board of directors met on June and declared a regular
quarterly cash dividend of $.40 per share for a total value of $200,000. The dividend is payable
on June 24 to all stockholders of record as of June 17. Excerpts from the statement of financial
position for Jensen Corporation as of May 31 are presented as follows.
Cash
Accounts receivable (net)
Inventories
Total current assets
Total current liabilities
Assume that the only transactions to affect Jensen Corporation during June are the dividend
transactions.
Question: 49 Jensen’s quick (acid test) ratio would be
A. Unchanged by the dividend declaration and decreased by the dividend payment.
B. Decreased by the dividend declaration and increased by the dividend payment.
C. Unchanged by either the dividend declaration or the dividend payment.
D. Decreased by the dividend declaration and unchanged by the dividend payment.
Answer (D) is correct.
The quick ratio is computed by dividing quick assets (cash, marketable securities,
and receivables) by current liabilities. The dividend declaration will increase
current liabilities to $1,200,000 and decrease the quick ratio to 1.0 ($1,200,000
quick assets ÷ $1,200,000 current liabilities). The payment of a cash dividend will
decrease quick assets (cash) and the current liabilities by the same amount.
Accordingly, the ratio (1.0) remains unchanged by the payment.
Study Unit 2: Ratio Analysis | Subunit 2: Liquidity Ratios -- Effects of Transactions
Question: 50 All of the following are affected when merchandise is purchased on credit except
A. Total current assets.
B. Net working capital.
Answer (B) is correct.
Working capital is the net of current assets and current liabilities. When
merchandise is purchased on credit, inventory (a current asset) is increased, and
accounts payable (a current liability) is increased by the same amount. Thus, no net
change in working capital results.
C. Total current liabilities.
D. Current ratio.
Study Unit 2: Ratio Analysis | Subunit 2: Liquidity Ratios -- Effects of Transactions
Question: 51 Birch Products, Inc., has the following current assets:
Cash
Marketable securities
Accounts receivable
Inventories
If Birch’s current liabilities are the firm’s
A. Current ratio will decrease if a payment of $100,000 cash is used to pay $100,000 of
accounts payable.
B. Current ratio will not change if a payment of $100,000 cash is used to pay $100,000
of accounts payable.
C. Quick ratio will decrease if a payment of $100,000 cash is used to purchase
inventory.
Answer (C) is correct.
The only difference between the current ratio and the quick ratio is the removal of
inventories from the numerator of the quick ratio. Thus, a shift from cash to
inventory will have a reducing effect on the quick ratio that it does not have on the
current ratio.
D. Quick ratio will not change if a payment of $100,000 cash is used to purchase
inventory.
Study Unit 2: Ratio Analysis | Subunit 2: Liquidity Ratios -- Effects of Transactions
Question: 52 Davis Retail, Inc., has total assets of $7,500,000 and a current ratio of 2.3 times before
purchasing $750,000 of merchandise on credit for resale. After this purchase, the current ratio
will
A. Remain at 2.3 times.
B. Be higher than 2.3 times.
C. Be lower than 2.3 times.
Answer (C) is correct.
The current ratio is the ratio of current assets to current liabilities. When the ratio is
greater than one, any change of equal dollar amount on both the numerator and
denominator will result in a lowering of the overall ratio (since the denominator
will increase by a proportionally greater amount). The purchase of merchandise on
credit is an example of such a change: Inventory increases in the numerator and
accounts payable increases in the denominator by an equal dollar amount.
D. Be exactly 2.53 times.
Study Unit 2: Ratio Analysis | Subunit 2: Liquidity Ratios -- Effects of Transactions
Question: 53 Markowitz Company increased its allowance for uncollectible accounts. This adjustment will
A. Increase the acid test ratio.
B. Increase working capital.
C. Reduce debt-to-asset ratio.
D. Reduce the current ratio.
Answer (D) is correct.
The current ratio is the ratio of current assets to current liabilities. By reducing net
receivables, an increase in the allowance for uncollectible accounts lowers the
amount of current assets (the numerator), reducing the overall ratio.
Study Unit 2: Ratio Analysis | Subunit 2: Liquidity Ratios -- Effects of Transactions
Question: 54 Garstka Auto Parts must increase its acid test ratio above the current 0.9 level in order to
comply with the terms of a loan agreement. Which one of the following actions is most likely
to produce the desired results?
A. Expediting collection of accounts receivable.
B. Selling auto parts on account.
Answer (B) is correct.
The acid test (quick) ratio consists of the quick assets (cash, marketable securities,
and net accounts receivable) divided by current liabilities. Exchanging merchandise
inventory for accounts receivable increases the numerator while having no effect on
the denominator, resulting in an increase in the overall ratio.
C. Making a payment to trade accounts payable.
D. Purchasing marketable securities for cash.
Study Unit 2: Ratio Analysis | Subunit 2: Liquidity Ratios -- Effects of Transactions
Question: 55 The owner of a chain of grocery stores has bought a large supply of mangoes and paid for the
fruit with cash. This purchase will adversely impact which one of the following?
A. Working capital.
B. Current ratio.
C. Quick or acid test ratio.
Answer (C) is correct.
The quick (acid test) ratio consists of the quick assets (cash, marketable securities,
and net accounts receivable) divided by current liabilities. Buying merchandise
inventory with cash reduces the numerator, lowering the overall ratio.
D. Price earnings ratio.
Study Unit 2: Ratio Analysis | Subunit 2: Liquidity Ratios -- Effects of Transactions
Question: 56 Both the current ratio and the quick ratio for Spartan Corporation have been slowly decreasing.
For the past two years, the current ratio has been 2.3-to-1 and 2.0-to-1. During the same time
period, the quick ratio has decreased from 1.2-to-1 to 1.0-to-1. The disparity between the
current and quick ratios can be explained by which one of the following?
A. The current portion of long-term debt has been steadily increasing.
B. The cash balance is unusually low.
C. The accounts receivable balance has decreased.
D. The inventory balance is unusually high.
Answer (D) is correct.
The only difference between the current ratio and the quick ratio is the removal of
inventories from the numerator of the quick ratio. Thus, a high inventory balance
can account for the disparity between the current and quick ratios.
Study Unit 2: Ratio Analysis | Subunit 2: Liquidity Ratios -- Effects of Transactions
Question: 57 The acid test ratio shows the ability of a company to pay its current liabilities without having
to
A. Reduce its cash balance.
B. Borrow additional funds.
C. Collect its receivables.
D. Liquidate its inventory.
Answer (D) is correct.
The acid test (quick) ratio consists of the quick assets (cash, marketable securities,
and net accounts receivable) divided by current liabilities. Thus, the numerator
consists of those assets that are more liquid than inventory.
Study Unit 2: Ratio Analysis | Subunit 2: Liquidity Ratios -- Effects of Transactions
Question: 58 When a fixed asset is sold for less than book value, which one of the following will decrease?
A. Total current assets.
B. Current ratio.
C. Net profit.
Answer (C) is correct.
When an asset is sold for less than book value, an accrual-basis loss is incurred.
This reduces net profit.
D. Net working capital.
Study Unit 2: Ratio Analysis | Subunit 2: Liquidity Ratios -- Effects of Transactions
Question: 59 If a company has a current ratio of 2.1 and pays off a portion of its accounts payable with cash,
the current ratio will
A. Decrease.
B. Increase.
Answer (B) is correct.
The current ratio is the ratio of current assets to current liabilities. Since the
numerator before the transaction was greater than the denominator, a reduction to
both factors of an equal dollar amount will have a proportionally greater effect on
the denominator, causing the ratio as a whole to increase.
C. Remain unchanged.
D. Move closer to the quick ratio.
Study Unit 2: Ratio Analysis | Subunit 2: Liquidity Ratios -- Effects of Transactions
Question: 60 Clauson, Inc., grants credit terms of 1/15, net 30 and projects gross sales for next year of
$2,000,000. The credit manager estimates that 40% of their customers pay on the discount
date, 40% on the net due date, and 20% pay 15 days after the net due date. Assuming uniform
sales and a 360-day year what is the projected days’ sales outstanding (rounded to the nearest
whole day)?
A. 20 days.
B. 24 days.
C. 27 days.
Answer (C) is correct.
The days’ sales outstanding can be determined by weighting the collection period
for each group of receivables by its collection percentage. Hence, the projected
days’ sales outstanding equal days [( days × 40%) + (30 days × 40%) +
(45 days × 20%)].
D. 30 days.
Study Unit 2: Ratio Analysis | Subunit 2: Liquidity Ratios -- Effects of Transactions
Question: 61 A condensed comparative balance sheet for a company appears below:
12/31/Year 1 12/31/Year 2
Cash
Accounts receivable
Inventory
Property, plant, & equipment
Accumulated depreciation ( ) ( )
Total assets
Current liabilities
Long-term liabilities
Stockholders’ equity
Total liabilities and equity
In looking at liquidity ratios at both balance sheet dates, what happened to the (1) current ratio
and (2) acid test (quick) ratio?
( ) ( )
Current Ratio Acid Test Ratio
A. Increased Increased
B. Increased Decreased
C. Decreased Increased
D. Decreased Decreased
Answer (D) is correct.
The current ratio is determined by dividing current assets by current liabilities. The
acid test ratio is determined by dividing quick assets by current liabilities. At
December 31, Year 1, the current ratio is 6 to 1 [($40,000 + $120,000 + $200,000)
÷ $60,000]. At December 31, Year 2, the current ratio is 4.3 to 1 [($30,000 +
$100,000 + $300,000) ÷ $100,000]. Hence, there was a decrease in the current
ratio. At December 31, Year 1, the acid test ratio is 2.667 to 1 [($40,000 +
$120,000) ÷ $60,000]. At December 31, Year 2, the acid test ratio is 1.3 to 1
[($30,000 + $100,000) ÷ $100,000]. Thus, the acid test ratio also declined.
Study Unit 2: Ratio Analysis | Subunit 2: Liquidity Ratios -- Effects of Transactions
Question: 62 Birch Products Ltd. has the following current assets.
Cash €
Marketable securities
Accounts receivable
Inventories
Total current assets €
If Birch’s current liabilities are € the firm’s
A. Current ratio will decrease if a payment of € cash is used to pay € of
accounts payable.
B. Current ratio will not change if a payment of € is used to pay € of
accounts payable.
C. Acid-test (quick) ratio will decrease if a payment of € cash is used to
purchase inventory.
Answer (C) is correct.
The only difference between the current ratio and the quick ratio is the removal of
inventories from the numerator of the quick ratio. Thus, a shift from cash to
inventory will have a reducing effect on the quick ratio but not on the current ratio.
D. Acid-test (quick) ratio will not change if a payment of € cash is used to
purchase inventory.
Study Unit 2: Ratio Analysis | Subunit 2: Liquidity Ratios -- Effects of Transactions
Question: 63 Alliance Ltd. has $80 million in current assets, comprised of $30 million in inventory and $50
million in cash and marketable securities The company’s current liabilities total million
If Alliance purchases an additional $10 million in inventory with $10 million in cash, the
effect of this transaction on the company would be to
A. Decrease the current ratio and increase the quick ratio.
B. Decrease the quick ratio while the current ratio remains unchanged.
Answer (B) is correct.
The quick ratio decreases from 1.00 ($50 million ÷ $50 million) to .80 ($40 million
÷ $50 million) due to the $10 million decrease in cash. The current ratio remains
unchanged because the $10 million decrease in cash is offset by the $10 million
increase in inventory.
C. Leave both the current ratio and the quick ratio unchanged.
D. Decrease the current ratio and decrease the quick ratio.
Study Unit 2: Ratio Analysis | Subunit 3: Activity Measures
Question: 64 Carson Corporation computed the following items from its financial records for the current
year:
Current ratio 2 to 1
Inventory turnover 54 days
Accounts receivable turnover 24 days
Current liabilities turnover 36 days
The number of days in Carson’s operating cycle for the current year was
A.
B.
C.
Answer (C) is correct.
The operating cycle is the time needed to turn cash into inventory, inventory into
receivables, and receivables back into cash. It is equal to the sum of the number of
days’ sales in inventory and the number of days’ sales in receivables. The number
of Carson’s days’ sales in inventory is given as days The number of days’ sales
in receivables is given as 24. Therefore, the number of days in the operating cycle
is 78 (54 + 24).
D.
Study Unit 2: Ratio Analysis | Subunit 3: Activity Measures
Question: 65 To determine the operating cycle for a retail department store, which one of the following pairs
of items is needed?
A. Days’ sales in accounts receivable and average merchandise inventory
B. Cash turnover and net sales.
C. Accounts receivable turnover and inventory turnover.
Answer (C) is correct.
The operating cycle is the time needed to turn cash into inventory, inventory into
receivables, and receivables back into cash. For a retailer, it is the time from
purchase of inventory to collection of cash. Thus, the operating cycle of a retailer is
equal to the sum of the number of days’ sales in inventory and the number of days’
sales in receivables. Inventory turnover equals cost of goods sold divided by
average inventory The days’ sales in inventory equals (or another period
chosen by the analyst) divided by the inventory turnover. Accounts receivable
turnover equals net credit sales divided by average receivables The days’ sales in
receivables equals 365 (or other number) divided by the accounts receivable
turnover.
D. Asset turnover and return on sales.
Study Unit 2: Ratio Analysis | Subunit 3: Activity Measures
Question: 66 Accounts receivable turnover ratio will normally decrease as a result of
A. The write-off of an uncollectible account (assume the use of the allowance for
doubtful accounts method).
B. A significant sales volume decrease near the end of the accounting period.
C. An increase in cash sales in proportion to credit sales.
D. A change in credit policy to lengthen the period for cash discounts.
Answer (D) is correct.
The accounts receivable turnover ratio equals net credit sales divided by average
receivables. Hence, it will decrease if a company lengthens the credit period or the
discount period because the denominator will increase as receivables are held for
longer times.
Study Unit 2: Ratio Analysis | Subunit 3: Activity Measures
Question: 67 Which one of the following inventory cost flow assumptions will result in a higher inventory
turnover ratio in an inflationary economy?
A. FIFO.
B. LIFO.
Answer (B) is correct.
The inventory turnover ratio equals the cost of goods sold divided by the average
inventory. LIFO assumes that the last goods purchased are the first goods sold and
that the oldest goods purchased remain in inventory. The result is a higher cost of
goods sold and a lower average inventory than under other inventory cost flow
assumptions if prices are rising. Because cost of goods sold (the numerator) will be
higher and average inventory (the denominator) will be lower than under other
inventory cost flow assumptions, LIFO produces the highest inventory turnover
ratio.
C. Weighted average.
D. Specific identification.
Study Unit 2: Ratio Analysis | Subunit 3: Activity Measures
Question: 68 The days’ sales in receivables ratio will be understated if the company
A. Uses a natural business year for its accounting period.
Answer (A) is correct.
The days’ sales in receivables ratio equals the days in the year divided by the
receivables turnover ratio (sales ÷ average receivables) Days’ sales may also be
computed based only on ending receivables. In either case, use of the natural
business year tends to understate the ratio because receivables will usually be at a
low point at the beginning and end of the natural year. For example, a ski resort
may close its books on May 31, a low point in its operating cycle.
B. Uses a calendar year for its accounting period.
C. Uses average receivables in the ratio calculation.
D. Does not use average receivables in the ratio calculation.
Study Unit 2: Ratio Analysis | Subunit 3: Activity Measures
Question: 69 When a balance sheet amount is related to an income statement amount in computing a ratio,
A. The balance sheet amount should be converted to an average for the year.
Answer (A) is correct.
In ratios that relate the income statement to the balance sheet (e.g., inventory
turnover, asset turnover, receivables turnover, and return on assets), the balance
sheet figure should be an average. The reason is that the income statement amounts
represent activity over a period of time. Thus, the balance sheet figure should be
adjusted to reflect assets available for use throughout the period.
B. The income statement amount should be converted to an average for the year.
C. Both amounts should be converted to market value.
D. Comparisons with industry ratios are not meaningful.
Study Unit 2: Ratio Analysis | Subunit 3: Activity Measures
Fact Pattern: The information below pertains to Devlin Company.
Statement of Financial Position as of May 31
(in thousands)
Year
Year
Assets
Current assets
Cash
Trading securities
Accounts receivable (net)
Inventory
Income Statement for the year ended May
(in thousands)
Year
Year
Net sales
Costs and expenses
Costs of goods sold
Selling, general, and
administrative
Prepaid expenses
Total current assets
Investments, at equity
Property, plant, and equipment
(net)
Intangible assets (net)
Total assets
Liabilities
Current liabilities
Notes payable
Accounts payable
Accrued expenses
Income taxes payable
Total current liabilities
Long-term debt
Deferred taxes
Total liabilities
Equity
Preferred stock, 6%, $100 par
value,
cumulative
Common stock, $10 par value
Additional paid-in capital --
common stock
Retained earnings
Total equity
Total liabilities and equity
Interest expense
Income before taxes
Income taxes
Net income
Question: 70 Devlin Company’s inventory turnover for the year ended May Year 2, was
A. 3.67 times.
B. 3.88 times.
Answer (B) is correct.
Inventory turnover equals cost of goods sold divided by average inventory. Hence,
the inventory turnover is 3.88 times per year {$330 COGS ÷ [($90 + $80) ÷ 2]}.
C. 5.33 times.
D. times.
Study Unit 2: Ratio Analysis | Subunit 3: Activity Measures
Fact Pattern: The information below pertains to Devlin Company.
Statement of Financial Position as of May 31 Income Statement for the year ended May
(in thousands)
Year
Year
Assets
Current assets
Cash
Trading securities
Accounts receivable (net)
Inventory
Prepaid expenses
Total current assets
Investments, at equity
Property, plant, and equipment
(net)
Intangible assets (net)
Total assets
Liabilities
Current liabilities
Notes payable
Accounts payable
Accrued expenses
Income taxes payable
Total current liabilities
Long-term debt
Deferred taxes
Total liabilities
Equity
Preferred stock, 6%, $100 par
value,
cumulative
Common stock, $10 par value
Additional paid-in capital --
common stock
Retained earnings
Total equity
Total liabilities and equity
(in thousands)
Year
Year
Net sales
Costs and expenses
Costs of goods sold
Selling, general, and
administrative
Interest expense
Income before taxes
Income taxes
Net income
Question: 71 Devlin Company’s asset turnover for the year ended May Year was
A. 0.08 times.
B. 0.46 times.
C. 0.67 times.
Answer (C) is correct.
Asset turnover equals net sales divided by average total assets. Consequently, the
asset turnover is .67 times per year {$480 net sales ÷ [($748 + $691) ÷ 2]}.
D. 0.83 times.
Study Unit 2: Ratio Analysis | Subunit 3: Activity Measures
Question: 72 The number of days’ sales in receivables is a measure of
A. Asset value.
B. Sales performance.
C. Profitability.
D. Liquidity.
Answer (D) is correct.
Turnover ratios are activity ratios that measure management’s efficiency in using
assets However the number of days’ sales in receivables (days in the year divided
by the receivables turnover ratio), also known as the average collection period, and
other turnover ratios are a measure of liquidity because these statistics show how
long it will take to turn inventory into cash.
Study Unit 2: Ratio Analysis | Subunit 3: Activity Measures
Question: 73 A change in credit policy has caused an increase in sales, an increase in discounts taken, a
reduction in the investment in accounts receivable, and a reduction in the number of doubtful
accounts. Based upon this information, we know that
A. Net profit has increased.
B. The average collection period has decreased.
Answer (B) is correct.
An increase in discounts taken accompanied by declines in receivables balances
and doubtful accounts all indicate that collections on the increased sales have been
accelerated. Accordingly, the average collection period must have declined. The
average collection period is a ratio calculated by dividing the number of days in a
year (365) by the receivable turnover. Thus, the higher the turnover, the shorter the
average collection period. The turnover increases when either sales (the numerator)
increase, or receivables (the denominator) decrease. Accomplishing both higher
sales and a lower receivables increases the turnover and results in a shorter
collection period.
C. Gross profit has declined.
D. The size of the discount offered has decreased.
Study Unit 2: Ratio Analysis | Subunit 3: Activity Measures
Fact Pattern: The Statement of Financial Position for King Products Corporation for the
fiscal years ended June 30, Year 2, and June 30, Year 1, is presented below. Net sales and cost
of goods sold for the year ended June 30, Year 2, were $600,000 and $440,000, respectively.
King Products Corporation
Statement of Financial Position
(in thousands)
June 30
Year Year
Cash
Marketable securities (at market)
Accounts receivable (net)
Inventories (at lower of cost or market)
Prepaid items
Total current assets
Land (at cost)
Building (net)
Equipment (net)
Patents (net)
Goodwill (net)
Total long-term assets
Total assets
Notes payable
Accounts payable
Accrued interest
Total current liabilities
Notes payable, 10% due 12/31/Year 7
Bonds payable, 12% due 6/30/Year 10
Total long-term debt
Total liabilities
Preferred stock -- 5% cumulative, $100 par, nonparticipating,
authorized, issued and outstanding, 2,000 shares
Common stock -- $10 par, 40,000 shares authorized,
30,000 shares issued and outstanding
Additional paid-in capital -- common
Retained earnings
Total equity
Total liabilities & equity
Question: 74 King Products Corporation’s inventory turnover ratio for the fiscal year ended at June 30, Year
2, was
A.
B.
Answer (B) is correct.
The inventory turnover ratio equals cost of sales divided by the average inventory.
Consequently, the inventory turnover is 4 times per year {$440,000 ÷ [($120,000 +
$100,000) ÷
C.
D.
Study Unit 2: Ratio Analysis | Subunit 3: Activity Measures
Fact Pattern: The Statement of Financial Position for King Products Corporation for the
fiscal years ended June 30, Year 2, and June 30, Year 1, is presented below. Net sales and cost
of goods sold for the year ended June 30, Year 2, were $600,000 and $440,000, respectively.
King Products Corporation
Statement of Financial Position
(in thousands)
June 30
Year Year
Cash $
Marketable securities (at market)
Accounts receivable (net)
Inventories (at lower of cost or market)
Prepaid items
Total current assets
Land (at cost)
Building (net)
Equipment (net)
Patents (net)
Goodwill (net)
Total long-term assets
Total assets
Notes payable
Accounts payable
Accrued interest
Total current liabilities
Notes payable, 10% due 12/31/Year 7
Bonds payable, 12% due 6/30/Year 10
Total long-term debt
Total liabilities
Preferred stock -- 5% cumulative, $100 par, nonparticipating,
authorized, issued and outstanding, 2,000 shares
Common stock -- $10 par, 40,000 shares authorized,
30,000 shares issued and outstanding
Additional paid-in capital -- common
Retained earnings
Total equity
Total liabilities & equity
Question: 75 King Products Corporation’s receivables turnover ratio for this period was
A.
B.
C.
D.
Answer (D) is correct.
The receivables turnover ratio equals net credit sales divided by the average
receivables balance. In this question, net sales must be used because the amount of
net credit sales is not given. Thus, the receivables turnover is 8 times per year
{$600,000 ÷ [($90,000 + $60,000) ÷ 2]}.
Study Unit 2: Ratio Analysis | Subunit 3: Activity Measures
Fact Pattern: The Statement of Financial Position for King Products Corporation for the
fiscal years ended June 30, Year 2, and June 30, Year 1, is presented below. Net sales and cost
of goods sold for the year ended June 30, Year 2, were $600,000 and $440,000, respectively.
King Products Corporation
Statement of Financial Position
(in thousands)
June 30
Year Year
Cash
Marketable securities (at market)
Accounts receivable (net)
Inventories (at lower of cost or market)
Prepaid items
Total current assets
Land (at cost)
Building (net)
Equipment (net)
Patents (net)
Goodwill (net)
Total long-term assets
Total assets
Notes payable
Accounts payable
Accrued interest
Total current liabilities
Notes payable, 10% due 12/31/Year 7
Bonds payable, 12% due 6/30/Year 10
Total long-term debt $
Total liabilities
Preferred stock -- 5% cumulative, $100 par, nonparticipating,
authorized, issued and outstanding, 2,000 shares
Common stock -- $10 par, 40,000 shares authorized,
shares issued and outstanding
Additional paid-in capital -- common
Retained earnings
Total equity
Total liabilities & equity
Question: 76 King Products Corporation’s average collection period for the fiscal year ended June 30, Year
2, using a 360-day year was
A. 36 days.
B. 45 days.
Answer (B) is correct.
The average collection period equals the number of days in a year divided by the
receivables turnover ratio. The receivables turnover was 8 times per year {$600,000
÷ [(90,000 + 60,000) ÷ 2]}. Thus, the average collection period is 45 days (360 ÷
)
C. 54 days.
D. 61 days.
Study Unit 2: Ratio Analysis | Subunit 3: Activity Measures
Question: 77 Based on the data presented below what is Beta Corporation’s cost of sales for the year?
Current ratio
Acid test ratio
Year-end current liabilities
Beginning inventory
Inventory turnover
A.
B.
C.
Answer (C) is correct.
The current ratio is the ratio of current assets to current liabilities. Since the current
ratio and current liabilities are known, current assets can be determined as follows:
Current assets ÷ Current liabilities = Current ratio
Current assets ÷ $600,000 =
Current assets = $600,000 × 3.5
=
Quick assets can be determined similarly:
Quick assets ÷ Current liabilities = Acid test ratio
Quick assets ÷ $600,000 =
Quick assets = $600,000 × 3.0
=
Assuming the company had no prepaid expenses, the difference between current
assets and quick assets is inventory.
Ending inventory = Current assets – quick assets
= $2,100,000 –
=
Once ending inventory is known, average inventory can be determined [($500,000
+ $300,000) ÷ 2 = $400,000], and, finally, cost of sales can be calculated:
Cost of sales ÷ Average inventory = Inventory turnover
Cost of sales ÷ $400,000 =
Cost of sales = $400,000 × 8.0
=
D.
Study Unit 2: Ratio Analysis | Subunit 3: Activity Measures
Question: 78 A change in credit policy has caused an increase in sales, an increase in discounts taken, a
decrease in the amount of bad debts, and a decrease in the investment in accounts receivable.
Based upon this information the company’s
A. Average collection period has decreased.
Answer (A) is correct.
An increase in discounts taken accompanied by declines in receivables balances
and doubtful accounts all indicate that collections on the increased sales have been
accelerated. Accordingly, the average collection period must have declined. The
average collection period is a ratio calculated by dividing the number of days in a
year (365) by the receivable turnover. Thus, the higher the turnover, the shorter the
average collection period. The turnover increases when either sales (the numerator)
increase, or receivables (the denominator) decrease. Accomplishing both higher
sales and a lower receivables increases the turnover and results in a shorter
collection period.
B. Percentage discount offered has decreased.
C. Accounts receivable turnover has decreased.
D. Working capital has increased.
Study Unit 2: Ratio Analysis | Subunit 3: Activity Measures
Question: 79 The ratio of sales to working capital is a measure of
A. Collectibility.
B. Financial leverage.
C. Liquidity.
Answer (C) is correct.
Like most ratios involving working capital, the working capital turnover (sales ÷
average working capital) is a measure of liquidity, which is the ability to meet
obligations as they mature. However, it is also an activity measure, and a high
turnover is preferable.
D. Profitability.
Study Unit 2: Ratio Analysis | Subunit 3: Activity Measures
Question: 80 A high sales-to-working-capital ratio could indicate
A. Unprofitable use of working capital.
B. Sales are not adequate relative to available working capital.
C. The firm is undercapitalized.
Answer (C) is correct.
A high sales-to-working-capital ratio is usually favorable because working capital,
by itself, is an unprofitable use of resources. A firm does not earn money by
holding cash, inventory, or receivables. Such assets should be minimized. However,
a high ratio of sales to working capital may indicate either very high sales (a good
situation) or a low supply of working capital (a potentially bad situation). Thus, a
high ratio could indicate that a firm is undercapitalized and does not have the
resources to invest in working capital.
D. The firm is not susceptible to liquidity problems.
Study Unit 2: Ratio Analysis | Subunit 3: Activity Measures
Fact Pattern: The data presented below show actual figures for selected accounts of McKeon
Company for the fiscal year ended May 31, Year 1, and selected budget figures for the Year 2
fiscal year McKeon’s controller is in the process of reviewing the Year 2 budget and
calculating some key ratios based on the budget. McKeon Company monitors yield or return
ratios using the average financial position of the company. (Round all calculations to three
decimal places if necessary.)
5/31/Year 2 5/31/Year 1
Current assets
Noncurrent assets
Current liabilities
Long-term debt
Common stock ($30 par value)
Retained earnings
Year 2
Operations
Sales*
Cost of goods sold
Interest expense
Income taxes (40% rate)
Dividends declared and paid in Year 2
Administrative expense
*All sales are credit sales.
Current Assets
5/31/Year 2 5/31/Year 1
Cash
Accounts receivable
Inventory
Prepaid expenses
Question: 81 The Year 2 receivables turnover ratio for McKeon Company is
A.
B.
C.
D.
Answer (D) is correct.
The receivables turnover ratio is equal to the total credit sales divided by the
average balance in accounts receivable. The average accounts receivable is equal to
$85,000 [($70,000 beginning balance + $100,000 ending balance) ÷ 2]. The
receivables turnover ratio is therefore equal to 4.118 ($350,000 credit sales ÷
$85,000 average receivables).
Study Unit 2: Ratio Analysis | Subunit 3: Activity Measures
Fact Pattern: The data presented below show actual figures for selected accounts of McKeon
Company for the fiscal year ended May 31, Year 1, and selected budget figures for the Year 2
fiscal year McKeon’s controller is in the process of reviewing the Year budget and
calculating some key ratios based on the budget. McKeon Company monitors yield or return
ratios using the average financial position of the company. (Round all calculations to three
decimal places if necessary.)
5/31/Year 2 5/31/Year 1
Current assets
Noncurrent assets
Current liabilities
Long-term debt
Common stock ($30 par value)
Retained earnings
Year 2
Operations
Sales*
Cost of goods sold
Interest expense
Income taxes (40% rate)
Dividends declared and paid in Year 2
Administrative expense
*All sales are credit sales.
Current Assets
5/31/Year 2 5/31/Year 1
Cash
Accounts receivable
Inventory
Prepaid expenses
Question: 82 Using a 365-day year McKeon’s inventory turnover is
A. 171 days.
Answer (A) is correct.
Inventory turnover in terms of days is determined by dividing 365 by the inventory
turnover ratio. The inventory turnover ratio is equal to the $160,000 cost of goods
sold divided by the $75,000 average balance in inventory [($80,000 beginning
balance + $70,000 ending balance) ÷ 2]. Hence, the inventory turnover ratio is
2.133 times per year. Dividing 365 by 2.133 results in an inventory turnover of
171 days.
B. 160 days.
C. 183 days.
D. 78 days.
Study Unit 2: Ratio Analysis | Subunit 3: Activity Measures
Fact Pattern: The data presented below show actual figures for selected accounts of McKeon
Company for the fiscal year ended May 31, Year 1, and selected budget figures for the Year 2
fiscal year McKeon’s controller is in the process of reviewing the Year budget and
calculating some key ratios based on the budget. McKeon Company monitors yield or return
ratios using the average financial position of the company. (Round all calculations to three
decimal places if necessary.)
5/31/Year 2 5/31/Year 1
Current assets
Noncurrent assets
Current liabilities
Long-term debt
Common stock ($30 par value)
Retained earnings
Year 2
Operations
Sales*
Cost of goods sold
Interest expense
Income taxes (40% rate)
Dividends declared and paid in Year 2
Administrative expense
*All sales are credit sales.
Current Assets
5/31/Year 2 5/31/Year 1
Cash
Accounts receivable
Inventory
Prepaid expenses
Question: 83 McKeon Company’s total asset turnover ratio for Year is
A.
B.
Answer (B) is correct.
Total asset turnover ratio is equal to $350,000 sales divided by average total assets.
The amount of average total assets is equal to the average of beginning total assets
of $435,000 ($180,000 current assets + $255,000 noncurrent assets) and ending
total assets of $485,000 ($210,000 current assets + $275,000 noncurrent assets).
The total asset turnover ratio is therefore equal to .761 ($350,000 ÷ $460,000).
C.
D.
Study Unit 2: Ratio Analysis | Subunit 3: Activity Measures
Fact Pattern: The year-end financial statements for Queen Bikes reflect the data presented as
follows Ten percent of Queen’s net sales are in cash
Year 1 Year 2 Year 3
Net sales 1,500 units at $100 1,200 units at $100 1,200 units at $125
Ending inventory 100 units at $50 100 units at $50 100 units at $50
Average receivables
Net income
Question: 84 Queen’s receivables turnover ratios for Year and Year are
A. 10.8 and 9.0, respectively.
B. 9.0 and 9.375, respectively.
Answer (B) is correct.
The receivables turnover ratio equals net credit sales divided by average accounts
receivables. For Year 2 the calculation is [($120,000 × .9) ÷ $12,000] = 9.0, and for
Year it is [($150,000 × .9) ÷ $14,400] = 9.375.
C. 10.00 and 10.42, respectively.
D. 1.00 and 1.04, respectively.
Study Unit 2: Ratio Analysis | Subunit 3: Activity Measures
Fact Pattern: The year-end financial statements for Queen Bikes reflect the data presented as
follows Ten percent of Queen’s net sales are in cash
Year 1 Year 2 Year 3
Net sales 1,500 units at $100 1,200 units at $100 1,200 units at $125
Ending inventory 100 units at $50 100 units at $50 100 units at $50
Average receivables
Net income
Question: 85 Queen’s inventory turnover ratios for Year and Year are
A. 24 and 24, respectively.
B. 12 and 18, respectively.
C. 12 and 12, respectively.
Answer (C) is correct.
The cost of the 1,200 units sold in Year 2 at $50 each would have been $60,000.
Dividing the $60,000 cost of sales by the $5,000 average inventory results in a
turnover of 12. The 1,200 units sold in Year 3 also would have cost $60,000 and the
turnover would again be 12.
D. 18 and 18, respectively.
Fact Pattern: Selected data from White Corporation’s financial statements for the year ended
November 30, Year 2, are as follows (all sales are on credit).
Current ratio
Quick ratio
Current liabilities
Accounts receivable turnover
Merchandise inventory turnover
Rate of return on assets
Selected account balances at November 30, Year
Accounts receivable
Merchandise inventory
Year 2 operations
Sales
Cost of goods sold
Question: 86 Assuming that prepaid expenses are immaterial White’s ending merchandise inventory is
A.
B.
Answer (B) is correct.
Both the current ratio and quick ratio are given. The only difference between these
two ratios is that inventory is included in the calculation of the current ratio, but not
the quick ratio. The current ratio equals current assets divided by current liabilities.
The current ratio is 1.4 times the current liabilities. Thus, current assets must be
( 450,000 × 1.4). The quick ratio is .86 [(current assets – merchandise
inventory) ÷ current liabilities of $450,000]. Quick assets are $387,000 ($450,000 ×
.86). Since the only difference between current assets and quick assets is
merchandise inventory (prepaid expenses are immaterial), merchandise inventory
must be $243,000 ($630,000 current assets – $387,000 of quick assets).
C.
D.
Study Unit 2: Ratio Analysis | Subunit 3: Activity Measures
Fact Pattern: Selected data from White Corporation’s financial statements for the year ended
November 30, Year 2, are as follows (all sales are on credit).
Current ratio
Quick ratio
Current liabilities
Accounts receivable turnover
Merchandise inventory turnover
Rate of return on assets
Selected account balances at November 30, Year
Accounts receivable
Merchandise inventory
Year 2 operations
Sales
Cost of goods sold
Question: 87 White’s balance in accounts receivable at November 30, Year 2, is
A.
Answer (A) is correct.
The accounts receivable turnover (3.65) equals sales divided by the average balance
in accounts receivable. Thus, dividing the $1,241,000 of sales by 3.65 gives an
average receivables balance of $340,000. The average receivables balance is the
average of the beginning and ending balances. Since the beginning balance was
$355,000, the ending balance must be $325,000 [($340,000 × 2) –
B.
C.
D.
Study Unit 2: Ratio Analysis | Subunit 3: Activity Measures
Fact Pattern: Selected data from White Corporation’s financial statements for the year ended
November 30, Year 2, are as follows (all sales are on credit).
Current ratio
Quick ratio
Current liabilities
Accounts receivable turnover
Merchandise inventory turnover
Rate of return on assets
Selected account balances at November 30, Year
Accounts receivable
Merchandise inventory
Year 2 operations
Sales
Cost of goods sold
Question: 88 The approximate number of days in White’s operating cycle is
A.
B.
C.
D.
Answer (D) is correct.
The operating cycle is the time required to convert cash into inventory, sell that
inventory for receivables, and collect cash in payment of the receivables. The
operating cycle is therefore a cash-to-cash cycle: the time that a company holds
inventory before sale plus the time that it holds receivables before collection.
Dividing the receivables turnover of 3.65 into 365 days gives the average credit
period (100 days). Dividing the inventory turnover of 3.30 into 365 days gives the
average holding period (110.6 days). The sum of the inventory holding period and
the credit period is the operating cycle (210.6 days).
Study Unit 2: Ratio Analysis | Subunit 3: Activity Measures
Fact Pattern:
Selected data from Ostrander Corporation’s financial statements for the years indicated are
presented in thousands.
Year 2 Operations
Net credit sales
Cost of goods sold
Interest expense
Income tax
Gain on disposal of a
segment
(net of tax)
Administrative expense
Net income
December
Year 2 Year 1
Cash
Trading securities
Accounts receivable (net)
Merchandise inventory
Tangible fixed assets
Total assets
Current liabilities
Total liabilities
Common stock
outstanding
Retained earnings
Question: 89 The number of days of receivables (using 365 days) for Ostrander Corporation in Year 2 is
A. 18.10 days.
Answer (A) is correct.
The average number of days’ sales in receivables equals days divided by the
accounts receivable turnover. The accounts receivable turnover is determined by
dividing the average accounts receivable balance into sales. The average balance
was $207 [($204 + $210) ÷ 2]. Dividing $207 into sales of $4,175 produces an
average turnover rate of times Thus the average days’ sales in receivables
was 18.097 days (365 ÷ 20.1691).
B. 20.17 days.
C. 17.83 days.
D. 18.36 days.
Study Unit 2: Ratio Analysis | Subunit 3: Activity Measures
Question: 90 In computing inventory turnover, the preferred base to use is the
A. Sales base because it is more likely to reflect a change in trend.
B. Sales base because it provides turnover rates that are considerably higher.
C. Cost of sales base because it is not affected by the method used to value inventory.
D. Cost of sales base because it eliminates any changes due solely to sales price changes.
Answer (D) is correct.
Inventory turnover is measured by dividing the cost of sales by average inventories.
Cost of sales is used rather than sales because the cost of sales base eliminates any
changes caused solely by sales price changes. Furthermore, using sales in the
numerator is inconsistent with valuing inventories at cost in the denominator.
Study Unit 2: Ratio Analysis | Subunit 3: Activity Measures
Question: 91 Selected data from Sheridan Corporation’s year-end financial statements are presented below.
The difference between average and ending inventory is immaterial.
Current ratio
Quick ratio
Current liabilities
Inventory turnover (based on cost of goods sold) 8 times
Gross profit margin
Sheridan’s net sales for the year were
A.
Answer (A) is correct.
Net sales can be calculated indirectly from the inventory turnover ratio and the
other ratios given. If the current ratio is 2.0, and current liabilities are $120,000,
current assets must be $240,000 (2.0 × $120,000). Similarly, if the quick ratio is
1.5, the total quick assets must be $180,000 (1.5 × $120,000). The only major
difference between quick assets and current assets is that inventory is not included
in the definition of quick assets. Consequently, ending inventory must be $60,000
($240,000 – $180,000). The inventory turnover ratio (COGS ÷ average inventory)
is 8. Thus, cost of goods sold must be 8 times average inventory, or $480,000,
given no material difference between average and ending inventory. If the gross
profit margin is 40%, the cost of goods sold percentage is 60%, cost of goods sold
equals 60% of sales, and net sales must be $800,000 ($480,000 ÷ 60%).
B.
C.
D.
Study Unit 2: Ratio Analysis | Subunit 3: Activity Measures
Fact Pattern:
Lisa, Inc.
Statement of Financial Position
December 31, Year 2
(000s)
Assets Year 2 Year 1
Current assets:
Cash
Trading securities
Accounts receivable (net)
Inventories (at lower of cost or market)
Prepaid items
Total current assets
Long-term investments:
Securities (at cost)
Property, plant, & equipment:
Land (at cost)
Building (net)
Equipment (net)
Intangible assets
Patents (net)
Goodwill (net)
Total long-term assets
Total assets
Liabilities & Shareholders’ Equity
Current liabilities:
Notes payable
Accounts payable
Accrued interest
Total current liabilities
Long-term debt:
Notes payable 10% due 12/31/Year 9
Bonds payable 12% due 12/31/Year 8
Total long-term debt
Total liabilities
Shareholders’ equity:
Preferred -- 5% cumulative, $100 par,
non-participating, 1,000 shares
authorized, issued and outstanding
Common -- $10 par 20,000 shares
authorized, 15,000 issued and
outstanding shares
Additional paid-in capital -- common
Retained earnings
Total shareholders’ equity
Total liabilities & equity
Question: 92 Assume net credit sales and cost of goods sold for Year 2 were $300,000 and $220,000,
respectively Lisa Inc ’s accounts receivable turnover for Year was
A. 4.9 times.
B. 5.9 times.
C. 6.7 times.
D. 8.0 times.
Answer (D) is correct.
The accounts receivable turnover is computed by dividing the net credit sales by
average accounts receivable. The average is $37,500 [($45,000 + $30,000) ÷ 2].
Hence, the turnover is 8.0 ($300,000 ÷ $37,500).
Study Unit 2: Ratio Analysis | Subunit 3: Activity Measures
Fact Pattern:
Lisa, Inc.
Statement of Financial Position
December 31, Year 2
(000s)
Assets Year 2 Year 1
Current assets:
Cash
Trading securities
Accounts receivable (net)
Inventories (at lower of cost or market)
Prepaid items
Total current assets
Long-term investments:
Securities (at cost)
Property, plant, & equipment:
Land (at cost)
Building (net)
Equipment (net)
Intangible assets
Patents (net)
Goodwill (net)
Total long-term assets
Total assets
Liabilities & Shareholders’ Equity
Current liabilities:
Notes payable
Accounts payable
Accrued interest
Total current liabilities
Long-term debt:
Notes payable 10% due 12/31/Year 9
Bonds payable 12% due 12/31/Year 8
Total long-term debt
Total liabilities
Shareholders’ equity:
Preferred -- 5% cumulative, $100 par,
non-participating, 1,000 shares
authorized, issued and outstanding
Common -- $10 par 20,000 shares
authorized, 15,000 issued and
outstanding shares
Additional paid-in capital -- common
Retained earnings
Total shareholders’ equity
Total liabilities & equity
Question: 93 Assume net credit sales and cost of goods sold for Year 2 were $300,000 and $220,000,
respectively Lisa Inc ’s average collection period for Year 2, using a 360-day year, was
A. 36 days.
B. 45 days.
Answer (B) is correct.
The average collection period is calculated by dividing 360 days by the accounts
receivables turnover of 8.0 {$300,000 ÷ [($45,000 + 30,000) ÷ 2]}. Thus, the
average collection period is 45 days (360 ÷ 8.0).
C. 54 days.
D. 61 days.
Study Unit 2: Ratio Analysis | Subunit 3: Activity Measures
Fact Pattern:
Lisa, Inc.
Statement of Financial Position
December 31, Year 2
(000s)
Assets Year 2 Year 1
Current assets:
Cash
Trading securities
Accounts receivable (net)
Inventories (at lower of cost or market)
Prepaid items
Total current assets
Long-term investments:
Securities (at cost)
Property, plant, & equipment:
Land (at cost)
Building (net)
Equipment (net)
Intangible assets
Patents (net)
Goodwill (net)
Total long-term assets
Total assets
Liabilities & Shareholders’ Equity
Current liabilities:
Notes payable
Accounts payable
Accrued interest
Total current liabilities
Long-term debt:
Notes payable 10% due 12/31/Year 9
Bonds payable 12% due 12/31/Year 8
Total long-term debt
Total liabilities
Shareholders’ equity:
Preferred -- 5% cumulative, $100 par,
non-participating, 1,000 shares
authorized, issued and outstanding
Common -- $10 par 20,000 shares
authorized, 15,000 issued and
outstanding shares
Additional paid-in capital -- common
Retained earnings
Total shareholders’ equity
Total liabilities & equity
Question: 94 Assume sales and cost of goods sold for Year 2 were $300,000 and $220,000, respectively.
Lisa Inc ’s inventory turnover for Year was
A. 3.7 times.
B. 4.0 times.
Answer (B) is correct.
The inventory turnover is computed by dividing cost of goods sold by average
inventory. Consequently, the turnover is 4 times {$220,000 ÷ [($60,000 + $50,000)
÷ 2]}.
C. 4.4 times.
D. 5.0 times.
Study Unit 2: Ratio Analysis | Subunit 3: Activity Measures
Question: 95 A ratio that measures the movement of current assets is
A. Working capital turnover.
Answer (A) is correct.
Ratios that measure movement of assets are known as turnover ratios. Working
capital turnover measures the firm’s use of working capital in relation to sales.
Sales are divided by average working capital. A high turnover is preferred.
B. Working capital to total assets.
C. Return on owners’ equity
D. The current ratio.
Study Unit 2: Ratio Analysis | Subunit 3: Activity Measures
Fact Pattern:
RST Corporation Comparative Income
Statements for the Years 5 and 6
Year 6 Year 5
Sales (all are credit)
Cost of goods sold
Gross profit
Selling and administrative expenses
Income before interest and income taxes
Interest expense
Income before income taxes
Income tax expense
Net income
RST Corporation
Comparative Balance Sheets
End of Years 5 and 6
Assets Year 6 Year 5
Current assets:
Cash
Short-term marketable investments
Accounts receivable (net)
Inventory
Total current assets
Noncurrent assets:
Long-term investments
Property, plant, and equipment
Intangibles
Total assets
Liabilities and Stockholders’ Equity
Current liabilities:
Accounts payable
Accrued payables
Total current liabilities
Long-term Liabilities:
0% Bonds payable, due in Year 12
Total liabilities
Stockholders’ equity:
Common stock, 2,400 shares, $10 par
Retained earnings
Total stockholders’ equity
Total liabilities and stockholders’ equity
The market value of RST’s common stock at the end of Year Six was per share
Question: 96 RST’s accounts receivable turnover for Year is
A. 19 times.
Answer (A) is correct.
The accounts receivable turnover equals net credit sales divided by average trade
receivables (net). In Year 6, the accounts receivable turned over 19 times
{$285,000 ÷ [($16,000 ending A/R + $14,000 beginning A/R) ÷ 2]}.
B. 17.8 times
C. 16.2 times.
D. 10 times.
Study Unit 2: Ratio Analysis | Subunit 3: Activity Measures
Fact Pattern: A company sells 10,000 skateboards a year at $66 each. All sales are on credit,
with terms of 3/10, net 30, that is, a 3% discount if payment is made within 10 days; otherwise
full payment is due at the end of 30 days. One half of the customers are expected to take
advantage of the discount and pay on day 10. The other half are expected to pay on day 30.
Sales are expected to be uniform throughout the year for both types of customers.
Question: 97 What is the expected average collection period for the company?
A. 5 days.
B. 10 days.
C. 15 days.
D. 20 days.
Answer (D) is correct.
The average collection period is the average time it takes to receive payment from
customers. Because one-half of the customers will pay on day 10 and half will pay
on day 30, the average collection period is 20 days [.5(10 days) + .5(30 days)].
Study Unit 2: Ratio Analysis | Subunit 3: Activity Measures
Fact Pattern: A company sells 10,000 skateboards a year at $66 each. All sales are on credit,
with terms of 3/10, net 30, that is, a 3% discount if payment is made within 10 days; otherwise
full payment is due at the end of 30 days. One half of the customers are expected to take
advantage of the discount and pay on day 10. The other half are expected to pay on day 30.
Sales are expected to be uniform throughout the year for both types of customers.
Question: 98 Assume that the average collection period is 25 days. After the credit policy is well
established, what is the expected average accounts receivable balance for the company at any
point in time, assuming a 365-day year?
A.
B.
C.
D.
Answer (D) is correct.
The expected average accounts receivable balance equals the average collection
period times the credit sales per day. Thus, the average accounts receivable balance
is $45,205.48 {[(10,000 units sold on credit × $66 price) ÷ 365 days] × 25 days}.
The foregoing calculation assumes that receivables are recorded at their gross
amounts.
Study Unit 2: Ratio Analysis | Subunit 3: Activity Measures
Fact Pattern: The following inventory and sales data are available for the current year for
Volpone Company. Volpone uses a 365-day year when computing ratios.
November 30, November 30,
Year 2
Year 1
Net credit sales
Gross receivables
Inventory
Cost of goods sold
Question: 99 Volpone Company’s average number of days to sell inventory for Year is
A.
B.
C.
D.
Answer (D) is correct.
The average days to sell inventory equals 365 days divided by the inventory
turnover (cost of goods sold ÷ average inventory). Thus, turnover is 5.0345 times
{$4,380,000 COGS ÷ [($960,000 + $780,000) ÷ 2]}. Average days to sell inventory
is 72.5 days (365 ÷ 5.0345).
Study Unit 2: Ratio Analysis | Subunit 3: Activity Measures
Fact Pattern: The following inventory and sales data are available for the current year for
Volpone Company. Volpone uses a 365-day year when computing ratios.
November 30, November 30,
Year 2
Year 1
Net credit sales
Gross receivables
Inventory
Cost of goods sold
Question: 100 Volpone Company’s operating cycle for Year is
A.
B.
C.
Answer (C) is correct.
The operating cycle is the length of time it takes a company to complete normal
operating activities. Thus, the operating cycle is a cash-to-cash cycle equivalent to
the average time that inventory is held plus the average time that receivables are
held. Volpone holds its inventory 72.50 days [365 days ÷ ($4,380,000 COGS ÷
$870,000 average inventory)] and its receivables 19.71 days [365 days ÷
($6,205,000 sales ÷ $335,000 average receivables)]. Its operating cycle is 92.21
days (72.5 + )
D.
Study Unit 2: Ratio Analysis | Subunit 3: Activity Measures
Fact Pattern: The following inventory and sales data are available for the current year for
Volpone Company. Volpone uses a 365-day year when computing ratios.
November 30, November 30,
Year 2
Year 1
Net credit sales
Gross receivables
Inventory
Cost of goods sold
Question: 101 Volpone Company’s average number of days to collect accounts receivable for Year is
A.
B.
C.
Answer (C) is correct.
The average collection period equals 365 days divided by the receivables turnover
(net credit sales ÷ average accounts receivable). Turnover is 18.52 times
{$6,205,000 sales ÷ [($350,000 + $320,000) ÷ 2]}. Hence, the average collection
period is 19.71 days (365 ÷ 18.52).
D.
Study Unit 2: Ratio Analysis | Subunit 3: Activity Measures
Question: 102 The Irwin Corporation has million per year in credit sales The company’s average day’s
sales outstanding is 40 days. Assuming a 360-day year what is Irwin’s average amount of
accounts receivable outstanding?
A.
B.
Answer (B) is correct.
Dividing $3 million of sales by 360 days results in an average of $8,333.33 per day.
Multiplying the average daily sales by the 40 days outstanding results in $333,333.
C.
D.
Study Unit 2: Ratio Analysis | Subunit 3: Activity Measures
Question: 103 Last year Johnson Company’s days’ sales in receivables was days This year days’ sales in
receivables is 91.25 days. Over the same time period, sales have declined by 20%. In this
period of time, what has happened to the level of Johnson Company’s accounts receivable?
A. Accounts receivables have increased.
B. Accounts receivables have decreased.
C. There has been no change in accounts receivable.
Answer (C) is correct.
The changes can best be understood by using illustrative numbers. If prior year
sales were $1,000, a 20% decline results in current year sales of $800. The formula
for days’ sales in receivables can be restated as follows:
=
=
=
Current Year Prior Year
365 × (AR ÷ $800) = 365 × (AR ÷ $1,000) =
365 × AR = 365 × AR =
AR = AR =
Thus, there was no change in the balance of accounts receivables.
D. There is not enough information provided to make a determination.
Study Unit 2: Ratio Analysis | Subunit 3: Activity Measures
Question: 104 The selected data pertain to a company at December 31:
Quick assets
Acid test ratio 2.6 to 1
Current ratio 3.5 to 1
Net sales for the year
Cost of sales for the year
Average total assets for the year
The company’s asset turnover ratio for the year is
A.
B.
C.
D.
Answer (D) is correct.
The asset turnover ratio equals $1,800,000 of net sales divided by $1,200,000 of
average total assets. The asset turnover ratio is therefore equal to 1.5.
Study Unit 2: Ratio Analysis | Subunit 3: Activity Measures
Question: 105 The ratio that measures a firm’s ability to generate earnings from its resources is
A. Days’ sales in inventory
B. Sales to working capital.
C. Days’ sales in receivables
D. Asset turnover.
Answer (D) is correct.
Asset turnover measures the level of capital investment relative to sales volume. It
is a measure of how well a company uses its assets. A high turnover is preferable
because it signifies that a given level of resources is being used to generate greater
sales.
Study Unit 2: Ratio Analysis | Subunit 3: Activity Measures
Fact Pattern: Broomall Corporation has decided to include certain financial ratios in its yearend
annual report to shareholders. Selected information relating to its most recent fiscal year is
provided below.
Cash
Accounts receivable:
– Beginning of year
– End of year
Prepaid expenses
Inventory:
– Beginning of year